13.07.2015 Views

2155741412188746

2155741412188746

2155741412188746

SHOW MORE
SHOW LESS

You also want an ePaper? Increase the reach of your titles

YUMPU automatically turns print PDFs into web optimized ePapers that Google loves.

MathématiquesTerm STMGLivre du professeur


© HACHETTE LIVRE 2013, 43 quai de Grenelle 75905 Paris Cedex 15Tous droits de traduction, de reproduction et d’adaptation réservés pour tous pays.Le Code de la propriété intellectuelle n’autorisant, aux termes des articles L. 122-4 et L. 122-5, d’une part, que les « copiesou reproductions strictement réservées à l’usage privé du copiste et non destinées à une utilisation collective », et, d’autrepart, que « les analyses et les courtes citations » dans un but d’exemple et d’illustration, « toute représentation oureproduction intégrale ou partielle, faite sans le consentement de l’auteur ou de ses ayants droit ou ayants cause, estillicite ».Cette représentation ou reproduction, par quelque procédé que ce soit, sans autorisation de l’éditeur ou du Centre françaisde l’exploitation du droit de copie (20, rue des Grands-Augustins 75006 Paris), constituerait donc une contrefaçonsanctionnée par les articles 425 et suivants du Code pénal.


SommaireProgressions possibles sur l’année p. 4Devoirs surveillés par chapitre (barème et résultats) p. 7Chapitre 1 : Statistique p.11Chapitre 2 : Évolution – Taux moyen p. 25Chapitre 3 : Probabilité conditionnelle p. 40Chapitre 4 : Étude de fonctions p.52Chapitre 5 :Lois de probabilité p. 69Chapitre 6 : Suites p. 83Chapitre 7 : Échantillonnage et estimation p. 99


Maths Terminale STMG : Progressions possibles sur l’annéeOn peut considérer qu’une année scolaire se décompose en 30 semaines. D’où une proposition de pland’une année scolaire :7 semaines avant les vacances de Toussaint2 semaines au retour pour un Bac blanc et la fin du 1 er trimestre5 semaines avant les vacances de Noëlde 6 à 8 semaines avant les vacances d’hiver et la fin du 2 e trimestrede 6 à 8 semaines avant les vacances de Printemps et Bac Blanc2 semaines pour finir le 3 e trimestrele reste de l’année pour les révisions du BacLe programme de maths est écrit en décomposant le contenu en trois parties : Information chiffrée, Suiteset Fonctions, Statistique et Probabilités. Cependant, les élèves ont besoin de temps pour assimiler etn’aiment pas toujours passer deux mois sur le même thème ! Nous proposons donc une progression ensuivant le livre et deux progressions « en spirale ».1 re progressionL’ordre proposé n’est pas celui du programme afin de pouvoir utiliser l’ouvrage dans l’ordre des chapitresen découpant les trois parties du programme.En fin d’année, les 5 sujets de Bac du livre et les 2 sujets complémentaires à télécharger permettent larévision de tous les chapitres.On peut aussi réserver les exercices « Pour approfondir » aux meilleurs éléments de la classe et fairetravailler les exercices type Bac résolus du livre à chaque chapitre pour les élèves ayant besoin de plus detemps.Chapitre 1 : 3 semaines dont 1 h de contrôle1 ertrimestreChapitre 2 : 3 semaines dont 1 h de contrôleChapitre 3 : 3 semaines dont 1h de contrôleChapitre 4 : 4 semaines dont 1 h de contrôle(garder une partie des exercices du cours 5 pour la fin d’année)2 etrimestreChapitre 5 : 5 semaines dont 1 h de contrôle(faire faire des exos sur les fonctions entre le cours 2 et lecours 3)Chapitre 6 : 1 semaine pour le cours 1si 3 h de Bac Blanc : Sujet 1 possibleChapitre 6 : 4 semaines dont 1 h de contrôlesi 3 h de Bac Blanc : sujets 2 ou 43 etrimestreChapitre 7 : 3 semaines dont 1 h de contrôlesi 3 h de Bac Blanc : sujets 3 ou 5Sujets de révision pour le BacLivre du professeur – Mathématiques Term STMG © Hachette Livre 2013 4


2 e progression : en spirale en commençant par le chapitre 1On peut compter une semaine pour chaque regroupement dans un chapitre : par exemple dans le Chapitre 1 onregroupe les cours 1 e et 2.trimestre 1 er 2 e 3 echapitre 1 : cours 1et 2chapitre 4 : cours 1 et 2chapitre 2 : cours 1 et 2chapitre 3 : cours 1chapitre 5 : cours 1chapitre 6 : cours 1 et 2chapitre 1 : cours 3 et 4chapitre 4 : cours 3chapitre 2 : cours 3 et 4chapitre 3 : cours 2 et 3Bac Blanc : sujet 1chapitre 5 : cours 3 et 4chapitre 6 : cours 3chapitre 7 : cours 1chapitre 4 : cours 4chapitre 6 : cours 4Bac Blanc : sujets 2 ou 4chapitre 7 : cours 2chapitre 4 :cours 5chapitre 6 : cours 5DSutiliser les exercices 1 ou 2des chapitres 1 2 3 4 ou 5utiliser les exercices 3 ou 4des chapitres de 1 2 3 ou 5utiliser les exercices 3 ou 4des chapitres 4 6 ou 7fin 1 er trimestre fin 2 e trimestre Révision BACLivre du professeur – Mathématiques Term STMG © Hachette Livre 2013 5


3 e progression : en spirale en commençant par le chapitre 2On peut compter une semaine pour chaque regroupement dans un chapitre : par exemple dans lechapitre 2 on regroupe les cours 1 e et 2trimestre 1 er 2 e 3 eDSchapitre 2 : cours 1et 2chapitre 1 : cours 1 et 2chapitre 6 : cours 1chapitre 3 : cours 1chapitre 4 : cours 1 et 2chapitre 5 : cours 1 et 2utiliser les exercices 1 ou 2des chapitres 1 2 4 ou 5ou exercice 1 du chapitre 3chapitre 1 : cours 3 et 4chapitre 6 : cours 2chapitre 3 : cours 2 et 3chapitre 4 : cours 3chapitre 5 : cours 3 et 4chapitre 2 : cours 3 et 4Bac Blanc : sujet 1chapitre 6 : cours 3utiliser les exercices 3 ou 4des chapitres de 1 2 3 ou 5ou les exercices 1 ou 2 deschapitres 3 et 6chapitre 7 : cours 1chapitre 4 : cours 4chapitre 6 : cours 4Bac Blanc : sujets 2 ou 4chapitre 7 : cours 2chapitre 4 :cours 5chapitre 6 : cours 5utiliser les exercices 3 ou 4des chapitres 3 4 6 ou 7fin 1 er trimestre fin 2 e trimestre Révision BACLivre du professeur – Mathématiques Term STMG © Hachette Livre 2013 6


T STMGDevoirs surveillés par chapitreTous ces sujets de 1 h disponibles en téléchargement sur le site d’Hachette Education sont composésde 4 exercices de 5 points chacun, dont au moins un QCM.Le premier exercice porte sur le début du chapitre. Le 3 e exercice porte sur la totalité et peut être unQCM.Le dernier exercice est toujours contextualisé.Suivant la progression choisie dans l'année, en spirale ou dans l'ordre du livre, le professeur pourraconstruire un Devoir Surveillé en prenant un exercice à la place d'un autre.Barèmes et RésultatsDS Chapitre 1Exercice 11. c) 2. c) 3. a) 4. c) 5. b)Exercice 21. 0,32 million d’euros, soit 320 milliers d’euros enmoyenne par an. (1 point)2. 0,32 est le coefficient directeur de la droite (AB) .(0,5 point)3. y = 0,32 ( x – 2004 ) + 1,75 y = 0,32 x – 639,53 . (0,5 point)4. Le point de coordonnées ( 2007 ; 2,83 ) n’est pas surla droite (AB) , car :0,32 × 2007 – 639,53 = 2,71 2,83 . (1 point)5. a) 3,35 millions d’euros. (1 point)b) La prévision est justifiée :0,32 × 2016 -639,53 = 5,59 . (1 point)Exercice 31. b) 2. c) 3. a) 4. c) 5. a)Exercice 41. G ( 3 ; 28886 ) . (1 point)2. y = 1128,6x + 25 500 . (1 point)3. a) La droite d’équation y = 1 150x + 25 500 passepar A ( 0 ; 25 500 ) et B ( 6 ; 32 400) . (1,5 point)b) 37 000 nuitées. (0,5 point)c) 1 150 × 10 + 25 500 = 37 000 . (1 point)y3600035000340003300032000310003000029000280002700026000 A25000-1 0 1GB2 3 4 5 6 7 8 9 10DxDS Chapitre 2Exercice 11. a) En D2 : = C2/B2 . (0,5 point)En E4 : = ( D4 – 1 ) * 100 . (0,5 point)b) En D2 : CM ≈ 1,0869 et en E4 : t = – 3,08 .De 2030 à 2050, on prévoit que la population de l’Indesera multipliée par 1,0869 et la population des États-Unis diminuera de 3,08 % . (2 points)2. Tableau des résultats (2 points)Exercice 21. CM global = 0,966 . (1 point)2. À la suite de ces trois évolutions, la cote ne revientpas à sa valeur de départ. (0,5 point)3. T : – 3,4 % . (1 point)14. CM réciproque = ≈ 1,0352 .0,966D’où : T ’ : ≈ 3,52 % . (1,5 point)18 3545. Valeur initiale : = 19 000 € . (1 point)0,966Exercice 31. b) 2. a) 3. c) 4. b) 5. a)Exercice 41. 26 469 nuitées. (1 point)2. De 2004 à 2006, augmentation de 12,48 % . (1 point)3. 114,87 . (1 point)4. CM moyen = 1,2726 1 6 ≈ 1,04100 . Entre 2004 et 2006,augmentation moyenne de 4,1 % par an. (2 points)Livre du professeur – Mathématiques Term STMG © Hachette Livre 2013 7


DS Chapitre 3DS Chapitre 4P(H ∩ C)5. P C (H) = = 0,54Partie BP(C) 0,821. a) et c) . (1 point)Exercice 11.Tableau complété (2 points)Exercice 11. a) f ’(x) = – 0,2 x + 1 . (1 point)défectueuse en bonb) Tableau de variations (2 points)total(D) étatx 1 5 10Bordeaux (B) 160 3200 3360 f ’(x) + 0 -Grenoble (G) 66 1200 1266 f(x)5,533Lille (L) 154 3500 36542. a) f(2) = 4,6 . Pour la fabrication et la vente de 200total 380 7900 8280 bracelets, l’artisan réalise 4 600 € de bénéfice. (1 point)2. a) P(B) ≈ 0,406 . (0,5 point)b) Le bénéfice maximal de 5 500 € est obtenu par lab) P(D) ≈ 0,046 . (0,5 point)fabrication et la vente de 500 bracelets. (1 point)c) B ⋂ D : « l’alarme choisie est fabriquée à Bordeauxet est défectueuse » . P( B ⋂ D ) ≈ 0,019 . (1 point)Exercice 2d) P( B ⋃ D ) ≈ 0,433 . (1 point)1. c) 2. a) 3. b) 4. c) 5. a)Exercice 3Exercice 21. b) 2. a) 3. c) 4. a) 5. b)1. f ’(x) = 6x – 24 – 150x 2. (1 point)2. a) On résout l’équation x² + x + 5 = 0 . = – 19 .Exercice 3Comme le discriminant est négatif, l’équation n’a pas de1. A ⋂ S : « Le touriste interrogé a voyagé en avion et solution et la parabole d’équation y = x² + x + 5 estest resté plus d’une semaine en Angleterre ». (0,5 point) située au dessus de l’axe des abscisses.A ⋃ S : « Le touriste interrogé a voyagé en avion ou est Donc P(x) = x² + x + 5 est toujours strictement positifresté plus d’une semaine en Angleterre ». (0,5 point) sur donc aussi sur [ 1 ; 15 ] . (1 point)P( A ⋂ S ) = 0,06 et P( A ⋃ S ) = 0,64 . (1 point) b) On considère f ’(x) sous forme de quotient factorisé2. a) Arbre complété (1 point)donné par la calculatrice formelle.Sur [ 1 ; 15 ] , on a 6 > 0 ; x² + x + 5 > 0 et x² > 0 . f ’(x)est du signe de x – 5 sur l’intervalle [ 1 ; 15 ] . (1 point)3. a) Tableau de variations (1 point)x 1 5 15f ’(x) - 0 +229425f(x)b) P(S) = 0,3 × 0,2 + 0,5 × 0,6 + 0,2 × 0,2 = 0,4 .85(0,5 point)b) Pour 5 tonnes de gravier commercialisées par jour, leP(B ∩ S)3. P S (B) = = 0,16≈ 0,267 . (1,5 point)coût moyen est minimum et vaut 85 € la tonne. (1 point)P(S) 1−0,4Exercice 4Exercice 41. H : « le salarié choisi est une femme ».Partie AP(H) = 0,4 . (0,5 point)1. f ’(x) = 20 ( 16 − x 2 )( x2. Arbre complété (1 point)+ 16 ) 2. (1 point)2. a) Sur , on a 20 > 0 et ( x² + 16 )² > 0 , on en déduitque f ’(x) a le même signe que N(x) = 16 – x² .(0,5 point)b) Tableau de variations (1 point)x - 4 4f ’(x) – 0 + 0 –2,53. H ⋂ C : « le salarié choisi est un homme et travaille à f(x)temps complet » . P ( H ⋂ C ) = 0,54 . (1 point)– 2,54. P(C) = 0,82 . (1 point)2. f(8) = 2 , signifie que cette salle de sport compte 200inscrits fin Août 2012. (0,5 point)3. A la sortie de l’algorithme : X = 4 et Y = 2,5 .(1 point)Livre du professeur – Mathématiques Term STMG © Hachette Livre 2013 8


DS Chapitre 5Exercice 11. c) 2. a) 3. b) 4. c) 5. a)Exercice 2Partie A1. P( A) = 0,44 . Arbre complété (1 point)2. a) P(X = 3) ≈ 0,176 ( à la calculatrice ou calculé àl’aide de l’arbre) . La probabilité que les trois côneschoisis contiennent moins de 100 mL de glace estd’environ 0,176, arrondi à 0,001 près. (1point)b) P(X = 2) ≈ 0,414 . (0,5 point)Partie B1. P( 98 ≤Y ≤ 102 ) ≈ 0,525 . (1 point)Selon cette loi normale, la probabilité qu’un cône deglace choisi au hasard dans la production, contienneentre 98 et 102 mL de glace est égale à environ 0,525, à0,001 près. (0,5 point)2. P( 94,4 ≤Y ≤105,6 ) ≈ 0,954 . (1 point)Exercice 31. P( 0 ≤ X ≤ 13 ) ≈ 0,691 . (1 point)2. a) La probabilité que la durée de l’installation dusystème d’alarme soit comprise entre 10 h et 12 h estenviron 0,341, à 0,001 près. (0,5 point)b) Par symétrie, P( 12 ≤ X ≤ 14 ) ≈ 0,341 . (0,5 point)c) Sous la courbe de densité (0,5 point)x6 7 8 9 10 11 12 13 14 15 16 17 18P( X >14 ) = P( X ≤ 10 ) = P( X ≤ 12 ) – P( 10 ≤ X ≤ 12)≈ 0,5 – 0,341 ≈ 0,159 . (1,5 point)3. [ µ - 2 ; µ + 2 ] = [ 8 ; 16 ] . (0,5 point)P( X [ 8 ; 16 ] ) ≈ 0,954 . (0,5 point)Exercice 4104 + 1361. a) µ = = 120 et = 136−104= 8 . (2 points)24b) P( 104 ≤ X ≤ 136 ) ≈ 0,95 . (1 point)2. P( 112 ≤ X ≤ 128 ) ≈ 0,68 . (2 points)DS Chapitre 6Exercice 11. a) 0,08 × 14 000 = 1 120 € . (1 point)b) On passe d’un terme de la suite ( u n ) au suivant enretranchant 1 120 . La suite ( u n ) est donc arithmétiquede premier terme u 0 = 14 000 et de raison a = – 1 120 .(1 point)c) u n = 14 000 – 1 120 n . (1point)d) u 5 = 14 000 – 1 120 × 5= 8 400 . (0,5 point)2. 12 retraits,14 000 – 1 120 × 12 = 560 . (1,5point)Exercice 21. a) Intérêts acquis la première année : 160 € .v 1 = 8 000 + 160 = 8 160 . (1 point)b) v n + 1 = 1,02 v n . (1 point)c) La suite ( v n ) est géométrique, son premier terme estv 0 = 8 000 et sa raison b = 1,02 . Donc :v n = 8 000 × 1,02 n . (1 point)2. a) v 8 = 8 000 × 1,02 8 ≈ 9 373 € . (1 point)b) Formules à saisir en C4 : i) = C3 * 1,02ou iiii) = $C$3 * ( 1 + $C$1 ) ^ B4 (1 point)Exercice 31. c) 2. c) 3. c) 4. c) 5. a)Exercice 41. a) b) Tableau complété (2 points)année n quartier A quartier B test A ≥ B2012 0 3000 2800 vrai2013 1 3100 2912 vrai2014 2 3200 3028 vrai2015 3 3300 3150 vrai2016 4 3400 3276 vrai2017 5 3500 3407 vrai2018 6 3600 3543 vrai2019 7 3700 3685 vrai2020 N = 8 3800 3832 faux2021 9 3900 39852022 10 4000 41442. a) Avant la ligne 5, B vaut b N . En ligne 5,l’algorithme affecte à la variable B la valeur de b N+ 1 ,calculée à partir de b N . (1 point)b) N = 8 . (1,5 point)Le test « A ≥ B » permet de comparer a n ≥ b n , suivantles valeurs de l’entier n .Or jusqu’en N = 7, le test « A ≥ B » est vrai, doncl’algorithme continue : en ligne 4, N vaut 8 : en ligne 5,B vaut 3 832 et en ligne 6, A vaut 3 800.L’algorithme revient au test : à ce moment N = 8 et Adevient inférieur à B. Donc la boucle TANT QUE sefinit et la valeur de N est alors 8.On peut habituer les élèves à présenter dans un tableaules différentes phases de l’algorithme pas à pas (voirexo 73 p.167, corrigé p.233 dans le manuel).Livre du professeur – Mathématiques Term STMG © Hachette Livre 2013 9


DS Chapitre 7Exercice 11. Tableau des fréquences (1 point):jour N°1 N°2 N°3fréquence 0,635 0,655 0,592. a) =12 400= 0,025 et I = [ 0,55 ; 0,65 ] . (1,5 point)b) P( 0,55 ≤ F ≤ 0,65 ) ≈ 0,954 ( résultat du cours : 0,95)(1 point)3. Avec une fréquence de paiement par carte bleue de0,655 , la journée N°2 est « hors norme » , car cettefréquence est supérieure à 0,65. De plus ellen’appartient pas à l’intervalle de fluctuation de lafréquence. (1,5 point)Exercice 21. Tableau complété (0,5 point)nombre de personnesinterrogéesnombre de personnessatisfaitesfréquence depersonnes satisfaitesSondage N° 1 N° 2 N° 3400 1024 100348 818 730,87 0,799 0,7312. a) I 400 = [ 0,85 – ; 0,85 + 1] = [ 0,8 ; 0,9 ] .400 400(0,5 point)b) f 1 = 0,87 [ 0,8 ; 0,9 ] . (0,5 point)3. En 1 : P + 1 / (racine N) (0,5 point)En 2 « l’hypothèse de 85 % de clients satisfaits ».En 3 : « on ne rejette pas l’hypothèse de 85 % declients satisfaits ». (1 point)4. Intervalle de fluctuation, à 95 % , de la fréquence declients satisfaits pour un échantillon de taille 1 024 :1I 1024 = [ 0,85 – ; 0,85 + 1] ≈ [ 0,818 ; 0,882 ] .1024 1024f 2 = 0,799 [ 0,818 ; 0,882 ] .Intervalle de fluctuation, à 95 % , de la fréquence declients satisfaits pour un échantillon de taille 100 :1I 100 = [ 0,85 – ; 0,85 + 1] = [ 0,75 ; 0,95 ] .100 100f 3 = 0,73 [ 0,75 ; 0,95 ] .Comme sur trois échantillons observés, on a deuxfréquences observées qui n’appartiennent pas àl’intervalle de fluctuation, à 95 %, de la fréquence declients satisfaits correspondant à la taille del’échantillon, on peut mettre en doute la proportionattendue de 85 % de clients satisfaits des produitsproposés par cette PME. (2 points)Exercice 31. f = 0,58 .I = [ 0,58 –(1 point)2. f = 138= 0,552 .250J = [ 0,552 –1; 0,58 + 1] = [ 0,48 ; 0,68 ] .100 1001; 0,552 + 1250 250] ≈ [ 0,488 ; 0,616 ] .(1 point)3. Avant le dépouillement et d’après le sondage, lenombre de voix attendu par la liste A , au niveau deconfiance à 95 %, est :au minimum de 0,48 × 4 550 = 2 184 voixet au maximum de 0,68 × 4 550 = 3 094 voix.Le résultat affirmé par les membres de la liste A estdonc correct. (1 point)4. D’après les intervalles de confiance I et J obtenus àpartir de ces deux sondages, la liste A peut obtenir entre48 % et 68 % des voix et la liste B peut obtenir entre48,8 % et 61,6 % des voix, avec un niveau de confiancede 95 % . Ces deux fourchettes contiennent des valeursinférieures à 50 % .Aucune des listes n’est certaine d’obtenir plus de 50 %des voix, au niveau de confiance de 95 % . (2 points)Exercice 41. a) 2. b) 3. b) 4. a) 5. b)Livre du professeur – Mathématiques Term STMG © Hachette Livre 2013 10


Intentions des auteures et conseilsNous avons voulu commencer par une partie nouvelleet facile du programme de terminale, connue desprofesseurs enseignants en STMG. Il existe denombreux exercices de Bac sur ce thème.En première, les élèves ont vu les séries statistiques àune variable : ce chapitre revient sur ces acquis.L’ajustement affine permet d’appréhender unemodélisation simple et son utilisation pourl’interpolation et l’extrapolation, et de revenir surl’équation de droite.Dans le cadre d’un enseignement en spirale, cechapitre peut être traité en 2 parties :• Cours 1 et 2 en tout début d’année, avant les cours 1du chapitre 4, le cours 1 du chapitre 2, le cours 1 duchapitre 3 et les cours 1 et 2 du chapitre 6 ;• Cours 3 et 4 qui doivent être traités avant le cours 3du chapitre 4, et peut être mis en lien avec le cours 2du chapitre 6 (suites arithmétiques).• Nous avons choisi de placer dans la partie cours ladétermination des paramètres d’une série statistiqueà une variable : la médiane et les quartiles, lediagramme en boîte, et le calcul de la moyenne et del’écart type.L’essentiel est la pratique de la calculatrice pourobtenir ces valeurs et connaître la signification etl’emploi de ces différents paramètres.• La fonction affine, la lecture du coefficient directeurd’une droite et son interprétation en termed’accroissement moyen sont des incontournables. Bienque vu et revu depuis la troisième, c’est loin d’être unacquis. Notre pratique en classe (littéraire,économique ou tertiaire) nous a confirmé que la façonla plus simple de lire un coefficient directeur est de lirecomme on écrit la fraction : entre deux points, enallant du point le plus à gauche vers celui à droite,« différence des ordonnées sur différence desabscisses ». Le signe est ainsi au numérateur.Comme la notion de vecteur directeur n’est plus unobjectif de seconde, il est inutile d’effectuer ledéplacement en partant à l’horizontale. De plus, il estnaturel que sous le trait de fraction (horizontal) onécrive le déplacement horizontal.Inutile aussi de parler de « parallèle à l’axe desordonnées » pour toute droite d’équation x = k : lesrepères employés sont toujours orthogonaux. Autantne pas se priver de parler de « droite verticale » !Pour l’équation réduite, y = a ( x – x A ) + y A est l’écriturela plus naturelle : pas d’équation à résoudre pourtrouver b, avec toutes les erreurs de signe que celapeut comporter. Nous avons cependant noté en p.13 laméthode. Bien faire comprendre le rôle des lettres y etx dans l’équation réduite, coordonnées d’un point Mquelconque de la droite . Et rappeler que dans uneéquation il y a « égal » ; donc ne pas oublier « y = ».• L’analyse d’un nuage de points d’une sériestatistique à deux variables permet d’envisager unemodélisation. Il peut être intéressant de faire visualiserun nuage dans des repères d’échelles différentes.Le calcul du point moyen permet de travailler sur leslistes de la calculatrice.• Si les points sont presque alignés, on peut envisagerun ajustement affine.Une première approche peut être faite à la main : oncouvre les points du nuage par un crayon, puis on traceune droite parallèle au crayon et passant par le pointmoyen.Le graphique présenté p. 16 peut expliquer les mots« par moindres carrés » : indiquer aux élèves que lasomme des carrés des distances représentées en jauneest minimale pour la droite de régression .Nous avons donné le vocabulaire employé dans toutesles disciplines autres que mathématiques (régressionlinéaire) et que l’on retrouve sur les calculatrices :RegLin et LinearReg.On peut aborder la notion de corrélation, sans insister,car se pose le problème de cause à effet.L’essentiel est de savoir interpréter les résultatsobtenus à la calculatrice.Livre du professeur - Mathématiques Term STMG © Hachette Livre 2013 11


Avant de commencerCes QCM reprennent des connaissances de seconde.On trouvera des compléments en Techniques de basepour la lecture d’un résultat à la calculatrice (arrondiset chiffres significatifs) p. 212 et la résolutiond’équation du 1 er degré p. 215.QCM 1 : Réponse b) On revoit le calcul de la moyenneà la calculatrice, ainsi que les notations et les unités.a) Faux : la moyenne de la série est x = 56 000 .b) Vrai : la nouvelle moyenne est x = 57 000.c) Faux : 5 véhicules seulement sur les 12 ont parcouruplus de 57 000 km.QCM 2 : Réponse c) Cela permet de revoir ladétermination de la médiane liée à la parité dunombre de valeurs.a) Faux : le nombre de valeurs est pair et la médianeest le « milieu » entre la 6 e et la 7 e valeur.b) Faux : la médiane ne dépend pas des valeursextrêmes.c) Vrai : on place la 13 e valeur 102 dans la série. Lamédiane devient alors la 7 e valeur.QCM 3 : Réponse c) Cela permet de revoir la lecturegraphique du coefficient directeur d’une droite etd’insister sur le quotient ΔyΔx .a) Faux : le coefficient directeur de 1 est 3 2 .b) Faux : la droite descend, son coefficient directeurest donc négatif.c) Vrai. Insister sur a = 1 , à deviner sur l’équation parlecture de x = 1 × x , et sur la droite qui monte de 1carreauQCM 4 : Réponse c) Cela permet de déterminerl’équation réduite d’une droite donnée par deuxpoints. Pour les élèves en difficultés, on peut leur fairefaire un tableau résumant les données.A B écartabscisse 1 6 6 – 1 = 5ordonnée 51 70 70 – 51 = 19a) Faux : c’est l’inverse.b) Faux : le coefficient directeur est a = 19/5 = 3,8donc l’équation réduite est y = 3,8 ( x – 1 ) + 51 .En développant, on obtient y = 3,8 x + 47,2 .c) Vrai : on vérifie en remplaçant x par 9 dansl’équation obtenue : 3,8 × 9 + 47,2 = 81,4 .CoursParamètres d’une série statistique à une variableÉtude d’une situationLes valeurs de la série sont chronologiques et nonrangées par ordre croissant. Demander aux élèves derentrer les valeurs sur leur calculatrice. rabats de couverture suivant les modèlesDès le premier cours, les élèves vont comprendrequ’une calculatrice est obligatoire. Ne pas accepter lesCasio 25 : on ne peut pas calculer sur les lois.On peut faire l’exercice sans calculatrice, par lecturedes valeurs obtenues. Sur Casio :1. a) Moyenne : 13,1 millions par an.b) Écart type : 1, 23 million par an.2. a) 12,5 est une valeur de la série, car il y a unnombre impair de valeurs.b) Non, il faut ranger les valeurs en ordre croissant.3. a) Écart interquartile :IQ = Q3 – Q1 = 14,5 – 12,2 = 2,3 .b) 2 σ = 2 × 1,227 = 2,45, légèrement supérieur à IQ .On verra le lien entre IQ et σ dans le cas d’unerépartition dite « normale » au chapitre 5 p.139.4. 15,6 est une 16 e valeur, supérieure à la valeurmaximale.a) La moyenne va augmenter, car 15,6 > 13,1 .b) La médiane change, c’est le milieu entre la 8 e et la 9 evaleur, valeurs rangées en ordre croissant, soit lecentre de [ 12,5 ; 12,6 ] . Donc Me‘ = 12,55 .c) Le premier quartile ne change pas, car 16 = 4 et la 4e4valeur est 12,2 . Donc Q1‘ = 12,2 .On peut demander aux élèves de construire lediagramme en boîte.Représentation sous Sinequanon.QCM 5 : Réponse c) On aborde l’ajustement affine etl’emploi du tableur et des formules.a) Faux : 2013 = 1968 + 45 et non 46 .b) Faux : les points « descendent », la fonction affineest décroissante.c) Vrai : on explique que = B1 - $B1 peut convenir.Livre du professeur - Mathématiques Term STMG © Hachette Livre 2013 12


Pour la représentation sur calculatrice, voir le livre de1 re STMG p. 203 et 205, mais inutile d’insister.Objectifs des exercices : Savoir déterminer lesparamètres d’une série statistique à une variable à lamain (médiane et quartiles) ou à la calculatrice(moyenne et écart type).La Casio 35+ donne les valeurs exactes des quartiles,valeurs de la série. Les calculatrices TI 82, 83 ou 84donnent des valeurs interpolées pour les quartiles.Fonction affine et droiteÉtude d’une situationNous avons choisi un contexte proche de ceuxrencontrés dans les recherches d’ajustement affine.Le but est de comprendre et d’interpréter le coefficientdirecteur en termes de croissance (ici annuelle). Lemodèle affine va permettre de déboucher sur la suitearithmétique dans le cas discret.1. a) ∆y = 816 – 616 = 200 , soit une augmentation de200 000 visiteurs en 8 ans.b) a = Δy= 200= 25 .Δx 8Variation annuelle de 25 000 visiteurs par an.2. Tableau des résultatsx 2 3 4 5 6 7 8 9 10 11y 616 641 666 691 716 741 766 791 816 841Placer le premier point ( 2 ; 616 ) sur un graphique.Remplir le tableau en ajoutant 25 par annéesupplémentaire, pour faire comprendre « l’escalier »de la représentation graphique et parler de suitearithmétique. Montrer que l’on arrive ainsi au 2 e point( 10 ; 816 ). Parler d’interpolation du nombre devisiteurs entre les deux dates 2002 et 2010 etd’extrapolation pour 2011.Objectifs des exercices : Lire les coefficients directeursde droite, construire des droites et déterminer deséquations réduites. Nous n’avons pas voulu mettretrop d’exercices sur ce thème, juste ce qui estnécessaire pour remettre en mémoire. Il nous sembleinutile d’insister pour les élèves qui n’ont pas encoreacquis ces notions. Mieux vaut revenir sur laconstruction à partir de l’équation réduite obtenuedans le cadre de l’ajustement affine, plus proche de cequi sera demandé au Bac. La lecture du coefficientdirecteur sera revue dans le cas du nombre dérivé.Nuage de points d’une série à deux variablesÉtude d’une situationLa série est une « vraie » série à deux variables, issuesde deux séries chronologiques.Ce cas est très fréquent lors d’une recherche decorrélation.Le graphique indique un lien entre les deux séries, cequi peut tout à fait indiquer une dépendance. Le nuagede points va le confirmer, de par sa forme dite« oblongue » par les économistes. On peut mêmepenser qu’il y a proportionnalité.1. a) Le point A est associé à l’année 2003 : ilcorrespondant aux valeurs les plus basses sur le 1 ergraphique.b) Le point B correspond à l’année 2000, qui a vu laplus forte fréquentation du Québec et le point C àl’année 2008 où la fréquentation du Canada est la plusforte.2. Moyenne x = 357,2 et y = 284,5 .On obtient le point G du graphique.Faire rentrer les valeurs des deux séries à lacalculatrice. On peut y visualiser le nuage de points.Sur TI.fr :1.Graph1… Onvalider l’écran ci-contrepuis9:ZoomStatSur Casio : STATGRPH (F1) SET (F6) Scatvalider les listes 1 et 2puis GPH1 (F1)Objectifs des exercices : Représenter des nuages depoints, entrer la série statistique double surcalculatrice et calculer le point moyen. L’exercice 25permet d’introduire une analyse du nuage de points envu d’un ajustement.Ajustement affineÉtude d’une situationOn reste dans le domaine du tourisme. Un tableurpermet facilement d’obtenir la droite de régression etson équation réduite. Nous avons choisi de parler derégression et de tendance, vocabulaire largementemployé dans les disciplines tertiaires.1. Le nombre de chambres d’hôtes n’est pas toujourscroissant : en x = 6 (1996) et x = 19 (2009) il y a unebaisse par rapport à l’année précédente.Mais globalement, la tendance est à la hausse.2. a) En 2003, c’est-à-dire x = 13 , on obtient :1,54 × 13 + 9 = 29,02Livre du professeur - Mathématiques Term STMG © Hachette Livre 2013 13


soit 29 000 chambres d’hôtes en France.Le point est sur la droite et correspond à un point dunuageb) Interprétation du coefficient directeur de :a = 1,54 , soit 1 540 chambres d’hôtes supplémentairespar an.3. a) En moyenne il ya 24 400 chambres d’hôtes par ande 1990 à 2010.b) On remplace x par 10 dans l’équation de pourretrouver l’ordonnée 24,4 de G :1,54 × 10 + 9 = 15,4 + 9 = 24,4.Donc le point G appartient à (propriété que l’onadmet).4. Par lecture graphique : environ 47 500 chambresd’hôtes.Par calcul, en x = 25 : 1,54 × 25 + 9 = 47,5 soit uneestimation de 47 500 chambres d’hôtes en 2015.On peut faire remarquer que la droite « desextrêmes » joignant les points en 1990 et 2010 nedonne pas un bon ajustement : pratiquement tous lespoints sont situés au dessus.Inutile d’insister sur les termes « par moindrescarrés ». On peut cependant visualiser les distances« en verticale » des points à la droite , et expliquerque la variable étant x , on regarde le comportementde la grandeur étudiée y .On peut reprendre le nuage de points du cours 3 etconsidérer comme ajustement la droite passant parl’origine du repère ( 0 ; 0 ) et le point G afin dedéterminer une fonction linéaire : d’après cetajustement, 80 % des touristes français au Canadavisitent le Québec.Atelier TICEÉquation de droiteBut : Introduire l’algorithmique à travers un algorithmeutile aux élèves « allergiques » aux droites depuis 4ans. Cet algorithme est expliqué sur AlgoBox p. 236.Il n’utilise pas de test ou de boucle, juste la saisie devaleurs, le calcul et la sortie de résultats.On peut demander à certains élèves de justifier laformule donnant B. Pour cela, demander d’écrire lecalcul donnant b à la question A. b) en laissantapparaître les nombres 2 et 5 , coordonnées de M.A. À la maina) Coefficient directeur de (MN) : a = Δy= − 6= – 1,5 .Δx 4b) On cherche b dans la forme y = a x + b,où x = 2 , y = 5 et a = -1,5 . D’où 5 = – 1,5 × 2 + b .Ainsi b = 5 – (-1,5) × 2 = 5 + 3 = 8 .c) Avec le point N( 6 ; – 1 ) on obtient – 1 = – 1,5 × 6 + bD’où b = – 1 + 9 = 8 .Donc l’équation de la droite est y = -1,5 x + 8.Nous parlons de forme y = a x + b plutôt qued’équation, car dans ces questions l’inconnue est b .L’écriture du calcul de b donne la formule à écrire dansl’algorithme D – A × C = B .Prendre l’autre point N montre que quel que soit lepoint choisi, on obtient la même équation (ce qui estloin d’être évident pour les élèves).B. Avec un algorithme1.a) A = F−Det B = D – A × C .E−CRepérer à quoi correspondent les variables C D E et F .b) A = -1,5 et B = 8 , obtenus à la partie A.2.a) Pour C = 1 , D = 3 , E = 3 et F = 8 , on obtient :A = 8−3= 2,5 et B = 3 – 2,5 × 1 = 0,5 .3−1b) Comme C = E , la calculatrice indique une erreur:sur TI : DIV PAR 0 sur Casio : Ma ERRORet renvoie à la division donnant A. On ne peut diviserpar 3 – 3 = 0 ! On confirme que les deux points sont surune droite verticale, sans coefficient directeur.L’équation de la droite est x = 3 .Ne pas hésiter à faire placer les points correspondantsdans un repère.3. On applique l’algorithme et on obtient :y = 0,18 x + 0,33 .Objectifs des exercices : La capacité attendue est desavoir utiliser un tel ajustement pour des estimations,des comparaisons. On aborde aussi la recherche demodèle, introduisant ainsi d’autres fonctions possiblesajustant un nuage de points, ce que nous proposonsdans la partie Pour réussir au Bac de ce chapitre.Livre du professeur - Mathématiques Term STMG © Hachette Livre 2013 14


4. Formules sur tableur :1. Nuage de points obtenus et ajustement affine :Âge moyen de la mariéeBut : Déterminer un ajustement affine. Une partie dela classe peut faire la partie A pendant que l’autreréalise la partie B sur tableur.A priori, il ne faut pas plus de temps pour faire l’un oul’autre et 15 min suffisent pour ce genre d’exercice. Onpeut faire 3 groupes dans la classe avant la séance, 1ou 2 groupes sur ordinateur, suivant le nombre demachines, mais travaillant individuellement, le 3 egroupe réalisant à la calculatrice l’atelier 3, outerminant l’atelier 1 sur calculatrice.A. Avec la calculatrice1. x = 5 et y = 29 .2. Droite des extrêmes : a = ΔyΔx= 30−2810−0= 0,2 d’oùl’équation y = 0,2 ( x – 0 ) + 28 y = 0,2 x + 28 .On peut aussi VERIFIER à l’aide de l’algorithme del’atelier 1B. Avec un tableur1. Réponse a) = B1 – 2000 ou c) = B1 - $B1 .2. a) b) On doit obtenir la figure.3. a) C’est une régression linéaire ou ajustement affine.b) y = 0,2109 x + 7,945Sur Excel, par défaut, les coefficients sont donnés avec4 chiffres significatifs. p. 213D’où l’équation y = 0,21 x + 28 .c) En 2015, on a x = 15 et y = 0,21 × 15 + 28 = 31,15 .L’âge des femmes au premier mariage, en moyenne,est estimé légèrement supérieur à 31 ans.En post Bac, on note les valeurs estimées avec un« chapeau » x = 15 et y = 31,15 .4. Pour la période de 2005 à 2010, on obtientl’équation y = 0,17x + 28 .Avec cet ajustement, on obtient, en 2015 :y = 0,17 × 15 + 28 = 30,55 ,ce qui est inférieur à 31 ans.Pour sélectionner la bonne plage de cellules, on cliquesur un point du nuage.Âge moyen du mariéBut : Reprendre l’étude faite précédemment enautonomie.y = 0,18 x + 30 avec deux chiffres significatifs.Si x = 15, alors y = 0,18 × 15 + 30 = 32,7 , soit un âgemoyen du marié de 32,7 ans en 2015.2. On obtient, sur 2005-2010 : y = 0,15 x + 30 .Si x = 15, alors y = 0,15 × 15 + 30 = 32,25 .On remarque que le nuage de points « s’infléchit » : lesderniers sont en dessous de la droite de régression.L’année 2005 marque un point d’inflexion.QCM page 21 pour répondre à un QCM p. 1951. b. Il faut vérifier à la calculatrice.On entre les valeurs en liste 1 de la calculatrice, on obtient : Rabats de couverture2. c. On commence par calculer le coefficient directeur de ladroite (AB) : a = ∆y= 0,58 . Ceci est suffisant pour=17,8−14,9∆x 7−2conclure que l’équation est y = 0,58 x + 13,74 .On peut vérifier que A( 2 ; 14,9 ) D : y = 0,58 ( x – 2 ) + 14,9et on distribue : y = 0,58 x + 13,74 .3. c. On peut résumer les données dans un tableau :Livre du professeur - Mathématiques Term STMG © Hachette Livre 2013 1512quantité en t : x 5 7prix en €/kg : f(x) 0,8 0,6On cherche f(x) = a x + b :diff érence de prixa =diff érence de quantit é=0,6− 0,87 − 5 = – 0,1 .Pour x =5, on a f(5) = 0,8 ; d’où :f(x) = – 0,1 ( x – 5 ) + 0,8 = – 0,1 x + 1,3 .4. b. Cela ne peut pas être c. car les x i vont de 0 à 25 .On calcule la moyenne des y i : environ 295,83.


5. c. On doit utiliser la calculatrice : les x i en liste 1 et les y i enliste 2.D’où l’équation y = 16,05 x + 95,19 .6. a. 2003 est l’année 1980 + 23 .Donc x = 23 , dans l’équation de la droite de régression de yen x : y = 16,05 × 23 + 95,19 = 464,34 ≈ 464 , soit 464 € parpersonne en 2003.Corrigés des exercices du chapitre 1ex1 Moyenne : x ≈ 2,58 .On place la 16 e valeur, 3,14 , dans la liste ordonnée desfréquentations. La médiane est le milieu de la 8 e et de la 9 evaleur. Me = 2,655 .ex2 x ≈ 3,04 et σ ≈ 0,08 .Par rapport à la période 1996-2010 , la fréquentationmoyenne a augmenté de 2,54 à 3,04 et l’écart type adiminué de 0,46 à 0,08 .ex3 Équations : d 1 : y = − 4 3 x + 4 d 2 : y = − 5 8 x + 5d 3 : y = − 613 x + 6 .ex4 a) La droite (CD) a pour coefficient directeur :a = ∆y= 13−3= 2 .∆x 6−1D’où l’équation réduite de (CD) :y = 2 ( x – 1 ) + 3 y = 2 x + 1 .b) a = ∆y= 50−80= – 1,5∆x 35−15D’où y = – 1,5( x – 15 ) + 80 y = – 1,5x + 102,5 .ex5 a) Tableau des nouvelles sériesex6 a) 2012 est l’année de rang 12.y = 0,081 × 12 + 0,991 = 1,963 .Le nombre de 2 millions de visiteurs espérés est proche decelui obtenu par l’ajustement affine.b) On résout l’équation :0,081 x + 0,991 = 2,29 x = 16,03 .En 2016, on peut estimer que le niveau de fréquentation del’année 2000 sera retrouvé.c) Graphiquement, l’ordonnée du point d’abscisse 16 , sur ladroite rouge, est d’environ 2,3 .ex7 QCM1. c. 2. c 3. c. 4. c. 5. b 6. a.À la calculatrice, on obtient Me = 22,5 .ex8 a) Nombre de prix relevés : n = 36 .Prix moyen d’une paire : x ≈ 54 € .Prix médian : Med = 49 € .b) Écart type : σ ≈ 27,95 € .c) Étendue : e = 105 – 12 = 93 €Écart interquartile : IQ = Q3 – Q1 = 55,5 € .ex9 x = 36,625 ; σ ≈ 13,81 .rang del’année i2005 2006 2007 2008 2009 2010 20111 2 3 4 5 6 7nombre totalde nuitées x i193 192 199 198 188 192 198dont étrangersy i70,5 68,8 72,4 71,1 63,2 64,9 66,5z i en % 36,6 35,9 36,4 36,0 33,6 33,8 33,5ex10 x ≈ 45,36 ; σ ≈ 16,71 .ex 11 L’utilisation de la calculatrice pour une série aveceffectifs est expliquée dans les rabats de couverture.b) Nuage de points : voir ci-après.c) La part des étrangers dans la fréquentation de l’hôtellerieen France a tendance à diminuer depuis 2005, la plus fortediminution étant en 2009, après la crise de 2008.La moyenne est 7,3 et l’écart type 1,526 .ex12 1. Min = 350 ; Q1 = 450 ; Me = 600 ; Q3 = 700 ;Max = 900 .Livre du professeur - Mathématiques Term STMG © Hachette Livre 2013 16


2. 25 % des ces familles ont un budget mensuel alimentationcompris entre 600 et 700 €.ex13 1. Dans cette ville, le montant des loyers desappartements considérés est compris entre 350 € et 1050 €.Au moins 25 % des loyers sont inférieurs ou égaux à 530 € ;au moins 50 % sont inférieurs ou égaux à 650 € et au moins75 % sont inférieurs ou égaux à 780 €.2. Étendue e = 700 € et IQ = Q3 – Q1 = 250 € .3. Diagramme en boîte à l’aide de Sinequanon : 2 « descend » et passe par B( 0 ; 4 ) : elle représente lafonction décroissante k. 3 est verticale et ne peut donc pas représenter unefonction. 4 « monte » et passe par C( 2 ; 0 ) : elle représente lafonction croissante h.La fonction f n’est pas représentée.b) Si a > 0, la fonction affine est croissante (cas de h ).Si a < 0 , la fonction affine est décroissante (cas de f et de k ).ex181. a) Représentation des droitesx300 400 500 600 700 800 900 1000On entre les paramètres en série isolée.b) Coefficient directeur de la droite (AB) : a = ∆y∆x = 2 3 .c) D’où l’équation de la droite (AB) :y = 2 3 ( x – 3 ) + 4 y = 2 3 x + 2 .2. Équation de : y = −1( x– 3 ) + 4 y = − 1 x + 5 .3 3ex19a) On trace ∆ à partir du point ( 0 ; 5 ) et si x = 10, alors y = 2 .ex14 1. a) On lit directement la valeur des paramètres decette série statistique car les valeurs sont rangées en ordrecroissant.b) Me = 3,25 ; Q1 = 3,04 et Q3 = 3,45 .c) Dans au moins 10 villes parmi les 20 considérées dansl’étude, le prix du m 3 d’eau est compris entre 3,04 et 3,45 €.ex15segment [AB] [BC] [CD] [DE]coefficient− 3 − 2 13directeur 234ex16droite 1 2 3 4coefficientdirecteurex17− 4 3− 1 3a) La droite 1 est horizontale : elle représente la fonctionconstante g.045b) Si x = 7 , alors y = - 0,3 × 7 + 5 = 2,9 . D’où E( 7 ; 2,9 ) .L’abscisse de F est solution de l’équation :– 0,3 x + 5 = 2,45 x = 8,5 .D’où F( 8,5 ; 2,45 ) .ex20 Coefficient directeur :a = ∆y 11 − f(3)=f = 10−12,4= – 0,3 .∆x 11 − 3 11−3D’où f(x) = – 0,3 ( x – 3) + 12,4 f(x) = – 0,3 x + 13,3 .ex21 a) Augmentation moyenne par an du PIB de la Chine7,30 − 1,93entre 2011 et 2004 :≈ 0,77 ,2011 − 2004soit environ 0,77 milliard de dollars par an en moyenne.Livre du professeur - Mathématiques Term STMG © Hachette Livre 2013 17


) Cette augmentation annuelle est le coefficient directeurde la droite passant par les deux points A et B placés.c) Si on place le point C ( 2009 ; 4,99 ) , il ne semble pasaligné avec A et B .Coefficient directeur de (AC) :4,99 − 1,932009 − 2004= 0,612 0,77 .Comme les droites (AB) et (AC) n’ont pas le mêmecoefficient directeur, les points A, B et C ne sont pas alignés.ex23 1. a) Faux : le nuage de points M i ( x i ; y i ) est le nuage 4.b) Faux : Le nuage de points N i ( x i ; t i ) est le nuage 2.c) Faux. d) Faux.2. a) Nuage 3. b) Nuage 1.ex24 a) Nuage de points M i ( x i ; y i ) :ex22 a) Points et droite (AB) :Bien respecter les consignes pour les unités sur les axes. p.30 et 194b) Point moyen G( 8,13 ; 7,67 ) .c) On a : 767 × 8,13 = 6235,71 .Donc le chiffre d’affaires engendré par la vente de 767 kg defraises au prix moyen de 8,13 € par kg est 6 237,71€.ex25 1. Nuage de points M i ( x i ; y i ) .b) En prenant x = 0 en 2000 : A(0 ; 9,9) et B(8 ; 14,2) .Coefficient directeur de la droite (AB) :a =14,2 − 9,98= 0,5375 .Ce coefficient directeur est l’accroissement annuel du PIBdes États-Unis entre les années 2000 et 2008 :accroissement de 537,5 millions de dollars par an.c) y = 0,5375 x + 9,9d) Si x = 6 , alors y = 13,125 . En 2006, selon ce modèle, lePIB des États-Unis est estimé à 13,125 milliards de dollars.2. a) b) Point moyen G( 4,5 ; 11,8225 ).3. Les points M 2 et M 3 sont en dessous du segment [M 1 M 4 ]et les points M 5 , M 6 et M 7 sont au-dessus de [M 4 M 8 ] .En M 4 , le nuage de points change d’inflexion (terme qui aété vu par les élèves en seconde dans le programme degestion).Livre du professeur - Mathématiques Term STMG © Hachette Livre 2013 18


ex26 1. Dans ce repère, le nuage de points donnel’impression que les valeurs sont très dispersées. Lavariabilité semble très importante (impression de chaos).Ce n’est plus le cas dans le deuxième repère, où les pointsdonnent l’impression d’une stagnation.b) G ( 20,8 ; 9,7 ) .c) Δ : y = – 0,125 x + 12 .d) G Δ .ex30a) : y = 2,1 x + 1,8b) Nuage de points2. Dans un repère avec une échelle moins étendue :3. « Le nombre d’accidents est relativement constant. »ex27 1. Dans les figures a. et b. , les points du nuage sontpresque alignés, un ajustement affine peut convenir.2. On peut choisir la droite passant par les points extrêmesdu nuage comme droite d’ajustement. Le coefficientdirecteur est 2.3. Le nuage de points de la figure c. peut être ajusté par uneparabole.ex 31 a) G( 6,75 ; 6,425 ) .b) Sortie calculatriceD’où l’équation : y = 0,670 x + 1,91ex32 a) G ( 6,13 ; 14,62 ) b) y = 3,05 x – 4,09 .ex33 a) G ( 44,23 ; 18,53 ) b) y = - 0,323 x + 32,8 .ex34 1.Nuage de pointsex28 a) La série A est représentée en fig. 1 et la série B enfig. 2 .b) On peut envisager un ajustement affine pour chacun deces nuages, car dans les deux cas, les points du nuage sontpresque alignés.Pour la série en fig. 2. l’ajustement est bien adapté. p. 16c) N ( 5,6 ; 5,8 ) est le point moyen du nuage en fig. 1 .(1 + 3 + 4 + 5 + 7 + 9 + 10)/7 ≈ 5,57 ≈ 5,6Respecter les arrondis : Techniques de base p. 213ex29 a) Tableau des valeursx 0 5 10 15 20y 12 12 11 10 1026 27 30 35 4010 9 8 8 72. d’équation y = -2 x + 16 .ex35 y = - 1,99 x + 35,1 . Entre les années 2007 et 2010, enmoyenne, le nombre de salariés dans les agences de voyagesa diminué de 1,99 millier par an.ex36 1. À l’aide de la calculatrice, on obtient :y = 0,5 x + 2,3 .Livre du professeur - Mathématiques Term STMG © Hachette Livre 2013 19


2. En 2015 = 2002 + 13 ; donc x = 13 .0,5 × 13 + 2,3 = 8,8 .Par extrapolation de l’évolution, on peut prévoir 8,8 millionsde visiteurs au Château de Versailles en 2015.ex37 1. a) En 2008 : 4,8 % .b) En 2015, on prévoit une part de 5,9 % .2. Ajustement affine y = 0,15 x + 3,65 .ex40 1. a) Nuage de points : voir ci-après.b) Le segment de coefficient directeur le plus grand sembleêtre celui reliant les deux derniers points du nuage.Autrement dit, la plus forte hausse annuelle du nombre dechômeurs en France depuis 1966 s’est produite entre 2008et 2012, du fait de la crise de 2008.ex38 1.année 1980 2000 2010nombre de stations en milliers 41,5 16,2 12,116,2 − 41,520= – 1,265 et12,1 − 16,210= – 0,41 .De 1980 à 2000, la baisse moyenne est de – 1 265 stationspar an et de 2000 à 2010, la baisse moyenne est de – 410stations par an.2. La droite passe par les points (2000 ; 15) et (2010 ; 12).12 − 15Son coefficient directeur est a = = – 0,3 .10D’où son équation : y = − 0,3 ( x – 2000 ) + 15 .3. Tableau de valeurs lues sur le graphiquex i 1980 1985 1990 1995 2000y i 41,5 32 24,5 18,4 16,2y = – 1,28 x + 2581,68 .Le graphique n’est pas homogène : de 1980 à 2000, lesvaleurs sont données tous les 5 ans et de 2000 à 2010 tousles ans. On ne peut donc pas interpoler par la droite pourles années avant 2000.2. a) Tableau de valeursy = 0,118 x – 0,4x i 9 12 18 28 32y i 0,5 1 2 3 3,2b) 2012 est l’année de rang 46 : 0,118 × 46 – 0,4 = 5,028 .Si la tendance des années 1974 à 1997 s’était maintenue, lenombre de chômeurs en 2012 est d’environ 5 millions.ex41 1. a) Effectifs cumulés :en D3, on saisit la formule =C3+D2 .ex39 1. a) A( 1 ; 38 ) et B( 5 ; 19 ) :équation de la droite (AB) : y = - 4,75 ( x – 1 ) + 38 y = – 4,75 x + 42,75 .b) 2015 est l’année de rang 8 : – 4,75 × 8 + 42,75 = 4,75 .Par extrapolation utilisant la droite (AB) , l’entreprise Nokiapeut prévoir 4,75 % de part de marché en 2015.2. a) Tableau de valeursx i 1 2 3 4 5y i 38 36 28,9 23,8 19Ajustement affine par moindres carrés :droite d’équation y = – 5,02 x + 44,4 .b) Par comparaison des coefficients directeurs, on constateque la droite descend plus rapidement que la droite (AB) .On interprète ce résultat en disant que la baisse annuelle dela part de marché de Nokia est plus importante si l’on utilisela droite plutôt que la droite (AB) pour déterminer desparts de marché de Nokia.c) – 5,02 × 8+ 44,4 = 4,24 .Par extrapolation avec la méthode des moindres carrés, lapart de marché prévue en 2015 est de 4,24 % .Part plus faible que celle obtenue à la question 1. b).b) • Comme N = 28 et 28 = 14, la médiane est le milieu entre2la 14 e et la 15 e valeur de la série. Or la 14 e valeur vaut 80,jusqu’à la 19 e ; donc Me = 80 .• N 4 = 7 : le quartile 1 est la 7e valeur.Or 62 est la valeur de la 6 e à la 8 e ; donc Q1 = 62 .• 3N 4= 21 : le quartile 3 est la 21e . Or 95 est la valeur de la21 e ; donc Q3 = 95 .c) Concrètement, 4 boîtes contiennent 95 € de dons et 24boîtes contiennent une somme inférieure ou égale à 95 € .2. a) x = 82,5 € ; σ ≈ 26,4 € .b)La médiane 80 € est inférieure à la moyenne 82,5 € .c) [ Q1 ; Q3 ] = [ 62 ; 95 ] .[ x – σ ; x + σ ] = [ 56,1 ; 108,9 ] .L’intervalle [ x – σ ; x + σ ] contient l’intervalle interquartile[ Q1 ; Q3 ] .ex42 1. a) Pour certains des 27 pays de l’UE, le PIB a diminuéentre le 1 er trimestre 2011 et le 1 er trimestre 2012 (presquela moitié vu la place de la médiane à 0,3).Livre du professeur - Mathématiques Term STMG © Hachette Livre 2013 20


) Q1 = – 1 : au moins 25 % des pays de l’UE ont vu leur PIBdiminuer de 1 % ou plus de 1 %.Q3 = 1,5 : au moins 75 % des pays ont eu une croissance duPIB inférieure ou égale à 1,5 % .2. Le rang de la France est celui de la médiane de la série.3. Pour au moins 25 % des pays, la croissance du PIB a étécomprise entre 0,3 % et 1,5 % sur la période indiquée.ex46 1. a) Tableau des valeurs de la série doubleex43 1. a) Me = 17,5et les quartiles à Q1 = 12 et Q3 = 28,25 .La moyenne x = 21 et l’écart type σ ≈ 11,73 .b) En K5 , on saisit = MOYENNE(A2 : J4)2. a) Étendue : 48 – 7 = 41 .b) CM = 48 ≈ 6,857 et ( 6,857 – 1 ) × 100 = 585,77Du minimum au maximum, le nombre de commandesaugmente de 586 % à 1 % près.ex44 a) 2012 est l’année de rang 12.b) Tableau de la série doubleannée 2000 2004 2008 2012rang x i 0 4 8 12coût en euros y i 24,4 28,7 32,2 35,32. a) Équation de : y = 0,905 x + 24,72 .b) Si x = 10 , alors y = 33,77 .En 2010, coût horaire de la main d’œuvre estimé à 33,77 € .c) Si x = 15 , alors y = 38,295 .En 2015, coût horaire de la main d’œuvre prévu à 38,30 € .ex45 1. En B2, on saisit = B1 – 2000 ou = B1 – $B$12. a) Équation de la droite derégression Δ de y en x obtenue à lacalculatrice : y = 0,465 x + 26,06 .Et sur tableur :b) La droite associée au coût horaire de la main d’œuvreen France a pour équation y = 0,905 x + 24,72 .La droite Δ associée au coût horaire en Allemagne a pouréquation y = 0,465 x +26,06 .abscisse 0 15ordonnée pour 24,72 38,295ordonnée pour Δ 26,06 33,0352. Avec seulement 4 points donnés, et des points presquealignés, on peut penser que l’ajustement affine est judicieuxpour le coût horaire en Allemagne. Mais on remarque unelégère tendance à augmenter plus que le coefficient. D’où larecherche d’une parabole.3. a) On obtient cette équation à l’aide d’Excel. p. 96g(x)= 0,019 x² + 0,24 x +26,36 pour x [ 0 ; 20 ] .Et g(12) = 31,976 ≈ 32 . Ce modèle donne un résultat àenviron 2 centimes d’écart du résultat donné dans l’exercice45 pour le coût de la main d’œuvre en Allemagne en 2012.b) g(15) = 34,235 . Résultat supérieur à celui calculé parextrapolation à l’exercice 45 en 2. b.ex47 1. a) Tableau des valeurs de la sériex i 1 2 3 4 5y i 95 215 265 283 340b) 2015 est l’année de rang x = 15 et :0,465 × 15 + 26,06 = 33,035.Par extrapolation, on peut prévoir, en 2015, un coût horairede la main d’œuvre en Allemagne de 33 € . arrondis p. 2136 7 8 9 10395 428 430 502 617b) Point moyen : G ( 5,5 ; 357 ) .2. a) A ( 1 ; 95 ) et B ( 10 ; 617 ) .b) Coefficient directeur de la droite (AB) : a =617 − 95D’où l’équation de la droite (AB) :y = 58 ( x – 1 ) + 95 y = 58 x + 37 .c) 58 × 5,5 + 37 = 356 ≠ 357 .On en déduit que la droite (AB) ne passe pas par G .3. Équation de la droite de régression de y en x :y = 48,6 x + 89,6 .Livre du professeur - Mathématiques Term STMG © Hachette Livre 2013 219= 58 .


4. Le coefficient directeur de la droite (AB) est supérieur àcelui de la droite , c’est donc le modèle de la droite desextrêmes qui prévoit l’accroissement le plus rapide dunombre de visiteurs.ex48 1. a) Nuage de points4. Estimationsa) Au prix fixé à 400 € pour ce logiciel, le nombred’acheteurs potentiels estimé à partir de la droited’ajustement par la méthode de Mayer est donné par :y = – 0,25 × 400 + 227,5 = 127,5soit environ 127 acheteurs.En utilisant la droite de régression par la méthode desmoindres carrés, on obtient :y = – 0,3 × 400 + 248,7 = 128,7soit environ 128 acheteurs.b) D’après la représentation graphique, on ne peut pasenvisager un prix de vente de ce logiciel à 850 €. Le nombred’acheteurs potentiels serait négatif.ex 50 Texte en françaisLe graphique ci-dessous présente la population duRoyaume-Uni, en milliers d’habitants, depuis 1950 tous les10 ans.1. Donner le tableau des valeurs où x i est le rang de l’annéedepuis 1950 et y i la population en millions d’habitants.2. Déterminer l’équation réduite de la droite d’ajustementaffine des points M i ( x i ; y i ) .3. Extrapoler la population du Royaume-Uni en 2030 d’aprèscet ajustement affine.ex51 QCM Utilisation de la calculatrice obligatoire1. b. 2. a. 3. a. 4.c. 5. c.ex52 1. Ajustements affinesa) Soit M 1 (1 ; 100) et M 7 (7 ; 113,5) .Le coefficient directeur de la droite Δ , droite des extrêmes(M 1 M 7 ) , est a = 113,5−100= 2,25 .7−1D’où l’équation réduite de la droite Δ :y = 2,25 ( x – 1 ) + 100 y = 2,25 x + 97,75 .b) À la calculatrice, on obtient une équation de la droited’ajustement affine de y en x : y = 2,16 x + 96,81 .b) Point moyen G( 430 ; 120 ) .2. Droite de Mayera) G 1 ( 330 ; 145 ) ; G 2 ( 530 ; 95 ) .b) Équation de la droite (G 1 G 2 ) : y = – 0,25 ( x -330 ) + 145 y = – 0,25 x + 227,5 .c) – 0,25 × 430 + 227,5 = 120 = y GCe calcul prouve que (G 1 G 2 ) passe par G (430 ; 120).3. Droite de régression de y en xÉquation de : y = – 0,3 x + 248,7 .Conseil : Bien respecter les arrondis demandés.c) Comme ajustement affine du nuage de points, on retientla droite d’équation y = 2,2 x + 96,8 .Coefficients arrondis au dixième.Pour tracer la droite D sur le graphique, on détermine lescoordonnées de deux points assez éloignés.Si x = 2 , alors y = 2,2 × 2 + 96,8 = 101,2 ;Si x = 8 , alors y = 2,2 × 8 + 96,8 = 114,4 .On place A( 2 ; 101,2 ) et B( 8 ; 114,4 ) , puis on trace ladroite (AB) .Livre du professeur - Mathématiques Term STMG © Hachette Livre 2013 22


La 20 e année après le début de l’étude, soit en 2014, onestime que la consommation d’énergie en France dans lesecteur des transports dépassera 55 millions de tep.C a) On ajoute aux points du nuage précédents les points decoordonnées ( 14 ; 55,5 ) , ( 15 ; 49,7 ) et ( 16 ; 50,1 ) .b) À partir de l’année 2007, le modèle admis en B.3. n’estpas un bon ajustement. En effet les trois nouveaux points dunuage sont très éloignés de cette droite.Après le Grenelle de l’environnement et la crise de 2008, laconsommation d’énergie a diminué et la droite n’indiqueplus la bonne tendance pour les valeurs de la consommationd’énergie en France dans le secteur des transports.d) 2015 est l’année de rang x = 9 .On calcule 2,2 × 9 + 96,8 = 116,6 .En supposant que ce modèle affine reste valable, l’indice desprix prévu en France en 2015 est 116,6.On rappelle qu’un indice n’a pas d’unité.2. Ajustement à l’aide d’un logiciela) Si x = 8 , alors y = 0,26 × 8² + 0,1× 8 + 100 = 117,44 .b) 0,26 × 9² + 0,1× 9 + 100 = 121,96 .Selon le modèle défini par la courbe , l’indice prévu en2015 est 121,96. La différence est importante.L’ajustement affine ne tient pas compte de la formeincurvée du nuage de points. On peut penser quel’ajustement par la fonction est meilleur pour 2015.ex54 Partie A1. Nuage de points M i ( x i ; y i )ex53 A Nuage de points M i ( x i ; y i ) :B 1.a) b) Point moyen G ( 8,8 ; 49,3 ) .2. Équation de la droite , droite d’ajustement de y en x ,obtenue à la calculatrice y = 0,5509x + 44,3838 .3. Dans la suite de l’exercice, on utilise pour la droite l’équation y = 0,55 x + 44,4 . Cette droite passe par le pointmoyen G et par le point de coordonnées ( 0 ; 44,4 ) .4. a) 2010 est l’année de rang 16. Or :0,55 × 16 + 44,4 = 53,2 .Donc, selon cet ajustement affine, en 2010, laconsommation d’énergie en France dans les transports estestimée à 53,2 millions de tonnes équivalent pétrole.b) On résout l’inéquation :0,55 x + 44,4 > 55 0,55 x > 10,6 x > 19,27 .2. a) Équation de la droite , droited’ajustement de y en x , obtenue àla calculatrice : y = 0,39 x + 58,95 .b) Cette droite passe par ( 0 ; 58,95 )et ( 12 ; 63,63 ) .3. 2015 est l’année de rang 15 et 0,39 × 15 + 58,95 = 64,8.Selon cet ajustement affine, en 2015, la population françaiseest estimée à 64,8 millions d’habitants.Partie B1. Pour ce nuage de points représentant les effectifs depopulation en Allemagne, on n’envisage pas d’ajustementaffine. En effet, la courbe passant près des points n’est pasune droite mais a une allure de parabole.2. Soit f(x) = - 0,022 x² + 0,165 x + 82,2 pour x [ 0 ; 13 ] .2013 est l’année de rang 13.Comme la courbe représentative de la fonction f réaliseun ajustement fiable de ce nuage de points, une estimationde la population allemande en 2013 est donnée par la valeurf(13) . Or f(13) = 80,627.D’où la population estimée d’environ 80,63 millionsd’habitants en Allemagne en 2013.ex55 1. De l’année 2000 à l’année 2010, la dette est passéede 827,3 à 1591,2 milliards d’euros.1591,2 − 827,3On obtient un accroissement annuel moyen de 76,39milliards d’euros, pour la période 2000-2010.10= 76,39 .Livre du professeur - Mathématiques Term STMG © Hachette Livre 2013 23


2. a) Nuage de points M i ( x i ; y i ) : voir ci-après.b) Les points sont relativement alignés. On peut tracer unedroite proche de tous les points du nuage, donc la forme dunuage permet d’envisager un ajustement affine.c) Le point moyen G a pour coordonnées ( 5 ; 1 144,6 ) .Cela signifie que le montant moyen annuel de la dette sur lapériode 2000-2010 est de 1 144,6 milliards d’euros.d) La droite passant par G et de coefficient directeur 76,4 apour équation :y = 76,4 ( x – 5 ) + 1 144,6 y = 76,4 x + 762,6 .Cette droite, tracée en bleu sur le graphique, passe par lepoint G et le point de coordonnées ( 0, 762,6 ).3. a) Équation de la droite , droite d’ajustement de y en x ,obtenue à la calculatrice : y = 73,5 x + 777,2 .b) 2013 est l’année de rang 13.73,5 × 13 + 777,2 = 1 732,7 .Selon cet ajustement affine, le montant de la dette prévupour la fin de l’année 2013, s’élève à 1 732,7 milliardsd’euros.4. En réalité, la dette de l’État français a augmenté plusrapidement que ne le prévoyait le modèle de la question 3.puisque celle-ci s’élevait déjà à 1 789,4 milliards d’euros dèsla fin du premier trimestre 2012, montant déjà supérieur àcelui attendu en 2013.La tendance des années 2006 à 2009 donne une impressionde parabole : fin 2012 la dette est estimée à 1 833,8 Mds.b) Cette droite passe par le point moyen G et par le pointde coordonnées ( 12 ; 0 ) .c) La droite traverse l’axe des abscisses au point decoordonnées ( 12 ; 0 ) .3. a) En utilisant l’ajustement affine précédent et d’après lerésultat de la question 2. c, le prix unitaire maximum que lasociété peut fixer si elle veut conserver des acheteurs est de12 €.b) On résout l’inéquation :– 16 x + 192 ≥ 0 16 x ≤ 192 x ≤ 12 .C’est-à-dire que pour un prix unitaire strictement inférieur à12 €, la société conserve des acheteurs pour ce produit.À 12 €, il n’y a plus d’acheteurs.4. Soit g(x) = 5 + 300, pour x > 3.x−2a) g(10) = 42,5 . Selon ce modèle, pour 10 € l’article, lasociété peut prévoir 42,5 milliers d’acheteurs.b) On peut représenter graphiquement cette fonction, ouréaliser une table de valeurs, on constate que la courbereprésentative de la fonction g ne traverse pas l’axe desabscisses. Pour x > 3 , g(x) > 0.On ne trouvera donc pas de prix unitaire maximum avec cemodèle. Faire visualiser le tableau de valeurs aux élèves.ex56 1. a) Nuage de points M i ( x i ; y i ) .b) Point moyen G ( 7,5 ; 72,5 ).2. a) Équation de la droite , droite d’ajustement de y en x ,obtenue à la calculatrice : y = -16 x + 192 .Livre du professeur - Mathématiques Term STMG © Hachette Livre 2013 24


Intentions des auteures et conseilsEn première, les élèves ont vu le taux d’évolution,variation absolue, variation relative, les évolutionssuccessives et le taux réciproque.La nouveauté du programme de terminale est l’indicebase 100 et le taux d’évolution moyen.Dans le cadre d’un enseignement en spirale, cechapitre peut être traité en 3 parties :• Cours 1 et 2 dans un 1 e temps en début d’année.• Cours 3 pour un travail sur le tableur et la mise enplace du symbole $ . Avant ou juste après l’ajustementaffine.• Cours 4 juste avant de traiter les suites.• Exploiter les relations entre valeurs de départ y 1 etvaleur d’arrivée y 2 et le taux d’évolution t est une descapacités attendues en première.Comme ces relations sont à la base de tous calculsd’évolution, nous avons choisi de les reprendre dans lepremier paragraphe du cours.La seule formule que l’on trouve dans les ouvrages desmatières tertiaires est :taux =valeur finale – valeur initialevaleur initiale× 100 .C’est pourquoi nous avons choisi de donner un tauxd’évolution « en % ». Hélas, on trouve dans cesouvrages ce type de calcul : t = 12−1010 ×100 = 2 % .Mais nous conseillons de ne pas pénaliser les élèvesqui écrivent un tel résultat (ils ne seront pas pénalisésle jour du Bac). Il n’est pas facile de faire comprendreaux élèves que 0,02 se traduit par 2 % .Bien que le programme de STG précédent indiquait0,02 = 2 % , nous avons évité cette écriture et avonsréservé la notation 2 % pour les phrases de conclusion.• Le programme indique que « les élèves doivents’approprier le coefficient multiplicateur comme outilefficace de résolution de problèmes ». Toutes lessituations portant sur les évolutions utilisent cet outil.Il est impératif que les élèves sachent traduire uneaugmentation de t % par CM = 1 +t. 100Cependant il est bon que les élèves conservent en têtequ’un pourcentage d’évolution porte sur un nombre !Les calculettes avec la touche % donnent des idéesfausses aux élèves : ainsi pour une hausse de 2 % d’unprix 60 €, on trouve sur les copies :60 + 2 % = 61,2 ou 60 × 2100= 61,2 ou 60 +2100 = 60,02 .Insister : « c’est un pourcentage DE QUOI ? ».Le symbole « = » n’est pas toujours compris. Il fautsavoir corriger les abus d’écriture dans des calculssuccessifs que l’on trouve toujours dans des copies enTerm STMG, car c’est ainsi que les élèves procèdentsur une calculette « 4 opérations et % ».Lors des suites géométriques, pour une augmentationde 2 %, nous avons écrit l’étape intermédiaire :u n+1 = u n + 2100 × u n = 1,02 u n .• Le taux d’évolution réciproque peut être interprétéde deux façons, d’où les deux formules proposées :soit le produit CM × CM’ = 1 (dans à retenir »soit × CM a pour réciproque ÷ CM (dans les conseils del’exo résolus).• La notion d’indice, base 100, est importante dans lesdonnées sociales pour comparer des évolutions et lirerapidement des taux d’évolution. De nombreuxjournaux économiques présentent des évolutions enindices (voir exercices 44 et 74 ) .Un indice base 100 n’est que le CM ramené à 100.Nous avons choisi de l’associer, dans la mise enpratique, au tableau de la « règle de trois ».Les indices synthétiques sont des moyennes pondérées(panier de la ménagère…).• Le taux moyen est la vraie nouveauté et une réelledifficulté pour les élèves.L’essentiel est de faire comprendre que si on connaîtl’évolution sur 4 années, on a le coefficientmultiplicateur sur 4 ans, or 1 an est 1 de 4 ans, donc le4coefficient multiplicateur sur 1 an est CM global élevéà la puissance 1/4… et remplacer ensuite 4 par n pourobtenir la formule générale. Nous préconisonsd’utiliser au maximum la calculatrice, y compris lesexposants décimaux. Nous utilisons cette façon deprésenter le taux moyen en ES depuis 20 ans avecsuccès. Nous ne conseillons pas d’utiliser la notationracine n-ième, trop compliquée et rappelant demauvais souvenirs.Livre du professeur - Mathématiques Term STMG © Hachette Livre 2013 25


Le taux moyen sera très utile pour le chapitre 6 sur lessuites et conduit aux suites géométriques. On peutainsi faire des extrapolations ou interpolations.Les calculs financiers utilisent beaucoup le tauxmoyen. Nous avons choisi de réserver ce thème auchapitre 6 sur les suites. Cela va permettre de revenirsur ce type de calcul.Nous avons réservé la notion de taux proportionnel auchapitre 6, comme conseillé dans le programme.Avant de commencerLes QCM, indépendants, permettent d’entrer dans lechapitre à travers des situations utilisant desconnaissances de premières ou de la vie quotidienne.Les QCM 4 et 5 nécessitent l’emploi de la calculatrice.Ne pas hésiter à montrer en quoi les réponses faussesle sont.QCM 1 : Réponse b) Pour reprendre la notion de tauxd’évolution et de coefficient multiplicateur.En travaillant les QCM 1 et 3 , on peut introduire letaux réciproque.QCM 2 : Réponse b) Savoir prendre des informationsdans un article. Un élève donnant la réponse c) ne litpas vraiment le texte : il ne regarde que les nombres.QCM 3 : Réponse c) Pour bien ancrer la notion devaleur initiale et valeur finale et aborder le tauxréciproque.Ne pas confondre 50100 ; 50200et50250 .QCM 4 : Réponse b) Introduire la notion de taux globald’évolution. La calculatrice est nécessaire.La réponse a) est donnée si l’élève effectue :153 – 180= - 27 et 183,6 – 153 = 30,6 .La réponse c) est du même ordre : 183,6 – 180 = 3,6 .On peut aller plus loin et mettre en valeur l’opération :153× 183,6= 0,85 × 1,2 = 1,02 .180 153QCM 5 : Réponse b) Introduire le taux moyen. Dansun premier temps, les élèves peuvent se contenterd’une vérification : 1,3 × 1,3 = 1,69 .On peut alors utiliser la racine carrée, si la classe lepermet.Pour ceux qui ont plus de difficulté, prendre un prix de100 € avant la mutation d’Alice et augmenter de 30 %,puis de 30 % .En divisant le taux par 2 , on obtient un tauxproportionnel 34,5 %, soit une augmentation globalede 80,9 % : 1,345 ² ≈ 1,8090 .CoursPourcentage d’évolutionÉtude d’une situationPenser à traduire au tableau le texte par un schéma ouun tableau et le remplir au fur et à mesure desquestions.On peut ignorer les questions et demander aux élèvesde compléter ce tableau avec les outils qu’ilsconnaissent. Les questions posées aideront à placer levocabulaire sur des notions connues.Il faut que les élèves comprennent ce qui est demandédans un exercice. vocabulaire lié à l’économie p. 2111.a) Poste recherche : variation absolue 10 000 .Variation relative (reprendre la formule connue desélèves)50 000−40 00040 000= 0,25 .b) 10 000 représente 25 % de 40 000 , ce qui donne letaux d’augmentation.10 000 représente 125 % de 50 000 , soit 125=1,25 . On100peut alors reparler du CM .2. Poste salaires : prendre en compte les différentespropositions des élèves pour obtenir le résultat :mettre en valeur l’utilisation du CM .350 000 + 350 000 × 8= 350 000 × 1,08 = 378 000100plus rapide à calculer à la calculatrice !3. a) CM = 1 – 15100 = 0,85 .Pour certains élèves il sera peut être nécessaire deprendre 100 € pour obtenir 85 € .b) Un schéma aide beaucoup : pour une multiplication,revenir en arrière c’est faire une division.Parler d’opération réciproque etéviter « opération contraire » !680000,85= 80 000 .Objectifs des exercices : Comprendre qu’uneaugmentation peut dépasser 100 %, faire accepterl’efficacité du CM dans le calcul des évolution, enparticulier pour obtenir la valeur initiale ou valeur dedépart . Les exercices 1, 2 et 3 sont des extraitsd’articles de journaux économiques : on y donnesouvent la valeur d’arrivée et le taux d’évolution.Nous avons privilégiez le calcul du taux à partir du CMen effectuant CM × 100 – 100 , pour éviter le problèmedes parenthèses oubliées chez certains élèves etLivre du professeur - Mathématiques Term STMG © Hachette Livre 2013 26


évoquer le « pourcent » chez les plus faibles. Un CM de0,85 revient à chercher comment compléter 85centimes à 100 centimes (ou 1 euro). De plus, ce typede calcul utilise la propriété de l’indice base 100.Évolutions successives – Taux réciproqueÉtude d’une situationPlacer au fur et à mesure des questions les résultatsobtenus sur le schéma.1. Effectuer les calculs en utilisant les CM .a) y 2 = 5 × 1,2 = 6 . b) y 3 = 6 × 1,25 = 7,5 .c) Plus de 45 %. Faire calculer le % d’augmentation :demander aux élèves leur façon de calculer et mettreen valeur ceux utilisant les CM .d) y 3 = 6 × 1,25 = 5 × 1,2 × 1,25 = 5 × 1,5Proposer une autre valeur de départ du chiffred’affaires pour faire apparaitre le CM global qui nechange pas; puis proposer d’autres taux d’évolutionpour faire apparaître la formule du CM global :CM global = CM 1 × CM 2 .3. a) Mettre en place le taux réciproque, en privilégiantla division dans un premier temps (revenir à la valeurde départ comme vu en cours 1).On divise 7,5 par 1,5 pour obtenir 5.b) Le CM’ est 0,6666 , soit une baisse de 33,33 % plusfaible qu’une baisse de 45 % .Faire apparaître que le produit des deux coefficientsmultiplicateurs CM × CM’ donne 1 .Faire travailler avec d’autres CM :hausse de 100 % , baisse de 50 % …Lire t/100 au lieu de t 1 /100 dans « Ce qu’il fautretenir ».On peut parler de « taux compensatoire ».Objectifs des exercices : Comprendre qu’il est inutiled’avoir la valeur de départ pour calculer un tauxd’évolution global ou un taux réciproque.Nous pensons aux élèves sortant d’un BEP : pourcertains d’entre eux, il sera nécessaire de leurpermettre de prendre 100 pour une valeur initiale pourvérifier leurs résultats à la calculatrice et les rassurer.Pour une hausse de 20 % , nous avons garderCM = 1,20 , pour « voir » les 20 centimes pour 1 euro.Remarque : Les chiffres donnés pour les paludiers(baisse forte de la production après le naufrage d’unpétrolier) et la société Stokomani sont réels.Indice simple, base 100Étude d’une situationLe tableur se prête au calcul d’indices : il est rare quel’on ne calcule qu’un seul indice. Soit on étudiel’évolution d’une grandeur par rapport à sa valeur à uninstant donné, soit on compare l’évolution de deuxgrandeurs d’échelle de valeurs différentes.C’est ce que nous avons choisi de montrer.Du fait des notations y 1 et y 2 employé dans le cours 1et 2 , nous avons été « obligé » de nommer I 1 l’indicede base. Dans les ouvrages économiques, on trouveplutôt I 0 .1. On retrouve un tableau de proportionnalité.a) En D4 : = D2 / $B2 * 100 .Le $ permet de bloquer la colonne indiquant la base.La formule peut alors être « tirer » vers la gauche, ladroite et vers le bas pour le bénéfice.b) Taux 10 % : 110 – 100 = 10 ( faire vérifier avec lesvaleurs 50 et 55 si nécessaire)c) Taux – 6 % : 94 – 100 = - 6 .2. Pour le bénéficea) Taux 5 % : 3,783,6× 100 – 100 = 105 – 100 = 5 ;même augmentation que le CA.b) Taux 11,1 % : 111,1 – 100 = 11,1 .c) Le bénéfice augmente plus que le CA.Objectifs des exercices : pour l’exercice résolu,comparer deux évolutions, l’une croissante(fortement) et l’autre décroissante. Savoir calculer etinterpréter des indices. L’exercice 8 confortel’utilisation du tableur vu dans la situation. On peutretrouver ce type de question au Bac, en particulierdans des QCM .Pour certains élèves il sera bon de parler de règle detrois, dont parlent les professeurs des matièrestechniques.Racine n-ième – Taux d’évolution moyenÉtude d’une situationLa notion de CM global doit être bien ancrée, pour quele calcul du taux moyen soit bien compris.Ne pas insister sur les propriétés des exposants et laformule ( a n ) p = a n × p , rappelant de mauvaisLivre du professeur - Mathématiques Term STMG © Hachette Livre 2013 27


souvenirs à ces élèves, Mais plutôt donner du sens àcet exposant 1/n , qui correspond au partage en nparties égales de la période de temps considérée.Avoir en tête que le but est de faire comprendre lestaux équivalents dans les calculs financiers :taux mensuel : 1/12 ; taux journalier : 1/360 …a) CM global = 792= 1,44 . On peut noter CMg .550b) CM 1 an = 1+ t; donc CM global = 1 + t 2100 100 .D’où l’équation à résoudre 1 +t 2= 1,44 .100c) 1,2 a pour carré 1,44 et 1,2 = 1,44 .On peut parler de « racine carrée » qui aideront àintroduire les mots « racine n-ième » du titre.d) Ainsi 1 + t= 1,20 et donc t= 20 .100On peut proposer aux élèves de vérifier à lacalculatrice que 1,44 0,5 donne aussi 1,2 .Puis aller plus loin en considérant 1 140 paniers en 4ans , vérifier que 550 × 1,2 4 ≈ 1 140 , puis cherchercomment calculer le taux annuel moyen 1,2 à partir dutaux global de 2,074 sur 4 ans.Objectifs des exercices : comprendre la notion de tauxéquivalent. Les taux globaux employés sont trèsimportants pour vraiment faire la différence avec lestaux proportionnels.Dans un premier temps, nous avons sciemment choiside ne pas faire calculer les taux proportionnels pourque les élèves ne fassent pas la confusion. Ils ne serontabordés qu’au chapitre 6 sur les suites géométriques. p. 168Les programmes ne mentionnent plus le calculapproché de (1 + t) n par 1 + n × t lorsque t est petit del’ordre de 1 ou 2 %. Nous conseillons de demander auxélèves un nombre suffisant de chiffres après la virgulepour les CM : 4 chiffres après la virgule ou au moins 3chiffres significatifs ( p. 212 ) pour des taux petitsinférieurs à 2 %. Autant que faire ce peut, habituer lesélèves à n’écrire qu’une seule opération sur lacalculatrice pour le calcul du taux moyen à partir desvaleurs de départ et d’arrivée, afin de leur fairecomprendre les erreurs dues aux arrondis.Dans la vie quotidienne, on rencontre des taux del’ordre de 0,2 % , 1,25 % …Un programme est proposé à l’exercice 79 p. 55.Atelier TICEFréquentation des campingsBut : Utiliser le tableur pour calculer des indices.Le graphique paru dans la presse s’avère faux : lapartie A va permettre de voir que les légendes ont étéinversées. De très nombreux graphiques de la Presseprésentent des incorrections. La partie B va permettrede réaliser le graphique correct.A. 1. Pour les campings 0 ou 1 étoilea) Indice 2011 base 100 en 2004 : 3855× 100 ≈ 90,5 .4261b) L’indice sur le graphique semble inférieur à 90.2. Pour les campings 2 étoilesa) Indice 2011 base 100 en 2004 : 24338× 100 ≈ 89,2 .27274b) L’indice sur le graphique semble supérieur à 90 .Le graphique est incorrect.B. Avec un tableur1.a) La formule en C11 devient en I11 :=I3/$B3*100b) En C14, cette formule devient =C6/$B6*100On obtient alors l’indice en 2005 pour le nombre denuitées des campings 4 étoiles, base 100 en 2004 .2. Si on écrit en C11 : =C3/$B$3*100 , on obtient enC14 : =C6/$B$3*100 donnant l’indice relatif auxcampings 0 ou 1 étoile en 2004.3. a) Voir ci-dessous.La cellule I11 contient le résultat calculé en A.1.a) et lacellule I12 contient le résultat calculé en A.2.a).b) On obtient le graphique ci-dessous, qui montre bienl’erreur de légende commise dans le quotidien.Livre du professeur - Mathématiques Term STMG © Hachette Livre 2013 28


Tableau de valeurs Atelier 1Consommations alimentairesBut : Utiliser un algorithme pour déterminer un tauxmoyen. Comparer des taux moyens annuels pourplusieurs produits.A. 1. Valeur de départ D = 8,6 en cellule B7.Valeur d’arrivée V = 21,8 en cellule F7.Nombre d’années N = 2008 - 1970 = 38 .2. Taux global G , en % :G = ( V/D - 1 )×100 =21,88,6− 1 × 100 ≈ 153,493.Taux annuel moyen T , en % :T = ( ( V/D )^( 1/N ) – 1) × 100=(4. Algorithme sous AlgoBox21,88,6138− 1) × 100 ≈ 2,48Utiliser pow( V/D , 1/N ) pour la puissance.On obtient pour la consommation de yaourts :Attention à l’écriture des opérations : bien cliquer surles variables déclarées V, D et N .Cigarettes : prix – consommationBut : Vérifier la maitrise du tableur pour des calculsd’indice, de aux d’évolution global ; moyen etréciproque.On remarque que les taux réciproques sont de mêmesgrandeurs :– 46,7 % pour le prix et 52,5 % pour les ventes.Résultats : voir ci-dessous.Tableau de valeurs Atelier 2Livre du professeur - Mathématiques Term STMG © Hachette Livre 2013 29


Tableau de valeurs Atelier 3QCM page 47 pour répondre à un QCM p. 1951. b. Les investissements augmentent de 17 % ; donc lavaleur en 2010 est plus petite: la réponse c. n’est paspossible.Les investissements sont multipliés par 1,17 .D’où la valeur initiale pour 2010 :valeur en 2011= 257≈ 219,7 .CM1,17On peut aussi tester les deux valeurs :257−213,3×100 ≈ 20,5 ; 257−219,7×100 ≈ 17213,3219,72. c Valeur de départ du véhicule :10 990 + 3 760 = 14 750Donc t =14 750 − 1099010 990× 100 ≈ – 25,5 .3. b Il faut traduire chaque taux d’évolution par soncoefficient multiplicateur et calculer :CM global = 1,12 × 0,85 × 1,30 = 1,2376 .D’où le taux d’évolution sur les trois mois de 23,76 % .Attention : les taux d’évolution ne s’ajoutent pas !4. c Pour une baisse de 36 %, le coefficient multiplicateurest CM =1 – 0,36 = 0,64 .Or le coefficient multiplicateur réciproque est :CM’ = 1= 156,25= 1,5625 et 1,5625 – 1 = 0,5625 = .CM 0,64 100Pour retrouver le chiffre d’affaires initial, la hausse doit êtrede 56,25 % pour compenser la baisse de 36 % .On peut aussi tester si le produit des coefficientsmultiplicateurs donne 1 :pour la réponse a. : 1,36 × 0,64 = 0,8704pour la réponse b. : 1,40 × 0,64 = 0,896pour la réponse c. : 1,5625 × 0, 64 = 1 .5. a On calcule l’indice :valeur en 199039,1× 100 = × 100 ≈ 75,5 .valeur en 2000 51,86. b Comme l’indice en 2010, base 100 en 2000, vaut 152,7alors, par lecture directe, la production a augmenté de52,7 % (pour une base 100) .7. c Comme le nombre de titres est 51,8 milliers en l’annéede base 2000, et l’indice est 152,7 en 2010 :51,8 × 1,527 = 79,0986.Donc 79,1 milliers de titres ont été édités en 2010.8. c De 2006 à 2011, il y a 5 évolutions annuelles. Durantcette période, les ventes ont augmenté de 89,8 % .Donc CM global = 1,898 .Et on cherche la racine 5 e du CM global :1CM 1 an = 1,898 5 ≈ 1,1367 .D’où le taux annuel moyen de 13,67 % , arrondi à 0,01 %près.La réponse b. correspond à 6 évolutions.Ce n’est pas la réponse a. car on ne divise pas le taux par 5.(Le taux moyen n’est pas le taux proportionnel.) vocabulaire économique p. 211On peut aussi tester les taux t donnés en calculant( 1+ ) 5 pour retrouver le CM global 1,898 :1001,1367 5 ≈ 1,898 et 1,1127 5 ≈ 1,705 .Livre du professeur - Mathématiques Term STMG © Hachette Livre 2013 30tCorrigé des exercices du chapitre 2ex1 CM = 12−22≈ – 0,454522Les exportations de blé russe ont diminué de 45 % (arrondià 1 % près) entre 2012 et 2013.ex2 a) Récolte de blé en 2011 : 56 millions de tonnes.b) CM = 49= 0,875. Et 0,875 × 100 – 100 = -12,5 .56Entre 2011 et 2012, la récolte a diminué de 12,5 % .ex3 Toutes les valeurs connues sont les valeurs d’arrivée en2012.Dans les deux cas, on cherche la valeur de départ en 2011.a) Diminuer de 14 %, c’est multiplier par :CM = 1 – 14= 0,86 .100valeur d′arrivéeD’où valeur de départ : = 743≈ 863,95 .CM0,86En 2011, les stocks de blé aux États-Unis étaient de 864millions de boisseaux, arrondis au million près.b) On a CM = 1,08 et 667≈ 617,59 .1,08En 2011, les stocks de soja étaient de 617,6 millions deboisseaux, à 0,1 près. calculs et arrondis p. 213ex4 CM global = ( 1 – 20100 ) ( 1 – 15100 ) ( 1 – 10100 )= 0,8 × 0,85 × 0,9 = 0,612 .Et 0,612 × 100 – 100 = – 38,8 . L’action a perdu 38,8 % .


ex5 a) Pour une hausse de 50 %, le coefficientmultiplicateur est CM = 1,50 .Le coefficient multiplicateur de l’évolution réciproque est :CM’ = 1= 1≈ 0,6667 . 0,6667 × 100 – 100 = - 33,33 .CM 1,5Une hausse de 50 % est compensée par une baisse de 33,33% , arrondi à 0,01 % près.b) Pour une baisse de 50 %, le coefficient multiplicateur estCM = 0,5.CM’ = 1= 1= 2 et 2 × 100 – 100 = 100 .CM 0,5Le taux d’évolution réciproque d’une baisse de 50 % est unehausse de 100 % .ex6 Pour une baisse de 95 % , le coefficient multiplicateurest CM = 1 - 0,95 = 0,05 .CM’= 1= 11900= 20 et 20 – 1 = 19 =CM 0,05 100Le taux d’évolution l’année suivante doit être de 1900 % .Rappel : une multiplication par 4 donne une augmentationde 300 % .ex7 a) CA en 2012 : 186 × 1,20 = 223,2 M€ .CA en 2010 : 186≈ 164,602 M€ à 1 millier d’euros près.1,13b) On détermine le taux d’évolution réciproque de lahausse de 20 % constatée en 2012.Le coefficient multiplicateur de la hausse de 20 % est CM =1,20. Celui de l’évolution réciproque est CM’ = 1≈ 0,8333 .1,20,8333 × 100 – 100 = -16,67 .Une baisse du CA de 16,67 % en 2013 ramènera le CA à savaleur de 2011.ex8 a) En D5, 104,6 est l’indice 2011 des ventes HP, base100 en 2009 . En E5, 87,4 est l’indice 2012 des ventes HP,base 100 en 2009.b) La formule = B2/$B2×100 saisie en B4, devient=D2/$B2*100 en D4 .c) En B5 , la formule de B4 devient = B3/$B3×100Le $ bloque la colonne B , mais pas la ligne 2 qui devient 3 .En E5, cela devient = E3/$B3×100.ex9 a) Indice 2011 des ventes de PC Acer, base 100 en2009 : 9,62× 100 ≈ 82 .11,7L’indice 2012 des ventes de PC Acer, base 100 en 2009 :8,63× 100 ≈ 74.11,7b) 74 = 100 – 26 . Entre 2009 et 2012, les ventes de PC Aceront diminué de 26 % .ex10 CM global = 54,983,4 .De 2001 à 2004, il y a 3 évolutions annuelles.On calcule alors la racine cubique.54,993,413 ≈ 0,8699 et 0,8699 × 100 – 100 = – 13,01De 2001 à 2004, le taux annuel moyen de baisse descigarettes vendues est de – 13 % .ex11 a) Entre 2003 et 2004, le nombre de cigarettesvendues est passé de 69,6 milliards à 54,9 milliards.CM = 54,9≈ 0,7888 et 0,7888 ×100 – 100 = -21,12 .69,6Entre 2003 et 2004, le nombre de cigarettes vendues abaissé de 21,12 % .b) Sur une année, il y a 12 évolutions mensuelles. Oncalcule la racine 12 e du coefficient multiplicateur annuel :54,969,6112 ≈ 0,9804 et 0,9804 × 100 – 100 = – 1,96 .Taux mensuel équivalent à ce taux annuel : – 1,96 % .Pourcentage d’évolutionex12 a) 1,32 b) 0,81 c) 1,03 d) 0,94 e) 3ex13 a) + 8 % b) – 54 % c) – 0,75 % d) + 32,8 %ex14 a) + 0,035 % b) + 125 % c) – 85,5 % d) + 4,5 %ex15 pour répondre à un Vrai-Faux p. 196valeur en 2010CM = = 1,222≈ 1,270 .valeur en 2009 0,962a) Vrai : 1,27×100 -100 = 27 .b) Faux , car a) vrai.c) Vrai, c’est le CM.ex16 CM = 1,035 .ex17 La valeur de l’action a diminué de 15,8 % .ex18 CM = 24502744 ≈ 0,8929 .La subvention a diminué de 10,71 % à 0,01 % près.ex19 Prix TTC = prix HT × ( 1 + 19,6) 100= 12 150 × 1,196 = 14 531,4 € .Rappel : le taux de TVA s’applique au prix hors taxe :Montant TVA = Taux × Prix HTLe prix TTC est le prix hors taxe augmenté du montant de laTVA .ex20 Prix HT =prix TTC1+ 7100= 481,501,07 = 450 € .Nouveau prix TTC = 450 × 1 + 10100= 495 € ..ex21 a) Prix HT =Montant TVATaux décimal = 138,180,196 = 705 € .Prix TTC : 705 + 138,18 = 843,13 € .b) TVA à 25 % : Prix TTC = 705 × 1,25 = 881,25 € .Livre du professeur - Mathématiques Term STMG © Hachette Livre 2013 31


ex22 QCM : La seule bonne réponse est a.65,2 – 1,2 = 64 . Pour le panel choisi, le taux d’occupationen 2011 était de 64 % .78 % est une part en pourcentage et 3,7 % le pourcentagede hausse maximal du revenu par chambre.ex23 1. En B3 on saisit = B2/B1En B4 on saisit = (B3 - 1) × 100 .2. Tableau des valeursUtiliser le boutonpour arrondir.ex24 1. En D3 on saisit =1 + E3/100 .En B4 on saisit =C4/D4 . En C5 on saisit =B5*D5 .En B6 on saisit =C6/D62. Tableau des résultatsex31 CM global = 1,1² × 0,95² ≈ 1,0920 .Globalement, le loyer a augmenté de 9,2 % .ex32 a) 1= 1= 0,8 .CM 1,25Le taux réciproque de + 25 % est – 20 % .b) 1= 1≈ 3,3333 .CM 0,3Le taux réciproque de – 70 % est + 233,33 % .c) 1= 1≈ 0,9524 et 0,9524 × 100 – 100 = - 4,756 .CM 1,05Le taux réciproque de + 5 % est - 4,76 % .d) 1= 1 1CM 1− 8,5 = ≈ 1,0929 .0,915100Le taux réciproque de – 8,5 % est + 9,29 % .ex33 Pour une baisse de 7 % , le coefficient multiplicateurest CM = 0,93 .Le coefficient multiplicateur de l’évolution réciproque est :CM’ = 1= 1≈ 1,0753 .CM 0,93Et 1,0753 × 100 – 100 = 7,53 .Les ventes de voitures doivent augmenter de 7,53 % pourcompenser cette chute.ex34 1. a) En cellule C2, on lit le deuxième taux d’évolutionde – 20 % .b) En C3 la formule écrite en B3 devient = 1 + C2/100et en D3 la formule devient = 1 + D2/100 .c) Tableau des résultatsÉvolutions successives-Taux réciproqueex25 a) CM global = 1,2 × 1,5 = 1,8 .b) CM global = 1,5 × 1,2 = 1,8 .Le taux d’évolution global ne dépend pas de l’ordre desévolutions, c’est une propriété de la multiplication.d) 0,966 est le coefficient multiplicateur global pour lestrois évolutions successives.e) En E2 on saisit la formule = (E3 – 1)*100 .2. Tableau des nouveaux résultatsex26 a) CM global = 1,2 × 0,8 = 0,96 .b) A l’arrivée le revenu est 96 000 € .ex27 CM global = 1,07 × 1,13 = 1,2091 .D’où une hausse globale de 20,91 % .ex28 CM global = ( 1 - 26,7100 ) × ( 1 - 13,6100 ) ≈ 0,6333et 0,6333 × 100 – 100 = -36,67 .Sur les deux ans, 2011 et 2012, le nombre de candidats auconcours a diminué de 36,67 % .ex29 CM global = 0,95 × 1,16 = 1,102 .Le bénéfice a augmenté de 10,2 % .ex30 CM global = 1,06 3 = 1,191016 .Le prix a augmenté de 19,1 % , à 0,01 près.ex35 Le coefficient multiplicateur de la deuxième évolution,noté CM , vérifie 1,08 × CM = 1,35 . D’où CM = 1,351,08 = 1,25 .La 2 e année, les exportations ont augmenté de 25 %.ex36 On connait CM global = 1 –Livre du professeur - Mathématiques Term STMG © Hachette Livre 2013 3228100= 0,72 et le coefficientmultiplicateur de la 2 e évolution : CM 2 = 1 - 25100 = 0,75 .Le coefficient multiplicateur de la première évolution, notéCM 1 , vérifie CM 1 × CM 2 = CM global .CM globalD’où CM 1 = = 0,72= 0,96 .CM 2 0,75Le prix de l’appareil a diminué de 4 % la première année.


ex37 CM global = 1,2 × 1,3 = 1,56 .On cherche la valeur initiale C du capital, connaissant lavaleur finale VF = 35 000 € et le coefficient multiplicateurglobal de l’évolution. D’où :C = VF= 35000≈ 22 435,90 .CMg 1,56Au départ, le commerçant doit placer 22 436 € .ex38 Le coefficient multiplicateur d’une baisse de 50 % est :CM = 0,5 = 1 . On parle aussi d’une diminution de moitié .2Le coefficient multiplicateur de l’évolution réciproque est :CM’ = 1= 1= 2 .CM 0,5Et 2×100 – 100 = 100 :multiplier par 2 revient à augmenter de 100 % .L’action doit augmenter de 100 % pour compenser la baissede 50 % .ex39 a) CM global = 0,82 × 1,06 × 1,12 ≈ 0,9735 .b) Comme le coefficient multiplicateur global est inférieur à1, cette cote n’est pas revenue à sa valeur de départ.c) t = 0,9735 × 100 – 100 = - 2,65 .À la suite des trois évolutions successives, le tauxd’évolution global de la cote est – 2,65 % .d) Le coefficient multiplicateur de l’évolution réciproque estCM’ = 1= 1= 1≈ 1,0272 .CM 1− 2,65 0,9735100La cote de cette valeur boursière revient à sa valeur dedépart après les trois évolutions successives si elleaugmente de 2,72 % .Indice simple, base 100ex40 a) 2011 est l’année de base 100 .b) Sur la période 2011-2012, l’indice passe de 100 à 115, ilaugmente de 15 % .c) Sur la période 2011-2013, l’indice augmente de 20 % .d) 4 000 × 1,20 = 4 800 .Un prix de 4 000 € fin 2011, devient 4 800 € fin 2013.ex41 On résume les données dans un tableau ; conseils pour le Bac p. 194année 2008 2013 2014valeur 300 180 270indice 100 I 1 I 2a) En 2013 : I 1 = 180× 100 = 60 .300b) Or 60 – 100 = - 40 .La valeur a donc perdu 40 % de 2008 à 2013 .c) En 2014 : I 2 = 270× 100 = 90 .300ex42 QCM1. Réponse c. Tableau des résultatsannée 2011 2012quantité 345 407indice 100 I AI A = 407× 100 ≈ 1183452. Réponse b Tableau des résultatsannée 2011 2012quantité 2500 2730indice 100 I BI B = 2730× 100 ≈ 1092500ex43 QCM1. Réponse b. CM global = 1,15 × 1,10 × 0,95 = 1,20175 , soitune hausse de 20,175 % .2. Réponse a. 1,063 3 ≈ 1,201 .3. Réponse b . 1,20175 × 0,8 = 0,9614 .Sur les 4 années, le coefficient multiplicateur global estinférieur à 1 . Le bénéfice global diminue, le tauxd’évolution global est négatif.ex44 1. 2 × 13,8 = 9,2 . En 2012, Zara réalise un chiffre3d’affaires de 9,2 milliards d’euros.2. a) Entre le 2 janvier et le 7 août 2012 , la cote d’inditexaugmente de 40,89 % .b) 81,52 – 100 = -18,48 .Dans le même temps l’indice Ibex diminue de 18,48 % .3. On résume les données dans un tableau :date 2 janvier 2012 7 août 2012Indice 100 140,9chiffre d’affaires ? 13,8La valeur du CA au 2 janvier 2012 est donnée par :Livre du professeur - Mathématiques Term STMG © Hachette Livre 2013 3313,8140,9× 100 ≈ 9,8 milliards d’euros.1360 ×116ex45 1. = 1577,6 .100À la dizaine près, 1 580 spectateurs sont entrés dans cettesalle la 5 e semaine.2. a) Tableau des résultatssemaine 1 2 3 4 5 6nombre despectateurs604 950 1360 785 1578 3207indice 44 70 100 58 116 236b) 236 – 100 = 136 . De la 3 e à la 6 e semaine, le nombred’entrées a augmenté de 136 % .ex46 a) Indice en 2007 : I 1 = 4350× 100 = 115 .3780De 2006 à 2007, l’indice de l’épargne est passée de 100 à115 : l’épargne a donc augmenté de 15 % .b) Indice en 2008 : I 2 = 114 . Indice en 2010 : I 4 = 97 .c) Variation relative totale sur la période 2006-2009 :valeur en 2009−valeur en 2006valeur en 2006= 2609−37803780≈ – 30,98 .


d) De 2006 à 2009, l’indice passe de 100 à 69 .Or 69 – 100 = – 31 .Racine n-ième – Taux d’évolution moyen14751250× 100 ≈ 124 . Ainsi, en moyenne, chaque année, le prix de la voiture a1190 1190 diminué de 9,71 % .L’épargne a donc diminué de 31 % sur cette période.ex 49 Le nombre réel x dont le cube est 7 vérifie x 3 = 7 .Donc les écritures x = 7 3 et x = 7 sont fausses.3ex47 1. a) en C3 , on place la formule = B3/B$2*100 .b) Tableau des résultatsLa racine cubique de 7 s’écrit :7 1 33 et 7 et vaut 1,913 à 0,001 près.ex50 De 1990 à 2009, il y a 19 évolutions annuelles.Le taux d’évolution moyen annuel t vérifie l’équation :1( 1 + t ) 19 = 7,27,24,7 4,719 .On ne demande pas le calcul.ex51 a) ≈ 1,1487 b) ≈ 1,1356c) ≈ 0,7401 d) ≈ 1,0006c) L’indice 110 obtenu en 2006 signifie que le prix du baril aaugmenté de 10 % entre les années 2000 et 2006.ex52 a) ≈ 1,0334 b) ≈ 0,89242. a) De 2008 à 2009 : CM = 50,975,7c) ≈ 1,1335 d) ≈ 1,0275et 0,6724 × 100 – 100 = – 32,76 .Le prix du brent a baissé de 33 % , à 1 % près .ex53 a) 1,78 1 3 ≈ 1,2119 .De 2008 à 2012 : CM = 90,6b) Une hausse de 78 % sur trois ans équivaut à un taux75,7annuel moyen d’augmentation de 21,19 % .Le prix du brent a augmenté de 20 % , à 1 % près.Attention: le taux annuel moyen n’est pas le tauxb) On en déduit le tableau suivant :proportionnel de 26 % :année 2008 2009 2012Taux proportionnel = 78prix en € 75,7 50,9 90,63indice 100 100-33 = 67 120ex54 a) 1,6 112 ≈ 1,0399 .b) D’après la question a), le taux mensuel moyen équivalentex48 1. Formule saisie en C3 = C2/$B$2 *100 .à une hausse annuelle de 60 % est 3,99 % .2. a) On applique la formule précédente donnant l’indice dela population en 2002, base 100 en 1990.ex55 a) x 20 = 2 x = 2 120 ≈ 1,0353 .1475× 100 = 118 .1250 b) Le coefficient multiplicateur du taux annuel moyenLa population a augmenté de 18 % entre 1990 et 2002.De 2002 à 2013, la population est multipliée par :d’accroissement d’une population qui double en unegénération de 20 ans est la solution positive de l’équationCM = 1190≈ 0,8068 et 0,8068 × 100 – 100 = -19,32 .résolue en a), c’est-à-dire 1,0353 .1475À 1 % près, la population a diminué de 19 % entre 2002 et D’où le taux cherché 3,53 % .2013.b) De 1990 à 2013 : CM global = 1190ex56 a) x 6 = 0,5 x = 0,5 1 6 ≈ 0,8909 .1250b) Le coefficient multiplicateur du taux d’évolution annuelet 0,952 × 100 – 100 = – 4,8 .moyen de cette production est 0,8909.De 1990 à 2013, la population a diminué de 4,8 %.0,8909 × 100 – 100 = - 10,91 .D’après les résultats de la question 2. a), on remarqueOn en déduit qu’une baisse de 50 % sur 6 ans équivaut à unqu’une augmentation de 18 % suivie d’une baisse de taux annuel moyen de diminution de 10,91 % .19 % ne correspond pas à une baisse de 1 %.3. On écrit 100 en cellule D4 .ex57 a) En 5 ans, le prix de la voiture baisse de 40 %, doncle prix est multiplié par 0,6 .Soit t le taux d’évolution annuel moyen (en écrituredécimale) du prix de cette voiture. On résout l’équation :(1 + t) 5 = 0,6 1 + t = 0,6 1 5 t =0,6 1 5 - 1 ≈ - 0,0971 .Livre du professeur - Mathématiques Term STMG © Hachette Livre 2013 34


) La baisse s’applique au prix de 12 000 :12000 × 9,71100 = 1 165,2 .La perte est de 1 165,20 € la 1 re année, si le taux moyens’applique dès la première année.ex58 1. Entre 1998 et 2005, l’indice des prix augmente de12,70 % et entre 1998 et 2012, il augmente de 26,63 % .2. a) 126,63≈ 1,1236 . Entre les années 2005 et 2012,112,70l’indice des prix est multiplié par CM = 1,1236 .b) On en déduit que l’indice des prix sur cette période aaugmenté de 12,36 % .3. Soit t le taux d’évolution moyen annuel de l’indice desprix entre 2005 et 2012 ( t en écriture décimale) .De 2005 à 2012, il y a 7 évolutions. On résout l’équation :(1 + t) 7 1= 1,1236 1 + t = 1,1236 71 t =1,1236 7 – 1 ≈ 0,0168 .Ainsi, en moyenne, l’indice des prix a augmenté chaqueannée de 1,68 % .ex59 1. CM = 250≈ 2,3810 et 2,3810 × 100 – 100 = 138,1 .105De 1981 à 1986, la dette de l’état français a augmenté de138,1 % .Le coefficient multiplicateur du taux d’évolution annuelmoyen de la dette sur cette période est la solution del’équation x 5 = 250105 x = 25010515 ≈ 1,1895 .Ainsi, de 1981 à 1986 , la dette a augmenté en moyenne de19 % par an.2. De 2008 à 2011, il y a trois évolutions annuelles.On résout l’équation :x 3 = 1717 x = 1717 3≈ 1,0920 .1318 ,6 1318,6De 2008 à 2011, la dette a augmenté en moyenne de 9,2 %par an.3. Pendant la période 2008-2011, l’accroissement annuellede la dette est beaucoup moins forte que durant la période1981-1986.ex60 Pour une augmentation de 3 points de pourcentage,la taxe passe de 7 % à 10 % . Ainsi pour une facture de100 € hors taxe, le montant TTC passe de 107 € à 110 €.CM = 110≈ 1,028 .107La facture a augmenté de 2,8 % .La hausse est la même quel que soit le montant de lafacture. En effet, le coefficient multiplicateur entre les deuxprix TTC est :prix HT × 1,10= 1,10prix HT ×1,07CM = ≈ 1,028 .1,07Ce qui donne une augmentation de 2,8 % et non 3 % .1Remarque : lorsque l’on donne un arrondi d’un taux enpourcentage, on devrait écrire :2,8 % à 0,1 point de pourcentage.ex61 Le nombre d’étudiants internationaux a augmenté de31,3 % entre 2005 et 2009.Ce nombre a été multiplié par CM = 1,313 .valeur d′arrivée 3 324 900valeur de départ = = ≈ 2 532 300 .CM1,3132 532 300 étudiants internationaux dans le monde en 2005.ex62 Le montant des revenus imposables diminue de 16 %entre 2012 et 2013 : donc CM = 1 – 0,16 = 0,84 /Et la valeur d’arrivée est 41 400 €.valeur d′arrivéevaleur de départ =CM41 400On en déduit la valeur de départ ≈ 49 286 .0,84On peut donc conclure : a) Faux b) Vrai c) Vrai .ex63 1. 83,6 % .2. CM = 0,81 .3. t= 6≈ 0,0123 ; hausse d’environ 1,2 % .100 489ex64 1.a) 0,17 × 18 = 3,06 . Ainsi 3,06 millions d’euros dechiffre d’affaires réalisés à l’exportation en 2011.b) Augmentation de 14 points de pourcentage pour la partde chiffre d’affaires provenant de l’exportation entre 2006et 2011.2. Chiffre d’affaires à l’exportation :• en 2006 : 0,03 × 10 = 0,3 million d’euros ;• en 2011 : 3,06 millions d’euros.t= 3,06−0,3100 0,3= 9,2Soit une hausse de 920 % et non 4 % !ex65 Pour l’année 2012, le taux de croissance de laproduction industrielle en Chine s’est élevé à :13,9 % – 5 % = 8,9 % .Lors d’une baisse en points de pourcentage, les % secomportent comme des unités. Aide exo 61Le coefficient multiplicateur correspondant est :CM = 1 + 0,089 = 1,089 .Sur 2011-2012 , on a :CM global = 1,139 × 1,089 ≈ 1,2404.Ainsi, sur la période 2011-2012, la production industriellede la Chine a augmenté de 24 % .ex66 a) CM global = 0,9 3 = 0,729et 0,729 × 100 – 100 = - 27,1 .La production a baissé de 27,1 % en 3 mois.b) 150 000 × 0,729 = 109 350 objets.Livre du professeur - Mathématiques Term STMG © Hachette Livre 2013 35


c) Le coefficient multiplicateur de l’évolution réciproque quipermet de revenir à la production initiale est :CM’ = 1= 1≈ 1,3717 .CM 0,729Une hausse de 37,17 % , à 0,01 % près, compense troisbaisses successives de 10 % .ex67 1. a) C = 1 + A100 × 1 + B100 .b) Cet algorithme permet de calculer le coefficientmultiplicateur global C correspondant à deux évolutionssuccessives A et B, données en pourcentage, et le taux Td’évolution associé exprimé en pourcentage.2. Si A = 20 et B = 30, alors C = 1,56 et T = 56 .ex68 1. a) Formule saisie en C3 = C2/B2b) en C4 = (C3 - 1) * 100 ou bien = C3 × 100 – 100c) en B6 = PRODUIT ( C3 : G3 ) ou = G2/B22. Tableau des résultatsEntre les années 2005 et 2010, en France, les ventes surinternet ont augmenté de 269 % .ex69 Vrai ou Fauxa) Faux : Soit N le nombre total d’inscrits, les 63 étudiantsreprésentent 42 % de N : 0,42 × N = 63 N = 630,42 = 150 .b) Vrai : 150 – 63 = 87 . c) Vrai : 15 = 0,1 × 150 .ex70 QCM1. Réponse a. 448−350× 100 = 28 , soit 28 % .350Hausse de 540 % pour P2 et baisse de 65 % pour P3.2. Réponse b. CM global = 1,1 × 1,2 × 0,85 = 1,122 .ex73 1. Part des ingrédients dans une baguette :0,261,2Part du revenu de l’artisan :0,091,20≈ 0,2167 ; soit 21,67 % .= 0,075 , soit 7,5 % .2. a) CM = 265= 1,325 . De septembre 2011 à septembre2002012, le prix de la tonne de blé a augmenté de 32,5 % .b) 0,26 × 0,325 = 0,0845 .Si on répercute cette augmentation du prix des ingrédients,le prix de la baguette augmente de 0,0845 € , c’est-à-dire0,08 € arrondi au centime près.c) Le prix de la baguette passe de 1,20 € à 1,28 €.0,08≈ 0,0667 .1,20Dans ce cas, la baguette augmente de 6,67 % .ex74 Résumer les données du graphique dans un tableau.2006 2011PQN en millions b) ? 263,4PQN indice 100 77PresseMag en millions 1123 874PresseMag indice 100 a) ?a) Indice de la presse magazine en 2011 :8741123× 100 ≈ 77,8 .b) Nombre d’exemplaires de presse quotidienne payés(PQN) en 2006 : 263,4× 100 ≈ 342,177soit 342,1 millions de journaux.C’est un tableau de proportionnalité .ex75 1. Réponse b. Formule entrée en B3 = A3 – A$2 .2. 1975 = 1968 + 7 est l’année de rang 7.2009 = 1968 + 41 est l’année de rang 41 .3.a) 10,4139,738≈ 1,069316 . De 1999 à 2009, la population del’unité urbaine de Paris a augmenté de 6,932 % à 0,001près.b) On retrouve ce taux à l’aide du tableur.ex71 QCM1. Réponse a. 0,2 × 0,6 = 0,12 .2. Réponse b.11− 16,18100≈ 1,1930 .3. Réponse c. 12320,8 = 1540 .ex72 1. a) Si T = 25 , alors C = 0,8 et R = -20 .b) Si T = -10 , alors C ≈ 1,1111 et R ≈ 11,11 .2. a) Taux de 80 % : T = 80.b) Taux réciproque : environ - 44,44 % .Si la cellule D7 est au format pourcentage, on saisit laformule suivante = C7/C6 - 1 .4. a) 10,413≈ 1,2142 . Entre 1968 et 2009, cette population a8,576augmenté de 21,42 % à 0,01 % près.b) De 1968 à 2009 il y a 41 années d’évolution.Livre du professeur - Mathématiques Term STMG © Hachette Livre 2013 36


Le coefficient multiplicateur du taux d’évolution annuelmoyen est : CM = 1,2142 1 41 ≈ 1,0047 et :( 1,0047 – 1 ) × 100 = 0,47.De 1968 à 2009, en moyenne, la population de l’unitéurbaine de Paris a augmenté de 0,47 % chaque année.ex76 1. a) Variation absolue : 151,1 – 90 = 61,1 litres.Variation relative : 61,1≈ 0,6788 , soit une augmentation de9067,9 % à 0,1 % près.b) Coefficient multiplicateur sur cette période de 18 ans :CM global = 151,1. 90D’où le coefficient multiplicateur correspondant au tauxmoyen annuel d’augmentation de la consommation :CM =c) 42,5×11310054,9b)42,51est :151,1 18≈ 1,02920 .90122,7 18≈ 0,96305 .44,744,6×100 ≈ 105 .42,554,9× 100 ≈ 129 . 42,5Ainsi entre 1990 et 2008, la consommation d’eauxminérales et de source a augmenté en moyenne de 2,9 %par an.c) Si ce taux de croissance annuel est resté constant, de2008 à 2012, il y a 4 augmentations successives de 2,9 %.On en déduit la consommation d’eaux minérales et desource en 2012 estimée à :151,1 × 1,029 4 ≈ 169,4 litres par habitant.2. Consommation de vins courants de 1990 à 2008 .a) Variation absolue : – 22 litres.Variation relative : – 49,2 % .b) CM global = 22,744,7 . D’où CM 1 an =0,96305 × 100 – 100 = – 3,695 .Ainsi, entre 1990 et 2008, la consommation de vinscourants a diminué en moyenne de 3,7 % par an.c) 22,7 × 0,963 4 ≈ 19,5 litres.Si cette évolution moyenne se maintient, la consommationde vins courants en 2012 est estimée à 19,5 litres parhabitant.ex77 1. Taux d’usure annuel de 20,25 % et taux mensueléquivalent :1 + t100 = (1,2025) 1/12 ≈ 1,0155 , soit 1,55 % par mois.2. Taux annuel de découvert de 19,24 % et taux journalieréquivalent :1 + t100 = (1,1924) 1/360 ≈ 1,000489 , soit 0,0489 % .Remarque : en réalité, le taux appliqué pour calculer lesagios est un taux proportionnel 19,24365 ≈ 0,0527 .ex78 1. a) Formule entrée en C3 = B3/$B$3*100b) Indice pour l’année 2006 :Indice pour l’année 2011 :≈ 48 .En 2007, la fréquentation des TGV est de 48 milliards devoyageurs-kilomètre.2. a) CM global = 54,942,5 ≈ 1,2918 .Entre 2005 et 2011, la fréquentation des TGV a augmentéde 29, 18 % à 0,01 % près.16 ≈ 1,0436 . Taux d’évolution moyen annuel de lafréquentation entre 2005 et 2011 : 4,36 % .3. a) CM global = 812,7800,8 ≈ 1,0149 .Entre 2005 et 2011, les déplacements en voitureparticulière ont augmenté de 1,49 % .b) Le coefficient multiplicateur de l’évolution réciproque1CM’ = ≈ ≈ 0,9853.CM global 1,0149( 0,9853 – 1 ) × 100 = – 1,47 .Si, à partir de 2011, les déplacements en voiture diminuentde 1,47 %, on retrouvera le même niveau qu’en 2005.ex79 1. Dans cet algorithme, la valeur de départ est stockéedans la variable D et la valeur d’arrivée en V . À la sortie, letaux T est en écriture décimale et non en pourcentage.2. On considère la situation en France.variable D V A Bvaleur 165,8 216,6 2000 2011G = V – 1 = 216,6– 1 ≈ 0,3064D 165,8N = B – A = 2011 – 2000 = 11T = ( 1 + G ) 1 N – 1=216,6165,81111 – 1 ≈ 0,0246 .En France, sur la période 2000-2011, le taux annuel moyend’augmentation de la fréquentation des salles de cinémaest de 2,46 % .2. En Espagne : D = 135,3 ; V = 95,6 et N = 11 .P =95,6135,3111 – 1 ≈ – 0,0311 .Ainsi, en Espagne, sur la période 2000-2011, le taux moyenannuel d’évolution de la fréquentation des salles de cinémaest de – 3,11 % .ex80 Texte en françaisOn veut comparer deux populations de ville :• ville A : de 8 millions à 11 millions en 15 ans.• ville B : hausse de 2,8 % par an.1. a) Calculer le taux d’évolution global de la ville A en 15ans.b) Déterminer le taux annuel d’évolution de la populationde la ville A.2. Quelle est la ville qui a la croissance démographique laplus forte ?Expliquer.Livre du professeur - Mathématiques Term STMG © Hachette Livre 2013 37


ex81 QCM1. Réponse b. 0,77 2. Réponse b. – 5,79 %3. Réponse c. – 18,26 % 4. Réponse c. – 12,25 %ex82 1. Indice de référence des loyers au premier trimestre2009 :101,81 × ( 1 + 2,24) ≈ 104,09.1002.t100=100 − 98,5898,58≈ 0,0144 , soit une hausse des loyers de1,44 % entre le 1 er trimestre 2006 et le 1 er trimestre 2007 .3. Si le taux précédent s’est maintenu jusqu’en 2012,l’indice au 1 er trimestre 2012 devient :100 × ( 1 + 1,44100 ) 5 ≈ 107,41 .4. Du 1 er trimestre 2006 au 1 er trimestre 2012, l’indice desloyers est multiplié par CM global = 108,2298,58 .De 2006 à 2012, on compte 6 évolutions annuelles.108,2298,5816≈ 1,0157 et (1,0157 -1) × 100 = 1,57 .Ainsi, entre 2006 et 2011, l’indice de référence des loyers aaugmenté en moyenne de 1,57 % par an. On trouve unrésultat légèrement supérieur à celui trouvé à la question 2.ex83 QCM1. Réponse b. Le diviser par 0,8 .2. Réponse b. 500 % .3. Réponse c. 0,68 .4. Réponse a. 126,3 .ex84 Partie A QCM1. Réponse d. L’année 2010 est l’année base 100.En 2012, l’indice passe de 100 à 103,9 ; cela correspond àune hausse de 3,9 % du loyer entre 2010 et 2012.2. Réponse b. CM = 656,71642,25 ≈ 1,02251De 2008 à 2009, le loyer a augmenté de 2,25 % (à 0,01près).3. Réponse c. Formule =( C$3 – B$3) * 100 / B$3On doit « bloquer » le n° de ligne, d’où $3 .Donc 1 an c’est un quart de 4 ans.Soit t le taux annuel moyen d’évolution du loyer (enécriture décimale). On résout l’équation :(1 + t) 4 = CM 1 + t =CM 1 4 t = CM 1 4 – 1 ≈ 0,0155 .Soit, en pourcentage, un taux annuel moyend’augmentation de 1,55 % .Pour arrondir convenablement, prendre l’habituded’effectuer à la calculatrice le dernier calcul seulement :2ex85 1. a) 1500 × ( 1 – ) = 1 470 € .1001 470 × ( 1 – 0,9) = 1 456,77 € .100b) Du 1/01/2003 au 1/01/2005, la valeur de la marchandiseest multipliée par CM global = ( 1 + 41) ( 1 + 13,4) ≈ 1,599 .100 100Au cours de ces deux années, la valeur de la marchandise aaugmenté de 59,9 % .c) Le coefficient multiplicateur de l’évolution annuellemoyenne entre le 1/01/2003 et le 1/01/2005 est :CM = 1,599 1 2 ≈ 1,265 .D’où le taux annuel moyen d’évolution de la valeur de lamarchandise entre le 01/01/2003 et le 01/01/2005 égal à26 ,5 % .2. a) Tableau des résultatsannée 2007 2008 2009indice 100 109,8 109,8 × 1,085 = 119,133b) Entre le 01/01/2007 et le 01/01/2009, la valeur de lamarchandise a augmenté de 19,1 % , à 0,1 % près.ex86 1. a) De 2005 à 2010, le montant du SMIC augmentede 13,3 % . D’où la valeur du SMIC en 2010 :1186,03 × ( 1 + 13,3100 ) ≈ 1343,77 € .b) Entre les années 2005 et 2011, le SMIC a augmenté de15,3 % .c) 1,153 1 6 ≈ 1,0240 .On obtient une hausse annuelle moyenne du SMIC égale à2,4 % entre les années 2005 et 2011 .2. a) La formule à saisir en C3 est = B3/$B$2*100b) Indice du SMIC en 2006 :1236 ,081186 ,03× 100 ≈ 104,2 .Partie B Calculs1. Indice : 667,23× 100 ≈ 101,5 .657,372. De 2008 à 2012, le loyer est multiplié par :CM = 683,01≈ 1,0635 .642,25De 2008 à 2012, on a 4 augmentations successives.Livre du professeur - Mathématiques Term STMG © Hachette Livre 2013 38


ex87 Partie A1. t= 78,2−75,2100 75,2≈ 0,0399 .Entre 2000 et 2011, l’espérance de vie à la naissanceaugmente de 3,99 % pour les garçons.2.78,275,2111≈ 1,0036 .(1,0036 – 1 ) × 100 = 0,36 .D’où le taux d’évolution annuel moyen de l’espérance devie à la naissance entre 2000 et 2011 égal à 0,36 % .Partie B1. 0,286 × 12 + 75,2 = 78,632 .Si ce modèle se poursuit, un garçon né en 2012 peutespérer vivre 78,6 ans.2. En utilisant le taux moyen d’évolution vu dans la partie A,on calcule : 78,2 × ( 1 + 0,36100 ) ≈ 78,5.L’espérance de vie en 2012 est 78,5 ans.3. Le résultat est sensiblement le même selon les deuxmodèles.ex88 1. Àsaisir en C3 = C2 + 1 ou = 1 + (B3 – B$2) / 42. a) 4011138 ≈ 29 .Des jeux de Rome en 1960, à ceux de Pékin en 2008, laparticipation a été multipliée par 29.b) (29 – 1) × 100 = 2800 .La participation a augmenté de 2 800 % .c) À partir de 1960 et jusqu’en 2008, il y a 12 olympiades.29 1 12 ≈ 1,3239 .On en déduit que le taux d’augmentation moyen departicipation aux jeux paralympiques de 1960 à 2008 est de32,4 % tous les 4 ans (à 0,1 % près) .d) 1 383 – 44 = 1 339 ; 2 628 – 94 = 2 534 .De 1960 à 2008, la variation absolue du nombre departicipants est de 1 339 pour les femmes et 2 534 pour leshommes.138344262894≈ 31,43 et ( 31,43 – 1 ) × 100 = 3 043 .≈ 27,96 et ( 27,96 – 1 ) × 100 = 2 696 .La participation des femmes a augmenté de 3 043 % et celledes hommes de 2 696 % .3. a) Le nuage de points décrivant la participation deshommes ne se présente pas à l’aide de points presquealignés. On n’envisagera pas d’ajustement affine.b) L’année 2016 est l’année de rang 15.Si x = 15, alors y = 174,1 × 15 – 909 = 1 702,5.En utilisant cet ajustement affine, on peut prévoir 1 703participantes aux Paralympiades de 2016.Livre du professeur - Mathématiques Term STMG © Hachette Livre 2013 39


Intentions des auteuresLe but de ce chapitre est de mettre en place les arbresde probabilité et savoir les utiliser. Le programmeindique : « Un arbre pondéré correctement construitconstitue une preuve. »Dans le cadre d’un enseignement en spirale, cechapitre peut être traité en 3 parties.Cours 1 : très tôt dans l’année, même en tout premierchapitre avant les pourcentages d’évolution.Cours 2 et 3 : après un début sur les fonctions parexemple. On peut se réserver quelques problèmesd’approfondissement (ex 46 47 48, qui sont desextraits de Bac) juste avant le chapitre 7 ou pour larévision de fin d’année.• Penser à prendre du temps pour mettre en place levocabulaire et les notations : c’est une réelledifficulté pour les élèves. C’est pourquoi nous avonsmultiplié les exercices où l’on passe d’une notation àune phrase et inversement pour décrire unévénement ou une probabilité.• Nous avions choisi en première de noter p(A) laproportion d’une sous-population A : le passage de laproportion à la probabilité dans le cas d’une situationd’équiprobabilité se fait alors plus rapidement.Ainsi le cours 1 s’appuie sur les connaissances desélèves et reprend ce qui a été vu en seconde etpremière : pour le calcul de probabilité, on utilise dessituations sous forme de tableaux à deux entrées etde diagrammes de Venn (les « patates ») enexercices. On a insisté sur la probabilité del’intersection, pour pouvoir la distinguer ensuite de laprobabilité conditionnelle :P( A ⋂ B ) n’est pas P(A) × P(B) .On rappelle que la notion d’indépendance n’a étéabordée que dans le cadre de la loi binomiale enpremière et n’est plus au programme de Terminale.• Nous avons choisi de ne pas parler de « formule desprobabilités totales », en accord avec le programme.Pour calculer la probabilité de B dans une partition,nous sommes restés à la somme des probabilités desintersections de l’événement B avec chaqueévénement de la partition, sans aller plus loin dans lesnotations. L’essentiel est la lecture de l’arbre pondéré.Pour certains élèves, le tableau à deux entrées estl’outil le plus facile à utiliser. Pour ces élèves, il serabien de leur apprendre à revenir à ce tableau avecpour correspondance : le total est 1 = 100 % .On se garde de donner une probabilité enpourcentage en maths, mais la vie quotidienne nes’en prive pas. Le faire remarquer, sans pénaliser.Nous avons délibérément choisi nos exemples dans lequotidien, les sciences sociales ou la gestion, et nondans le monde des jeux de cartes ou le tirage dans desurnes. Notre expérience dans ces classes nous conduità éviter le « tirage de boules » en particulier. Nousn’avons d’ailleurs pas trouvé de sujet de Bac décrivantde telles situations, dans cette section.Avant de commencerCes 4 QCM permettent d’utiliser les différentesreprésentations que les élèves ont pu rencontrer enseconde et en première pour décrire des situationsconduisant aux probabilités.On retrouve dans ces QCM les premières questionsposées dans un exercice de probabilité. Le dernierQCM peut se poursuivre par un arbre de probabilité,déjà vu en seconde.QCM1 : Réponse b) Ce QCM permet de rappeler lafaçon de construire et de lire un diagramme de Venn(en patates) ex8 p. 72P(D) est faux, car dans D on a 65 + 260 personnes.P( M ⋃ D ) =260 + 65 + 505Livre du professeur - Mathématiques Term STMG © Hachette Livre 2013 401000= 0,830 .On peut aussi voir le contraire de M ⋃ D qui contientles 170 personnes restantes.Faire lire également P( M ⋂ D ) .QCM2 : Réponse b) Lecture d’un tableau à deuxentrées en termes de probabilité.


Une telle situation peut-être exploitée de façon plusapprofondie : on peut choisir de nommer A(Avion)T(Train), C(Car), S avec assurance et S sans assurance,et demander aux élèves de traduire symboliquementles probabilités :a) P(A ⋃ S) b) P( C ⋂ S ) et c) qui demande une autrenotation, car cela ne peut pas être P( A ⋂ S ) .Pour a) on lit 29 + 10 – 8 = 31 et non 39 .b) 0,6 est vrai, on parle de l’intersection de lacolonne « en car » et de la ligne « sans assurance ».c) la réponse est fausse : 8/10 = 0,8 .Conseil : on peut demander de déterminer d’autresprobabilités d’intersection.QCM3 : Réponse b) QCM qui reprend le diagrammeen boîte (à moustaches) vu en première et enexercices dans le ch1 (p.27).On peut demander une lecture au Bac, dans un QCMou le début d’un sujet de probabilité.a) Faux : confusion entre « au moins » et « au plus ».Savoir distinguer « au moins », « au plus », « moinsde » et « plus de » sera très utile dans les chapitres 5et 7 sur les lois de probabilité.b) Vrai : P( salaire [ Q1 ; Me ] ) = 0,25 .Comme l’entreprise est très grande, on peut enprofiter pour faire comprendre que le salaire est unevariable continue de 900 € à 2200 € et qu’aucunsalarié n’a exactement 1100 € . Être « inférieur » ou« inférieur ou égal » revient au même en terme deprobabilité dans le cas d’une variable continue.c) Faux : la réponse est 0,25 .La répartition d’un ensemble en quatre groupes demême effectif est importante.Le diagramme en boîte se réalise très facilement àl’aide du logiciel Sinequanon (logiciel français gratuit)Il suffit de rentrer les paramètres :QCM4 : Réponse c) On revient sur l’inclusion de souspopulationsvue en première.a) Faux : P(F ⋂ C ) = 0,1b) Faux : l’événement cité est C ⋃ H.c) Vrai : car les deux autres sont faux. On trouve lasolution à l’aide du tableau à deux entrées ci-dessous.cadre : C non cadre totalfemme : F 10 % 30 % 40 %homme : H 10 % 50 % 60 %total 20 % 80 % 100 %CoursÉvénement et probabilitésÉtude d’une situationLa lecture d’un tableau à deux entrées est l’approchela plus simple pour les élèves : tout peut se vérifier.Nous avons choisi de noter en couleur les effectifsutiles pour les questions 2et 3.On rappelle qu’un salarié est une personne travaillantdans une entreprise, quel que soit son poste. À ne pasconfondre avec un employé, catégoriesocioprofessionnelle particulière (secrétaires, agentsde bureau, mais aussi agents hospitaliers, vendeurs,pompiers ou gens de maison… ) Voir INSEE :http://www.insee.fr/fr/methodes/default.asp?page=definitions/emploi-salarie.htma) Avec les effectifs en noir, on peut retrouver tous leseffectifs.b) 78 salariés ont un salaire de 1800 € et plus et sontemployés.B : employéA: 1800 €et plus150= 141150 = 0,94 .3.a) A et S sont des événements contraires : à euxdeux ils constituent la totalité des possibles.Donc P(A) + P(S) = 1 . Ainsi :P(S) = 1 – P(A) = 1 – 0,7 = 0,3 .b) P( A ⋂ C ) = 15= 0, 1 . Il y a 15 salariés à la fois15078= 114 – 36S: moinsde 1800 €45 – 6 – 3= 36cadre et de salaire 1 800 € et plus.Donc P( A ⋃ C ) = P(A) + P(C) – P(A⋂C) = 0,8 .total114=150– 21 – 15C : cadre 21 – 6 = 15 6 21I :Intérimaire15 – 3 = 12 3 15total105= 150 – 4545 1502. P(A) = 105= 0,7150114P(B) = 150P(E) = P( A ⋂ B ) = 78150P(F) = 114+105−78Livre du professeur - Mathématiques Term STMG © Hachette Livre 2013 41


Faire comprendre que, en ajoutant les salariés cadreset les salariés ayant un salaire de 1 800 € et plus, on acompté deux fois ceux ayant les deux critères.D’où la soustraction.Vérifier en effectuant 78+12+15+6.150Terminer en demandant P(A ⋂ B) et P( A ⋃ B ).Faire le lien entre o⋃ , ⋃nion et le symbole ⋃et entre eT , iNTer et le symbole ⋂ p.218Objectifs des exercices : reprendre les connaissancesvues en seconde a priori. Il ne faut pas hésiter à bienreprendre ces bases (d’où leur place ici et non entechniques de base). La lecture de tableau à deuxentrées doit être maitrisée.On rencontre de nombreux tableaux de ce type dansle T.E.F. (Tableau de l’Économie Française) dès quel’on donne des répartitions. Souvent le 100 %n’apparaît pas dans ces tableaux. On peut téléchargerle T.E.F. gratuitement chaque année.http://www.insee.fr/fr/ffc/tef/tef2013/tef2013.pdfTee-shirt pas de Tee-shirtJean 20 % 50 %pas de Jean 80 % 50 %100 % 100 %4.a) Question 1.b)b) Arbre pondéré complet.Probabilité conditionnelle P A ( B ) et arbreÉtude d’une situationLes élèves ont peut-être réalisé des arbres de choixdans les classes antérieures, non pondérés.La difficulté est dans la lecture de l’arbre pondéré etsurtout faire la différence entre P( A ⋂ B ) et P A (B) .C’est pourquoi nous avons mis en rouge les mots :Parmi , On sait que , Sachant quequi indiquent le conditionnel.Ne pas hésiter à traduire les données dans un tableau,ceci est rassurant pour certains élèves qu’il ne fautpas «perdre». Nous conseillons de ne pas introduiretrop rapidement les notations et d’utiliser le pluslongtemps possible le support concret, d’où l’écrituredes quotients à l’aide de mots.Ce chapitre peut réconcilier les élèves allergiques auxfonctions !Tee-shirt : A pas de Tee-shirtJean : B 24 40 64pas de Jean 96 40 136120 80 200Corriger le plus vite possible les élèves qui donnentdes probabilités en % , mais sans pénaliser.Nous avons pris le parti de présenter les arbres enallant du haut vers le bas et non de gauche à droite.Ce choix permet de faire penser à un tri que l’oneffectue pour mettre des fiches dans des cases.D’où le dessin d’une case pour indiquer A ⋂ B en basde la branche. On peut faire compléter en notant lesautres cases : cela conduit à la répartition del’ensemble des clients en quatre groupes.De plus, avec un tableau numérique, il est facile d’enprendre l’image et d’effectuer une rotation de 180° etdessiner le tronc de l’arbre !1.a) P(A) = 0,6 .b) Part des acheteurs de Jean dans les acheteurs deTee shirt : 24= 0,2 , à ne pas mettre en % .120Ne pas parler de probabilité ici.c) P( A ⋂ B ) = 24= 0,12 .200À bien différentier du calcul précédent.2. Probabilité demandée :clients achetant jean et pas de tee −shirt= 40= 0,5 .clients n′achetant pas de tee −shirt 80Nous conseillons de ne pas donner ici la notation :mieux vaut attendre d’avoir complété l’arbre.3. Le tableau est un tableau marginal.Ne pas demander à faire écrire l’opération théorique :P( A⋂B ) = P(A) × P A (B)trop lourde pour de nombreux élèves. Il est essentielqu’ils sachent utiliser l’arbre pondéré et qu’ils gardenten mémoire le principe multiplicatif vu en 1 ère sur leschéma de Bernoulli. Dans la partie « Ce qu’il fautretenir », le 0,4 sur la branche vers A est une coquille.Objectifs des exercices : Savoir passer d’un tableau àdeux entrées à un arbre pondéré, interpréter lanotation P A (B) et savoir différentier les deuxprobabilités P( A ⋂ B ) et P A (B) dans un contexte.Livre du professeur - Mathématiques Term STMG © Hachette Livre 2013 42


Probabilité totale dans une partitionÉtude d’une situationC’est la question classique demandée au Bac lesannées précédentes. C’est l’application du calcul de laprobabilité P( A ⋂ B ) . Nous n’avons pas voulu écrireet parler de la « formule des probabilités totales »,vocabulaire absent du programme. De même lanotion d’événements indépendants n’est plus auprogramme.1.a) 0,3 est la probabilité que la dragée soit blanche,sachant qu’elle est aux amandes : P A (B) = 0,3 car 30 %des dragées aux amandes sont blanches.b) P( B ⋂ A ) = P( A ) × P A (B) = 0,4 × 0,3 = 0,12 .Bien insister sur le produit des pondérationsrencontrées sur le chemin « du tronc » vers la brancheB finale.c) P S (B) = 0,8 car 80 % des dragées au sucre sontblanches.Le verbe « donner » indique qu’une interprétation dutexte suffit pour la réponse et aucun calcul n’est àfaire. Verbes p. 210D’où P( B ⋂ S ) = P( S ) × P S (B) = 0,1 × 0,8 = 0,08 .d) P C (B) = 0,6 car 60 % des dragées au chocolat sontblanches.D’où P( B ⋂ C ) = P( C ) × P C (B) = 0,5 × 0,6 = 0,3 .2. Comme une dragée blanche est soit aux amandes,soit au sucre, soit au chocolat, les événements A , S etC forment une partition et la probabilité de B est lasomme des probabilités des trois intersections :P( B ) = P( B ⋂ A ) + P( B ⋂ S ) + P( B ⋂ C )= 0,12 + 0,08 + 0,3 = 0,5 .Remarque : on peut en déduire que 50 % des dragéessont blanches. On peut faire remarquer la position descompléments dans la phrase expliquant uneprobabilité conditionnelle : l’événement conditionnel,écrit en indice, se retrouve avant le verbe encomplément du sujet et l’événement cherché est écritaprès le verbe, en complément d’objet :P A (B)= p, car p % de « A » sont « B » .100Pour certains élèves, plus habitués à lire des tableaux,on peut leur expliquer en formant le tableau suivant :amande sucre chocolat totalblanches 0,3×0,4 0,8×0,1 0,6×05 0,5part de blanches 12 % 8 % 30 % 50 %On peut aller plus loin en demandant la probabilitéconditionnelle sachant B réalisé :P B (A) = P( A ⋂ B )= 0,12= 0,24P( B ) 0,5ce qui permet de revenir à la définition de laprobabilité conditionnelle.Objectifs des exercices : réaliser un arbre pondérécomplet, déterminer une probabilité totale, manipulerles notations et passer du texte aux notations etréciproquement.Les premiers exercices proposés sont très guidés etdans l’esprit de ce qui est demandé en STMG.On trouvera de nombreux exercices dans les annalesdes années antérieures. Cependant, l’épreuve au Bacétant de 3 h (et non 2 h en CRGH), les questionspeuvent être plus nombreuses et plus ouvertes.Atelier TICEÉlection régionaleBut : À partir d’un tableau à deux entrées, dresserdeux tableaux marginaux (inutile de donner cevocabulaire) indiquant le conditionnement.On parle de tableau de répartition selon le vote pourun candidat ou selon l’âge du votant.Partie 1a) En B7 : part des personnes interrogées ayantl’intention de voter pour le candidat X .En E7, on obtient la formule = E6/$F$6 .b) En G3 on saisit = F3/$F$6 pour obtenir la part despersonnes interrogées ayant un âge dans [ 18 ; 30 [ .Partie 21.a) Il y a 100 personnes interrogées ayant moins de30 ans parmi les 300 personnes désirant voter Xavier :P X (J) = 100= 1 ≈ 0,3333 .300 3b) En B10, on saisit la formule = B3/B$6 ,qui devient = B5/B$6 en B12 .c) Comme seule le n° de ligne est « bloquée » par $, laformule saisie en B10 devient = E3/E$6 par copie versla gauche, puis = E5/E$6 par copie vers le bas en E12 .P Y (V) = 300= 0,75 . En C2, on obtient = C5/C$6 .400d) Tableau de valeurs2. En I10, on saisit la formule = B3/$F3 ,qui devient = E5/$F5 en cellule L12 .On peut faire faire ensuite les arbres pondéréscorrespondants.Livre du professeur - Mathématiques Term STMG © Hachette Livre 2013 43


Ancienneté des actifsBut : Traduire le principe multiplicatif appliqué sur la«totalité» de l’arbre en termes d’algorithme, avec uneboucle itérative. On peut aussi commencer à lire letableau pour savoir comment trouver les 100 % .Interpréter aussi le nombre 75,90 % en bleu en celluleE3 , puis 40,2 % en cellule C5 .Ce type de lecture peut être demandé au Bac dans lesmatières techniques.1. 46,8 % des actifs du secteur tertiaire (soit 75,90 %des actifs) ont de 1 à moins de 10 ans d’ancienneté.Vérifier que B4+B5+B6 donne 100 % .2. a) Il y a 4 secteurs, donc N = 4 .b) P(A) = 0,0292 et P A (B) = 0,077 (en cellule B4 ) .c) P( A ) × P A (B) = P( A ⋂ B ) = 0,0292 × 0,077= 0,0022484 ≈ 0,00225 avec 3 chiffres significatifs.On calcule par ce produit la probabilité qu’un actif prisau hasard soit dans le secteur Agriculture et a moinsd’un an d’ancienneté.d) En B9 on saisit la formule = B3 * B4qui devient = E3 * E4 en cellule E9e) En F9 on saisit la formule = SOMME( B9 : E9 )et on obtient 0,106 avec 3 chiffres significatifs.On retrouve le résultat annoncé P(B) = 0,106 .Traduire en un programme n’a pas d’intérêt.Actifs de 10 ans ou plus d’anciennetéBut : Compléter le tableau par les lignes 10 et 11 avecle même type de calculs qu’en ligne 9 .1. On obtient :P(D) = P( D⋂A) + P( D⋂I ) + P( D⋂C ) + P( D⋂T )≈ 0,433 .2. P(E) ≈ 0,461 .On obtient le tableau suivant de résultats.1. b. P( T ⋂ R ) = 180400 = 0,45 .QCM page 71 pour répondre à un QCM p. 1952. c. P( G ⋃ R ) = P(G) + P(R) – P( G ⋂ R ) = 160+ 130– 70400 400 400On peut aussi calculer 90+70+60400= 220400 .= 0,55 .3. b. P A (S) = 1 – P A ( S ) = 1 – 0,2 = 0,8 .Car la somme des probabilités sur les branches issues d’unmême nœud, ici A, est égale à 1 .4. c. C’est la définition d’une probabilité conditionnelle :P M (S) = P( S ⋂ M ).P( M )5. b. On regarde la totalité de l’arbre :P(R) = P ( A ⋂ R ) + P( B ⋂ R ) + P( C ⋂ R )= 0,5 × 0,7 + 0,3 × 0,5 + 0,2 × 0,4 = 0,58 .6. a. P R (A) =P( A ⋂ R)P(R)= 0,5×0,70,58≈ 0,6 .Corrigés des exercices du chapitre 3ex1 a) P(G) = 423+195= 0,412 .1500b) P(H) = 975= 0,65 .1500c) P( G ⋂ H ) = 420= 0,28 .1500P( G ⋃ H ) = P(G) + P(H) – P( G ⋂ H ) = 0,782 .ex2 a) P(K) = 1951500 + 9751500 – 1471500 = 10231500 = 0,682 .b) P(L) = 1 – 1951500 = 0,87 .ex3 P A (B) =et P B (A) =P( A ⋂ B)P(A)P( A ⋂ B)P(B)= 0,1= 0,40,25= 0,1= 0,5 .0,2ex4 Arbre de probabilitécomplété ci-contre.Livre du professeur - Mathématiques Term STMG © Hachette Livre 2013 44


ex11 1. P( E ⋃ F ) = P(E) + P(F) – P( E ⋂ F )P A (B) = 0,7= 0,65 + 1 – 17P A (B) = 0,23 60 P A (B) = 0,3 .2. P( G ⋃ H ) = P(G) + P(H) – P( G ⋂ H ) = 0,44 .ex5 a) P(C) = P( C ⋂ A ) + P( C ⋂ A ) = 0,08 + 0,18 = 0,26 .b) P(D) = P( D ⋂ A ) + P( D ⋂ A ) = 0,12 + 0,33 = 0,45 .c) Les événements C et M sont disjoints.P( C ⋃ M ) = P(C) + P(M) = 0,26 + 0,29 = 0,55.ex12 P( R ⋃ S ) = P(R) + P(S) – P( R ⋂ S ) .D’où P( R ⋂ S ) = P(R) + P(S) – P( R ⋃ S )= 0,76 + 0,43 – 0,55 = 0,64 .Ou bien P( C ⋃ M ) = 1 – P(D) = 0,55 .ex6 a) P M (A) = P( A ⋂ M )= 0,2ex13 1. a) En cellule B2, on place la probabilité que l’élèveP(M) 0,29b) P M (A) = P( A ⋂ M )= 0,15×0,6de terminale STMG du lycée soit une fille majeure, c’est-àdire0,15 .≈ 0,31 .P(M) 0,29On obtient la probabilité que l’article choisi ne soit pas de lamarque Armor Lux, sachant que c’est une marinière.Ainsi dans ce magasin, 31 % des marinières ne sont pas deb) L’information concernant la part des élèves majeurs seplace en cellule D2 .c) Tableau des valeursla marque Armor Lux.c) P( M A) = P(M) + P(A) – P( M ⋂ A ) = 0,29 + 0,4 – 0,2= 0,49 .ex7 P(A) = 0,14 P(B) = 0,39 P(C) = 0,22 et P(D) = 0,75 .2. a) P( F ⋃ M ) = P(F) + P(M) – P( F ⋂ M )ex8 Bien lire un diagramme de Venn (dit «à patates») := 0,6 + 0,3 – 0,15 = 0,75 .les nombres notés concernent la partie seule, par exemple12 est l’effectif deOn obtient ce résultat , en F2 , en saisissant la formulel’intersection A et B et 20= B4 + D2 – B2 .est l’effectif de la partie Ab) P( F ∩ M) = 0,25 .privée des 12 qui sont aussiEn F3 , on peut saisir = C3 ou bien = 1 – F2 .dans B.a) A «l’étudiant n’est pas inscrit au concours A «.A ⋂ B «l’étudiant est inscrit au deux concours A et B « .ex14 QCM1. Réponse c. P(G) = 70224A ⋃ B «l’étudiant est inscrit à au moins l’un des concours A 2. Réponse a. P( A ⋂ S ) = 88224ou B « .3. Réponse a. P( A ⋃ G ) = P(A) + P(G) – P( A ⋂ G )b) Au total, il y a 100 étudiants := 20620 + 12 + 45 + 23 = 100 .224 224 224P(A) = 20+12= 0,32 ; P(B) = 45+124. Réponse a. = (E2 + B4 – B2) / $E$4= 0,55 ; et100100P( A ) = 1 – 0,32 = 0,68 . ex15 a) L’effectif de l’univers Ω est 100 .c) P( A ⋂ B ) = 1220+12+45= 0,12 et P( A ⋃ B ) = =0,77b) P(A) = 0,2 et P(B) = 0,38 .100 100c) P( A ⋂ B ) = 0,08 et P( A ⋂ B ) = 0,3 .ou bien P( A ⋃ B ) = P(A) + P(B) – P( A ⋂ B )= 0,32 + 0,55 – 0,12 = 0,77 . d) P A (B) = 820A (B) = 3080On a aussi P( A ⋃ B ) = 1 – 23100ex16 1. On utilise les effectifs indiqués, comme dans l’exo 8P(A) = 0,25 et P(B) = 0,4ex9 1. Faux. P( A ⋃ B ) = P(A) + P(B) – P( A ⋂ B ) .2. Vrai. P( A ) = 1 – P(A) = 1 – 5 = 2 .P( A ⋂ B ) = 0,06 et P( A ⋃ B ) = 0,59 .7 7 2. a) P A (B) est la probabilité que le lecteur ait lu les3. Faux. P( A ⋃ B ) = 0,15 + 0,6 – 0,1 = 0,65 .résultats de basketball sachant qu’il a lu les résultats4. Vrai. P( E ⋃ F ) = P(E) + P(F) – P( E ⋂ F ) = 0,63 .d’athlétisme. P A (B) = 625b) Pex10 1. a) Par lecture des effectifs du tableau :B (A) est la probabilité que le lecteur ait lu les résultatsP(A) = 180220= 0,45 et P(B) = d’athlétisme sachant qu’il a lu les résultats de basketball.400 400 P B (A) = 6b) H ⋂ B : « le salarié est un homme et travaille dans40l’équipe 2 » . P( H B ) = 90c) P B (A) est la probabilité que le lecteur ait lu les résultats400 d’athlétisme sachant qu’il n’a pas lu les résultats de2. F ⋃ B : « le salarié est une femme ou travaille dans19basketball. P B (A) =l’équipe 2 » . P( F ⋃ B ) = P(F) + P(B) – P( F ⋂ B )19+41 60= 150On peut lire ces trois résultats en utilisant les effectifs du400 400 400 400 diagramme.H ⋂ A : « Le salarié est un homme et ne travaille pas dansl’équipe 1» . P( H ⋂ A ) = 90ex17400P(A) = 0,1 P( A ) = 0,9Livre du professeur - Mathématiques Term STMG © Hachette Livre 2013 45


ex18 P( A ⋂ B ) = 0,1 × 0,7 = 0,07 .2. a) P S (F) = 56P( A ⋂ B ) = 0,1 × 0,3 = 0,03 et P( A ⋂ B ) = 0,9 × 0,2 = 0,18 .ex19 a) P( A ⋂ B ) = P A (B) × P(A) = 0,6 × 0,8 = 0,48 .b) P B (A) = P( A ⋂ B ) = 0,48P(B) 0,5 = 0,96 .c) P(A) × P(B) = 0,8 × 0,5 = 0,4 et P( A ⋂ B ) = 0,5 .Les deux résultats sont différents.ex20 a) P(A) = 0,1 et P(C) = 1 – 0,1 = 0,9 .b) P A (H) = 0,8 et sur la branche allant de A à F, on place laprobabilité P A (F) = 1 – 0,8 = 0,2 .c) Sur la branche allant de C à H, on lit la probabilité de Hsachant que l’événement C est réalisé, notée P C (H) = 0,3 .d) Arbre pondéré complet3. Arbre pondéré complet ci-après.P( A ⋂ B)ex21 P( A ⋂ F ) = 0,1 × 0,2 = 0,02 .ex27 1. P A (B) = = 0,3P(A) 0,6 = 0,5 .P( C ⋂ H ) = 0,9 × 0,3 = 0,27 .P(A ⋂ B)2. P A (B) = = 0,28P(A) 1−0,6 = 0,7 .ex22 QCMArbre pondéré complet1. c. P A (B) = 1 – 0,2 = 0,8 .2. b. P( A ⋂ B ) = 0,15 × 0,8 = 0,12 .3. c. P A (B) = 0,4 .ex23 1.a) P(F) = 40100 = 0,4 .b) Arbre pondéré ci-contre.2. Parmi les filles, la part des élèves ayant choisi l’optionmusique est P F (M) = 2340 = 0,575 .a le caractère A est :3. Arbre pondéré complet ci-dessous.P A (B) =ex24 1.a) P(M) = 50= 0,5 .ex30 a) Faux : car de P100 A (B) =b) P M (H) = P( H ⋂ M )= 0,27P(M) 0,5= 0,54 .2. P C (F) = P( F ⋂ c )= 0,17P(C) 0,5 = 0,34Arbre pondéré complet .ex25 1. a) P(E) = P( T ⋂ F ) = 105200 = 0,525 .b) P( S ⋂ F ) = 24200 = 0,12 . On en déduit :80 = 0,7 .b) P T (F) = 105120 = 0,875 .c) P F (T) = 105129 ≈ 0,81 .La probabilité que l’élève approuve le point du règlementintérieur, sachant qu’il est en terminale, est égale à 0,81 .ex 26 1. D’après l’énoncé, 35 % des pièces proviennent dufournisseur C, d’où P(C) = 0,35 .2. a) 5 % des pièces du fournisseur A ont un défaut, donc :P A (D) = 0,05 .b) P B (D) = 0,1 et P C (D) = 0,03 .ex28 P(A) = 0,8 et P( A ⋂ B ) = 0,2 .La probabilité qu’un individu ait le caractère B, sachant qu’ilP( A ∩ B)= 0,2P(A) 0,8 = 0,25 .ex29 QCM Réponse c.En effet, de P( A ⋂ B ) = 0,45 et P A (B) = 0,3 , on déduit :P(A) = P( A ⋂ B )= 0,45P A (B) 0,3 = 1,5 .Cette situation est impossible, car une probabilité n’estjamais supérieure à 1.P( A ⋂ B), on déduit :P(A)P(A) = P( A ⋂ B )= 0,3P A (B) 0,75 = 0,4 .On complète l’arbre de probabilité à l’aide de l’énoncé etdu résultat de a) .Livre du professeur - Mathématiques Term STMG © Hachette Livre 2013 46


) Faux , car P A (B) = 1 – 0,75 = 0,25 .c) Vrai , car P( A ⋂ B ) = 0,6 × 0,9 = 0,54 .d) Faux , 0,1 = P A (B) .ex31 Arbre pondéré completex35 1. a) P( F ⋂ A ) = 0,5 × 0,6 = 0,3 .P( F ⋂ B ) = 0,3 × 0,5 = 0,15 et P( F ⋂ C ) = 0,2 × 0,1 = 0,02 .b) P(F) = P( F ⋂ A ) + P( F ⋂ B ) + P( F ⋂ C )= 0,3 + 0,15 + 0,02 = 0,47 .2. P(H) = 1 – P(F) = 0,53 .Donc la probabilité de choisir un homme est 0,53 .ex36 a) Arbre pondéré completex32 Arbre pondéré completb) P( A ⋂ B ) = 0,6 × 0,8 = 0,48et P( A ⋂ B ) = 0,4 × 0,3 = 0,12 .c) On en déduit : P(B) = P( A ⋂ B ) + P( A ⋂ B ) = 0,6 .ex37 P(B) = P( A ⋂ B ) + P( A ⋂ B )= 0,4 × 0,25 + 0,6 × 0,9 = 0,64 .ex38 a) Arbre pondéré completex33 1. a) P(D) = 0,75 et P(E) = 0,25 .b) En B5 on saisit la formule = B4/$D$4En C5 on saisit la formule = C4/$D$4 .2. a) P D (A) = 150750 = 0,2 .b) Pour retrouver le résultat précédent, on place en celluleB7 la formule = B2/B$4 .3. P E (A) = 25 = 0,1 à saisir en C7 par = C2/C$4 .250Une boîte manuelle est une boîte non automatique.4. P D (A) = 600= 0,8 ou bien 1 – P D (A) .750P E (A) = 225= 0,9 ou bien 1 – P E (A) .250ex34 1. a) P( A ⋂ B ) = 0,6 × 0,8 = 0,48 et P( A ⋂ B ) = 0,12 .b) P(B) = P( A ⋂ B ) + P( A ⋂ B ) = 0,6 .2. a) P( A ⋂ E ) = 0,6 × 0,2 = 0,12 et P( A ⋂ E ) = 0,28 .b) P(E) = P( A ⋂ E ) + P( A ⋂ E ) = 0,4 .3. On a bien P(E) = 1 – P(B) .b) P( A ⋂ B ) = 1 × 0,36 = 0,12 .3c) P(B) = P( A ⋂ B ) + P( A ⋂ B )= 1 × 0,36 + 2 0,36+0,44× 0,22 =3 3 3d) P B (A) = P( A ⋂ B )= 0,12 × 15 = 0,45 .P(B)4ex39 1. P D (A) est la probabilitéque le téléviseur soit acceptésachant qu’il a un défaut.P D (A) = 1 – 0,96 = 0,04 .2. Arbre pondéré ci-contre.3. a) D A : « le téléviseur a undéfaut et est accepté » .P( D A ) = 0,04 × 0,04 = 0,0016 .P( D ∩ A ) = 0,96 × 0,98 = 0,9408 .b) P(A) = P( D A ) + P( D ∩ A ) = 0,9424 .ex40 1. Arbre pondéré complet= 0,83 = 415 .Livre du professeur - Mathématiques Term STMG © Hachette Livre 2013 47


2. P( A L ) = 0,2 × 0,08 = 0,016P( A M ) = 0,0525 et P( A S ) = 0,045 .b) On en déduit :P(A) = P( A ⋂ L ) + P( A ⋂ M ) + P( A ⋂ S ) = 0,1135 .c) D’où la probabilité qu’un inscrit ne soit pas absent :P(A) = 1– P(A) = 0,8865 et 0,8865 > 0,85 .Ainsi globalement, plus de 85 % des inscrits sont présents.ex41 1. Arbre pondéré complet2. a) E ⋂ C est l’événement :« l’ancien élève poursuit ses études et vit en colocation ».P( E ⋂ C ) = 0,65 × 0,4 = 0,26 .b) P(C) = P( E ⋂ C ) + P( T ⋂ C ) = 0,26 + 0,35 × 0,12 = 0,302 .3. P C (E) = P( E ⋂ C ) = 0,26P(C) 0,302 ≈ 0,861 .ex42 QCM Bonnes réponses1. Réponses b. et c.2. Réponses a. b. c.ex43 1. a) Arbre pondéré completb) P E (G) = 0,3 . P( G ⋂ S ) = 0,4 × 0,15 = 0,06 .P( G ⋂ E) = 0,6 × 0,3 = 0,18 .2. P(G) = P( G ⋂ S ) + P( G ⋂ E) = 0,06 + 0,18 = 0,24 .3. P G (S) = P( S ⋂ G )= 0,06P( G ) 0,24 = 0,25 .La probabilité que le client ait acheté le congélateur ausupermarché, sachant qu’il a acheté l’extension de garantieest 0,25 .On peut en déduire que, dans l’ensemble des clients, 25 %de ceux ayant acheté l’extension de garantie ont acheté lecongélateur au supermarché.ex44 1. a) P(C) = 0,75 et P(O) = 0,25 .b) P O (A) = 0,18 est la probabilité que le client ait acheté unjeu action/aventure sachant que le jeu est pour PC ou Mac.2. a) Arbre pondéré completb) P A (C) est la probabilité de l’événement : « le client aacheté un jeu pour console sachant que c’est un jeuaction/aventure ». On ne peut pas en lire directement lavaleur sur l’arbre pondéré ci-dessus.3. a) A ⋂ O est l’événement : « le client a acheté un jeuaction/aventure pour Mac ou PC ».b) A ⋂ C est l’événement : « le client a acheté un jeuaction/aventure pour une console de jeux ».c) P(A) = P( A ⋂ O ) + P( A ⋂ C )= 0,25 × 0,18 + 0,75 × 0,35 = 0,3075 .4. a) P A (C) = P( A ⋂ C ) .P(A)b) D’où P A (C) =0,75 × 0,350,3075≈ 0,854 .ex45 1. a) Soit I l’événement : « le ménage paie l’impôt surle revenu ».D’après les données du tableau , P( I ) = 0,441 ,donc P(I) = 1 – 0,441 = 0,559 .b) La probabilité que le ménage ait un revenu imposablepar part d’au plus 18 750 € est 0,441 + 0,124 = 0,565 .Remarque : Dans ce cas, la probabilité qu’il paie un impôtait 0,024. On peut faire une courbe de Lorenz de répartition(au programme de gestion).Une coquille s’est glissée dans le tableau :lire la tranche C de 18751 à 38750P( D ⋂ I )2. On cherche P I (D) = .P(I)Dans l’ensemble des ménages , on a D ⋂ I = D .On en déduit P I (D) = P(D)P(I) = 0,1430,559 ≈ 0,256 .ex46 Partie A QCMArbre pondéré complet1. c. P( C ⋂ A ) = 0,054 .2. a. P(A) = 0,45 .3. b. P A (C) .Partie B1. P(B) = 1 – P(A)= 1 – 0,45 = 0,55 .Au second tour, Béa estélue avec 55 % des voix.2. Si les adhérents qui ontvoté Albin au premier tour avaient tous voté Albin ausecond tour, après avoir remplacé la probabilité 0,99 par 1et 0,01 par 0 sur l’arbre précédent, on aurait :P(A) = P( A1 ⋂ A ) +P ( B1 ⋂ A ) + P( C ⋂ A )= 0,4 × 1 + 0,33 × 0 + 0,27 × 0,2 = 0,454 .Albin n’aurait pas été élu.ex47 1. a) P(C) = 1 – ( 0,4 + 0,25 ) = 0,35 .En C3 , on saisit la formule =D3 – (A3 + B3) .b) P C (R) = 1 – 0,6 = 0,4 . On place ce nombre en C5 .Livre du professeur - Mathématiques Term STMG © Hachette Livre 2013 48


c) Comme la billetterie B a vendu autant de billets pour leconcert de H que pour le concert de R , on place 0,5 en B4et en B5 .2. Arbre pondéré complet2. La proportion de filles est la plus importante enterminale avec une part de 60,7 % . Ensuite on a 60 % defilles en première et 54 % en seconde.3. a) P( C ⋂ R ) = 0,35 × 0,4 = 0,14 .b) En C6, ce résultat s’obtient par la formule = C3*C54. P(R) = P( A ⋂ R ) + P( B ⋂ R ) + P( C ⋂ R )= 0,4 × 0,25 + 0,25 × 0,5 + 0,35 × 0,4 = 0,365 .En D6, on saisit la formule = A3*A5+B3*B5+C3*C5ex49 Texte en françaisOn étudie l’emploi dans un groupe de personnes, âgées de20 à 64 ans, vivant au Royaume Uni.On interroge une personne au hasard dans ce groupe.On note les événements :M : « La personne est un homme »W : » La personne est une femme »A : « la personne a un emploi » .Les données publiées en 2011 dans Eurostat permettentd’établir l’arbre pondéré ci-dessous.ex48Partie 1 Tableau de répartitionSeconde Première Terminale totalfille 189 192 170 551garçon 161 128 110 399total 350 320 280 950Partie 21. a) P(A) = 350≈ 0,368 .950b) B ⋃ C est l’événement : « l’élève choisi est en premièreou en terminale ».P( B ⋃ C ) = 600≈ 0,632 .9502. A F : « l’élève choisi est une fille de seconde » .P( A ⋂ F ) = 189≈ 0,199 .9503. P A (F) = 189= 0,54 représente la probabilité que l’élève350choisi soit une fille sachant qu’il est en seconde.P F (A) = 189≈ 0,343 représente la probabilité que l’élève551choisi soit en seconde sachant que c’est une fille.Partie 31. Arbre pondéré completa) Donner la probabilité que la personne interrogée n’aitpas d’emploi, sachant que c’est un homme.b) Parmi les femmes, quelle est la part des actifs ?c) Calculer la probabilité que la personne interrogée soit unhomme avec un emploi.d) Calculer la probabilité P(A) .ex50 QCM1. Réponse b. 15,5 % des candidats ont échoué et :0,155 × 703 059 ≈ 108 974 ≈ 109 000 .P(A ∩ B)2. Réponse b. P(A) = = ( 1/5 ) / ( 1/2 ) = 1 P A (B)× 2 = 2 .5 53. Réponse c. On ne peut pas conclure car on ne sait pas siles événements C et D sont disjoints.4. Réponse a.employéscadresfemmes 27 8hommes 33 12La société compte 80 salariés dont 20 cadres, la probabilitéque le salarié choisi soit un homme sachant que c’est uncadre est 1220 = 3 5 .ex51 QCM1. Réponse c. P( A) = 1 – P(A) = 2 3 .2. Réponse a. P(B) = P( A ⋂ B ) + P( A ⋂ B ) = 1 18 + 1 9 = 3 18 = 1 6 .3. Réponse c. P A (B) = P( A ⋂ B ) =( 1/18 ) / ( 1/3 ) = 3 P(A)= 1 . 18 64. Réponse b. P( A ⋃ B ) = P(A) + P(B) – P( A ⋂ B )= 1 + 1 – 1 = 6+3−1= 8 = 4 .3 6 18 18 18 9Livre du professeur - Mathématiques Term STMG © Hachette Livre 2013 49


ex521. a) P(V) = 1 – P(V) = 1 – 60200 = 0,7 .b) Par lecture directe de l’énoncé, on lit P V (D) = 0,2 .2. Arbre pondéré completa) V ⋂ D : « le client choisit un vase vert avec un défaut » .b) P( V ⋂ D ) = 0,3 × 0,2 = 0,06 .c) P(D) = P( V ⋂ D ) + P( V ⋂ D ) = 0,06 + 0,7 × 0,1 = 0,13 .3. P D (V) = P( V ⋂ D ) .P( D )Or P( V ⋂ D ) = 0,3 × 0,8 = 0,24et P(D) = 1 – P(D) = 1 – 0,13 = 0,87 .Donc P D (V) = 0,240,87 ≈ 0,28 .ex53 QCM1. Réponse c. = C3/D42. Réponse a. P( C ⋂ F ) = 15160 ≈ 0,094 .3. Réponse c. P H (E) = 7693 ≈ 0,817 .4. Réponse a. = B3/D3ex54 QCMPour aider à la compréhension, on note les événements :I : « le message pris au hasard est indésirable »A : « le message pris au hasard est accepté »S : « le message pris au hasard est supprimé par le logicielStopoSpam » .On résume les données à l’aide de l’arbre pondéré cidessous.D’où le taux d’évolution mensuel moyen : ≈ 0,95 % .Partie 21. Réponse a. P( A ⋂ B ) = 0,1 × 0,7 = 0,072. Réponse c. P(B) = P( A ⋂ B ) + P( A ⋂ B )= 0,07 + 0,9 × 0,2 = 0,25 .3. Réponse a. P B (A) = P( A ⋂ B )= 0,07P(B) 0,25 = 0,28 .ex561. Parmi les clients qui partent en groupe, 55 % partent enFrance d’où P G ( E ) = 0,55 .On sait que 75 % des personnes seules partent à l’étrangerdonc P G (E) = 0,75 .2. Arbre pondéré complet3. P( G ⋂ E ) = 0,63 × 0,45 = 0,2835 .4. P(E) = P( G ⋂ E ) + P( G ⋂ E )= 0,2835 + 0,37 × 0,75 = 0,561 .5. P E (G) = P( G ⋂ E )= 0,2835≈ 0,505 .P(E) 0,561ex57 1. P(G) = 230577 ≈ 0,399 .2. a) P(D) = 314577 ≈ 0,544 .La formation de droite est composée de 314 députés dont46 femmes et 268 hommes. On en déduit :P D (H) = 268314 ≈ 0,854 .b) P G (F) = 64230 ≈ 0,278 .c) La formation des 33 députés du centre ne compte qu’uneseule femme.On en déduit P C (H) = 3233 ≈ 0,970 .Arbre pondéré complet1. Réponse d. P(A) = 1 – 0,76 = 0,24 .2. Réponse b. P( A ⋂ S ) = 0,24 × 0,03 = 0,0072 .3. Réponse a. P(S) = P( I ⋂ S ) + P( A ⋂ S )= 0,76 × 0,95 + 0,24 × 0,03 = 0,7292 .4. Réponse c. P S ( I ) = P( I ⋂ S ) 0,76 × 0,95= ≈ 0,99 .P(S) 0,7292ex55 Partie 11. Réponse b. Le coefficient multiplicateur global del’évolution du prix est CM global = 1,1 × 0,9 = 0,99 .Comme ce coefficient est inférieur à 1, on en déduit que leprix a finalement baissé.2. Réponse a. Le coefficient multiplicateur associé au tauxd’évolution mensuel moyen est :CM = 1 + 12100112 ≈ 1,0095 et ( CM – 1 ) × 100 ≈ 0,95 .4. a) D ⋂ F : « le député est une femme de la formation dedroite » .b) P( D ⋂ F ) = 0,544 × 0,146 ≈ 0,079 .5. La question revient à déterminer P F (D) = P( D ⋂ F ) .P(F)Livre du professeur - Mathématiques Term STMG © Hachette Livre 2013 50


D’après la question 4. b) , on connaît P( D ⋂ F ) .D’après l’énoncé, l’Assemblée nationale compte 46 femmesdans la formation de droite, 64 femmes dans la formationde gauche et 1 femme dans la formation de droite.46 + 64 + 1D’où P(F) = ≈ 0,192 .577On peut aussi utiliser les des probabilités totales à partir del’arbre pondéré précédent pour obtenir P(F) .D’où P F (D) = P( D ⋂ F )= = 0,079P(F)0,192 ≈ 0,411 . ex58 1. Arbre pondéré complet2. P( E ⋂ R ) = 0,1 × 0,8 = 0,08 .3. P(R) = P( E ⋂ R ) + P( E ⋂ R ) = 0,08 + 0,9 × 0,6 = 0,62 .4. P R (E) = P( E ⋂ R )= 0,08P(I) 0,62 ≈ 0,129 .Livre du professeur - Mathématiques Term STMG © Hachette Livre 2013 51


SIntentions des auteures et conseilsLe but de ce chapitre étant l’étude de fonctions dansle cadre de problème, nous avons réduit au minimumle côté théorique de la dérivation.La nouveauté de ce chapitre est l’étude des fonctionsrationnelles qui demande donc de connaître ladérivée de la fonction inverse et la dérivée d’unquotient. Le cas de la dérivée d’un l’inverse d’unefonction est vu en exercice.Dans le cadre d’un enseignement en spirale, cechapitre peut être traité en 3 parties :• Cours 1 et 2 , plutôt après les fonctions affines, aupremier trimestre : les exercices permettent de fairefonctionner l’équation du second degré et le signe dutrinôme (repris dans les techniques de base).• Cours 3 sur les tangentes, au second trimestre,avant ou après Noël, pour revenir sur les droites et leslectures graphiques• Cours 4 et 5 , vers Pâques, après quelques chapitressur les probabilités.Les capacités à mettre en place• Connaître la fonction dérivée de x ⟼ x n , la dérivéed’une somme, d’un produit par un nombre k et d’unefaçon plus générale, la dérivée d’une fonctionpolynôme .En première, les élèves ont vu la dérivée d’un fonctionpolynôme du second degré, son sens de variationétant en lien avec la parabole (vue en seconde), puisla tangente à une parabole et la dérivée d’unefonction polynôme du 3 e degré. Le nombre dérivé n’apas été introduit par le coefficient directeur de latangente (pas du programme) : la tangente est uneapplication du calcul de la dérivée.En première, la dérivée d’une somme et celle duproduit par une constante n’ont pas été vues.Ainsi « 4 x 3 » n’est pas traité comme 4 × x 3 ayant pourdérivée 4 × 3 x² , mais directement par 3 × 4 x² , moinshabituel dans les pratiques pédagogiques en lycée.Nous avons fait le choix de conserver cette approche(demandée par le programme), surtout dans lapremière partie du cours.Nous nous sommes permis parfois d’écrire : « ladérivée de x² est 2x » ou de parler de la dérivée f’ (x).Nous rappelons que la dérivée d’un produit de deuxfonctions est hors programme, et d’ailleurs de peud’utilité dans les applications que l’on rencontre dansles fonctions polynômes ou rationnelles, les fonctionsexponentielles et logarithmes n’étant plus auprogramme.• Connaître la fonction dérivée de x ⟼ 1/x , ladérivée d’un quotient et la dérivée d’une fonctionrationnelle.Nous avons choisi d’en faire la partie 4 du cours, pourque les élèves aient eu le temps de s’approprier ladérivée d’un polynôme, l’étude des fonctions• Savoir étudier les variations et extremums d’unefonction polynôme ou rationnelle, traité en deuxparties : cours 2 et cours 5. Bien que la méthoded’étude des variations reste la même, les difficultéstechniques sont importantes pour les élèves detertiaires sur les fonctions rationnelles. Les fractionssont loin d’être maitrisées.• Savoir déterminer une équation d’une tangente etsavoir la tracer placé en cours 3 pour revenir sur leslectures graphiques et quelques applications (rythmede croissance, lié à la vitesse)La grande majorité des exercices présente dessituations économiques, de gestion ou technologiquesen particulier dans la partie Pour approfondir. Ontrouvera un glossaire non exhaustif dans lestechniques de base p. 211 et 212.Une place est faite également pour les lecturesgraphiques (remédiation p. 213 214), car on lesrencontre souvent dans les QCM des sujets de Bac.Avant de commencerNous avons fait le choix de ne pas tester toutes lesconnaissances de 1 er sur la dérivation (sauf QCM 3).Il nous a semblé souhaitable de reprendre aumaximum cette notion dans le cours.Livre du professeur - Mathématiques STMG Term STMG © Hachette Livre 2013 52


En revanche, les thèmes abordés vont permettre dereprendre des techniques utiles : sens de variation ettrinôme. p. 215QCM1 : Réponse c) ici, x =3 et non 300 .a) Faux : la parabole représentant le bénéfice B doitêtre tournée vers le bas (ce qui est la situationclassique).b) Faux : B(7) = 0 , mais il faut faire attention auxunités : la quantité x est en centaines d’objets.QCM2 : Réponse a) le sens de variation est bien dansce sens, d’après QCM 1 a) et le sommet de la paraboleest en = . p. 216b) Faux, mais on peut demander le signe et réviserainsi les connaissances sur le trinôme.Faire visualiser la courbe sur la calculatrice dans lafenêtre X [ 0 ;10 ] , utiliser les valeurs du tableau devariations Y [ – 21 ; 4 ] ou le zoom Min–Max descalculatrices .Sur TI :et saisir f(x) en Y1Choisir la fenêtre par Xmin = 0 et Xmax = 10puis le Zoom minmaxSur Casio :et saisir f(x) en Y1Choisir la fenêtre V–Window [F3] :puis DRAW [F6]Xmin = 0 et Xmax = 10Zoom [F2] AUTO [F5]QCM5 : Réponse b) Retour sur les lectures graphiques p.213b) Faux : ne pas confondre signe de x et signe de f(x) .CoursDérivée de x ⟼ x n et d’une fonction polynômeÉtude d’une situationLa situation est théorique. Elle reprend les notionsvues en première pour les polynômes du 2 e et 3 edegré. Puis introduction de la dérivée d’une sommesur un exemple. La dérivée du produit par un nombreest simple.1. a) v’(x) = – 2 × 6x + 5 = – 12 x + 5b) 2x c) 3 x²On peut faire voir que dériver a x² c’est aussi dériverx² , puis multiplier par a ; on a 2 × a x = a × 2x .De même pour a x 3 , on a 3 × a x² = a × 3 x² .2. a) f(x) = 5 x 3 – 3 x² + x + 3b) On a f’ (x) = 15 x² – 6 x + 1 .3. a) u’(x) + v’(x) = 15 x² + 6x – 4 + ( – 12 x + 5 )= 15 x² – 6 x + 1b) On retrouve le même résultat.On peut utiliser un logiciel de calcul formel pour faired’autres essais sur la dérivée d’une somme.Par exemple sur Ti-Nspire dans une page d’éditeurmathématique ou sur XCas.Pour vérifier le calcul d’une dérivée, on peut utiliser lacalculatrice graphique.On saisit en Y1 la fonction f, en Y2 la dérivée calculéeà la main et en Y3 la fonction obtenue par nombredérivé point par point. On compare ensuite lestableaux de valeurs de Y2 et Y3 .Sur TI :en Y3 , saisir nbrDérivé( parY1 par Y–VARS 1:Fonction… 1:Y1QCM3 : Réponse c) que l’on peut trouver directementen traçant la courbe dans la fenêtre [ 0 ; 10 ] .a) Faux : erreur fréquente. Rappeler le calcul dubénéfice B(x) = R(x) – C(x) p. 211b) Faux : le calcul de dérivée est nécessaire pourdémontrer la bonne réponse.Les valeurs obtenues sont des valeurs approchées.Sur Casio : p GRAPH, saisir le nombre dérivé en Y3.i Calc [ F2 ] d/dx [F1] Y [F1] 1$ X [F5]TABL [ F6 ]QCM4 : Réponse a) Retour sur le trinôme du seconddegré : discriminant et racines. p.216Les racines sont – 5 et 3 .Livre du professeur - Mathématiques STMG Term STMG © Hachette Livre 2013 53


Objectifs des exercices : Appliquer les formules dedérivée et effectuer des calculs d’image en utilisant lacalculatrice (pour apprendre à entrer correctementune fonction). Dans les exercices contextualisés, nousavons insisté sur la précision et les unités, réelledifficulté pour les élèves.Sens de variation d’une fonction polynômeÉtude d’une situationNous avons choisi cette étude plutôt que l’étude denouveaux cas de maladie que l’on rencontre dansd’autres sections. Ce type de fonction est une étudede productivité en économie : on peut expliquer quecet investissement publicitaire est nécessaire, car lasociété est en perte de clientèle. Ainsi f(0) = – 5s’interprète comme une perte de 50 clients s’il n’y aaucun investissement. Pour inverser la tendance, il estnécessaire d’investir au moins 10 000 € en publicitépour que la société trouve de nouveaux clients. Enrevanche, trop de publicité sature l’idée desconsommateurs et le nombre de nouveaux clientsbaisse, sans qu’il y ait perte de clientèle ! On rejointalors l’idée de niveau de satisfaction.1.a) f’(x) = – 3 x² + 12 x .b) La factorisation par x est suffisante : mettre 3x enfacteur ajoute une difficulté inutile, car 3x = 0 nedonne pas toujours x = 0 chez les élèves ! p. 215f’ (x) = x (– 3 x + 12 ) nulle en 0 et en 4 .La parabole est tournéevers le bas.Signe :x 0 4 5,5f’(x) 0 02. Tableau de variationx 0 4 5,5f’(x) 0 027f(x)– 510,125Lecture du tableau de variation et lien entre signe dela dérivée et sens de variation. Lorsque la fonction estcroissante, sur [ 0 ; 4 ] , la dérivée est positive.Sur l’intervalle [ 4 ; 5,5 ] où la dérivée est négative,alors la fonction est décroissante.3. Le maximum est atteint en x = 4 , lorsque la dérivées’annule en changeant de signe. Vu dans l’exo résolupour la recherche d’un bénéfice maximal.Après x = 4 , soit 40 000 € d’investissement, le nombrede nouveaux clients diminue, mais n’est pas négatif.D’après les questions posées en 2. on peut indiquerque le théorème est réciproque, ce qui sera fort utiledans les lectures graphiques. L’exemple choisi pourillustrer ce théorème montre une fonction dont ladérivée s’annule en 2 , mais sans changer de signe : lafonction n’a donc pas d’extremum en 2.Objectifs des exercices : Reprendre le lien entre signede la dérivée et sens de variation, en particulier parlecture de graphiques (exos 34 à 38). Savoir résoudrel’inéquation f’(x) ≤ 0 demande une bonne maîtrise desconnaissances. Les exercices reprennent l’étude defonctions polynômes vues en première. Attention, uncoût total de fabrication est toujours croissant.Tangente à une courbeÉtude d’une situationNous avons repris la fonction précédente, au alentourdu point d’abscisse 2 . Ce point est d’ailleurs le pointd’inflexion de la courbe, pratiquement confondueavec la tangente en 2 de 1,5 à 25 .On rappelle que la notion d’approximation linéairen’est pas au programme de STMG.1. a) f’(x) = – 3 x² + 12 x .b) f’(2) = – 3 × 2² + 12 × 2 = –12 + 24 = 12 .2. Le point A a pour ordonnée f(2) = 11 .La droite a pour équation y = 12 ( x – 2 ) + f(2) y = 12 x – 24 + 11 y = 12 x – 13 .3. Les valeurs de g(x) sont les valeurs arrondies de f(x)à 0,1 près, pour x proche de 2 (de 1,7 à 2,3 ).On peut faire tracer la tangente sur la calculatrice rabats de couvertureSur TI : Dans l’écran graphique, dans la fenêtre :X [ 0 ; 5,5 ] et Y [ –10 ; 30 ]DESSIN 5:Tangente(et saisir l’abscisse 2 .La tangente se trace et l’équation réduite apparaît, lescoefficients lus doivent être arrondis.Pour effacer : DESSIN 1:EffDessin .Sur Casio : Dans l’écran graphique, dans la fenêtre :X [ 0 ; 5,5 ] et Y [ –10 ; 30 ] .Sketch [F4] Tang [F2] et saisir l’abscisse :2 . La tangente se trace.Pour effacer : Sketch [F4] Cls [F1] .Livre du professeur - Mathématiques STMG Term STMG © Hachette Livre 2013 54


Objectifs des exercices : Bien que le programme placela notion de tangente après l’étude des variations desfonctions, nous avons choisi de faire lire des nombresdérivés sur des courbes données a priori (exos 51 à57), car on trouve souvent ces lectures dans les QCMdonnés au Bac les années précédentes. Cela nous apermis de présenter la dérivée de la fonction inverse,par lecture de nombres dérivés, en traçant lestangentes à l’aide d’un logiciel. Mais ce n’est pas unecapacité attendue du programme.Dérivée de x ⟼ 1/x et d’une fonctionrationnelleÉtude d’une situationLa dérivée de la fonction inverse est introduite par deslectures graphiques de nombres dérivés pour utiliserle cours 3. La lecture est simple sur un logiciel (TiNspire ou GeoGebra) .1. La fonction inverse étant décroissante sur ] – ∞ ; 0 [et sur ] 0 ; + ∞ [ , la dérivée est toujours négative surchacun de ces intervalles.Rappeler que la fonction inverse n’est pas décroissan–te sur * .2. Par lecture graphiquex 1 1/2 4 – 1 – 1/2 – 2f(x) 1 2 1/4 – 1 – 2 – 1/2f’ (x) 1 – 4 – 1/16 – 1 – 4En C : = = . En F : =Si le livre est fermé, on peut aller plus loin en utilisantla calculatrice et le nombre dérivéexplications sur le cours 1La formule de dérivation d’un quotient de polynômesest admise. La formule que nous proposons où lesdérivées u’ et v’ s’écrivent en premier est le résultatde notre pratique en classes littéraires, ES et STG .Sa mémorisation « active », utilisant les mots « haut »et « bas » en alternance, a été prouvée sur unegénération d’élèves. Elle leur évite d’avoir à écrire :u(x)= … u’(x)= … et v(x) = … v’(x) = …avant d’appliquer la formule.Comme u’ et v’ sont en première place, et que ladérivée d’une fonction polynôme est « moinspuissante » que le polynôme, le développement dunumérateur de la dérivée s’en trouve simplifié et doncmoins d’erreurs de calcul, surtout lorsque u’(x) est unnombre négatif !!! Tous les exercices corrigés utilisentcette formule. On rappelle qu’une écriture comme :( x² – 2x + 4) ’ = 2x – 2 ne peut être acceptée et n’estpas utilisée depuis plus de 30 ans dans les ouvrages.Objectifs des exercices : Les exercices sont asseznombreux pour pratiquer les formules, y compris avecles variables q et t , contextualisés. La pratique ducalcul formel peut permettre aux élèves decomprendre que l’on dérive par rapport à unevariable. Cela sera utile pour les futurs BTS.Pour la dérivée de l’inverse d’une fonction, nousavons mis la formule en dehors de la synthèse.Nous avons évité dans les premiers exercices des casoù u’(x) ou v’(x) valent 1.Sens de variation une fonction rationnelleÉtude d’une situationCette partie de cours permet de faire la synthèse surl’étude d’une fonction d’une manière générale.Nous avons choisi d’étudier deux fonctions un peuplus compliquées dans le cours et dans l’exo résolu.Nous pensons qu’à ce moment de l’année (au 3 etrimestre) les élèves qui suivent ont besoin d’uneétude plus poussée. Des cas plus simples et très aidéssont proposés en exercices (de 84 à 89 p. 106 et 107).1. f’(x) = 1 + 25 × = 1 – .On peut demander si on peut étudier le signe d’unetelle somme directement.2. a) La réduction au même dénominateur n’est pasun acquis de seconde pour les élèves de STMG etrappelle de mauvais souvenirs. p. 217Peut-être proposer au départ la réduction au mêmedénominateur : 1 – = , sans calculer plus.f’(x) = = = .Pour les élèves qui ont du mal avec la réduction, onpeut expliquer que la forme est souvent donnée lejour du Bac pour permettre d’aller plus loin.Il est vain de chercher à faire factoriser la différencede deux carrés, ce n’est plus un objectif : les élèvessavent étudier le signe d’un trinôme qui leur sembleplus facile à appliquer (plus algorithmique).b) Le discriminant est ∆ = b² – 4 a c = 100 . D’où :x 1 = = = – 7 et x 2 = = 3 .c) Le dénominateur est un carré, toujours positif, ladérivée est du signe du numérateur.La phrase est courte : les élèves doivent connaîtrecette explication « par cœur ».3. Le signe de f’(x) est le signe de N(x) sur [ 0 ; 13 ] .On obtient l’allure de laparabole représentant N(x) .x 0 3 13f’ (x) 0 0f(x)9,511,754. Au bout de 3 heures, le nombre de bactériesaugmente, car la fonction devient croissante à partirde x = 3 .Livre du professeur - Mathématiques STMG Term STMG © Hachette Livre 2013 55


Pour le théorème d’existence d’un extremum nousavons choisi une phrase courte pour pouvoirl’appliquer comme dans l’exercice résolu.Objectifs des exercices : Il ne faut pas perdre de vueque l’étude de fonctions doit avoir un but, c’estpourquoi toutes les études proposées en exercicesont contextualisées. Nous avons essayé de donnerdes situations autres que des fonctions de coût ou debénéfice, que l’on trouvera dans les exercicesd’approfondissement ou les exercices de type Bac.On trouvera l’utilisation du tableur et l’algorithmiquedans la partie Pour approfondir.L’exercice type Bac est un grand classique, rencontréquatre fois dans les annales.Atelier TICEÉvolution d’un chiffre d’affairesBut : montrer aux élèves comment on peut modéliserdes données à l’aide d’une fonction à partir d’unnuage de points. Le premier modèle, affine, est vu auchapitre 1: il constitue une bonne révision.L’atelier est réalisé sous Excel. L’ajustement par unefonction polynôme n’est pas possible sous OpenOffice 3. Il peut l’être à l’aide des fonctionnalités de lacalculatrice.A. 1. Peut être fait sous OpenOffice.2. a) b) On obtient l’équation y = 19 x – 6 .c) Ajustement non judicieux, car pour 2008–2009 ladroite est en dessous des points du nuage et en 2011,elle passe très au dessus.3. a) b) On obtient y = 0,05 x² + 19,6 x – 7,3 .c) Cet ajustement est très proche de l’ajustementaffine : les deux courbes coïncident pratiquement.On peut demander aux élèves plus rapides de rentrerdans leur calculatrice les deux fonctions et deregarder les valeurs pour x variant de 1 à 11 (ou decréer un tableau de valeurs sur une feuille Excel).4. a) b) C(x) = –0,339 x 3 + 6,06 x² – 11,04 x + 29,71 .c) L’ajustement est très bon.2. a) f’(x) = –1/3 × 3 x² +12 x – 11 = – x² + 12 x – 11 .b) Signe de f’(x) , trinôme qui s’annule en 1 et 11 .Tableau de variation sur[ 0 ; 15 ] , les valeurs sontarrondies au millier d’euros.x 0 1 11 15f’ (x) 0 0f(x)30≈191≈25903. Ce modèle envisage qu’après 2011 le chiffred’affaires baisse.Graphique obtenu sous Excel :Nombre de clientsBut : utiliser un tableur pour obtenir des valeurs etcomprendre les 0 du tableau.On peut demander aux élèves de tracer le nuage depoints et de relier les points par une courbe lissée :Choisir l’onglet « Insertion » Graphique, puis :B Étude du modèleOn approche – 0,339 par – – et les coefficientssuivants par des entiers :6,06 ≈ 6 ; – 11,04 ≈ –11 et 29,71 ≈ 30 .1. a) La formule donnée est correcte :– 1/3 × X^3 c’est aussi – X^3/3b) On peut comparer en calculant la différence :B5 = B3 – B4 .L’écart est faible : 2 au maximum en 2006 .Livre du professeur - Mathématiques STMG Term STMG © Hachette Livre 2013 56


1. a) =–1*A2^4+12*A2^3–48*A2^2+80*A2Attention à la position du "moins" au début :=–A2^4+12*A2^3–48*A2^2+80*A2–A2^4 calcule (–A2)^4 et non – (A2)^4 .b) f(2) = 48 .2. a) La courbe passe par l’origine du repère.Le Drive n’a pas de client au départ.b) La dérivée s’annule en 2 et en 5 : comme il y achangement de signe de part et d’autre, la fonctionsemble admettre des extremums.3. a) f’ (x) = – 4 x 3 + 36 x² – 96 x + 80 .1. C’(x) = 3 x² – 18x + 35 et M(x) = x² – 9x + 35 + .2. Tableau de valeurs et graphique ci-après.3. Pour une quantité de 6 tonnes de pâte alimentaire,le coût marginal est égal au coût moyen. Le coûtunitaire est alors de 35 euros par kg .Cette pâte alimentaire peut-être une pâte depistache, base de pâtisserie.Conseil : comme tous les élèves ne sauront pasvérifier cette factorisation, ne pas insister pour ne pasperdre du temps sur le TD en salle informatique.La forme factorisée est obtenue à l’aide de XCas.Faire développer ( –4x + 20 ) ( x² – 4x + 4 ) .b) Utiliser la forme donnée par calcul formel :=–4*(A2–5)*(A2–2)^2Sinon, cela peut permettre de vérifier la dérivéecalculée.f’ (1) = 16 et f’ (3) = 8 .c) ( x – 2 ) ² est positif pour tout réel x et nul en x = 2 .Donc f’ (x) est du signe de –4 ( x – 5 ) .D’où le tableau de variation de la fonction sur [ 0 ; 6 ] .x 0 2 5 6f’ (x) 0 075f(x)480484. Après 6 mois, le nombre de clients abonnésdiminuant fortement et devient négatif après 6moiset demi. Cette tendance peut amener Norhane àlancer une campagne publicitaire.Le modèle n’est plus valable après x= 6 .Coût moyen - coût marginalNous invitons les collègues à étudier nos ouvragesDéclic 1 re ES et Term ES pour les fonctions de coûts.Dans l’ouvrage de 1 re STMG p. 82, on trouvera uneapproche du coût marginal, coût supplémentaireengendré par une production supplémentaire d’unepetite quantité, abordable pour des élèves de gestion.Le but de cet atelier est de retrouver une propriétédes fonctions de coûts : le coût marginal est égal aucoût moyen lorsque le coût moyen est minimal.Graphiquement cela correspond à la quantité pourlaquelle la tangente à la courbe de coût total passepar l’origine, car en ce point M de la courbe de coûttotal, le coefficient directeur de la tangente (coûtmarginal) est égal au coefficient directeur de la droite(OM), quotient du coût total par la quantité (coûtmoyen).Livre du professeur - Mathématiques STMG Term STMG © Hachette Livre 2013 57


QCM page 99 pour répondre à un QCM p. 1951. C’(x) = 0,8 × 3 x² – 25 × 2x + 300 = 2,4 x² – 50 x + 300 .Zoom 0:ZMinMax2. On a : B(x) = R(x) – C(x) p. 211On dérive cette somme :B’(x) = R’(x) – C’(x) = 110 – ( 2,4 x² – 50 x + 300 )= – 2,4 x² + 50 x – 190 .3. On calcule la dérivée f ’(x) = – 6 x² – 6x + 36 .On étudie le signe de f ’(x) . = b² – 4 a c = ( – 6)² – 4( – 6)(36) = 900 .Donc > 0 et = 30 .L’équation f ’(x) = 0 a deux solutions :x 1 = = = 2 et x 2 = = – 3 .L’allure de la parabole donne le signe de f ’(x) sur [ – 5 ; 5 ] .Rappel : le tracé de la parabole et sa position par rapport àl’axe des abscisses suffit pour expliquer le signe dupolynôme.Sur [ –3 ; 2 ], la dérivée est positive, donc la fonction f estcroissante sur [ –3 ; 2 ] . 2 nd degré p. 216 et signe p. 217Si on ne demande pas de justification, on peut visualiser lacourbe de la fonction f à la calculatrice, sur [ – 3 ; 2 ] , etconclure par lecture graphique.4. Le nombre dérivé f ’(a) est le coefficient directeur dela tangente à la courbe au point d’abscisse a, d’où :f ’(0) = 2 ; f ’(4) = 0 et f ’(6) = – 1 .5. Équation de la tangente au point E d’abscisse 3 :y = f ’(3) ( x – 3 ) + f(3)y = 0,5 ( x – 3 ) + 3,75 y = 0,5 x + 2,25 .6. b. f ’(x) = = = . rabats de couvertureCorrigés des exercices du chapitre 4ex1 a) f ’(x) = –12x + 72 . b) f(2) = 0 et f(10) = 0 .ex2 p. 87 a) f ’(x) = –3x² + 10 b) f ’(x) = 8x 3 – 15x² + 16x .ex3 a) f ’(x) = – 0,03x² – x . b) f ’(x) = 5x 4 –30 x² – 6x + 5 .ex4 f(x) = x 3 – 12 x 2 + 260 sur [ –2 ; 10 ] .f ’(x) = 3 x² – 24 x = 3x ( x – 8 ) .x –2 0 8 10f ’(x) + 0 – 0 +26060f(x)2044ex5 f(x) = x 3 – 15 x 2 + 85 x + 150 sur [ 0 ; 11 ] .f ’(x) = 3x² – 30x + 85 . = – 120 .La fonction dérivée ne s’annule pas et le polynôme f‘(x) eststrictement positif pour tout réel x.x 0 11f ’(x) +f(x)601150g(x) = – 0,1 x 3 + x 2 – 6 x + 100 sur [ 0 ; 11 ] .g’(x) = – 0,3x² + 2x – 6 . = – 3,2 .La fonction dérivée ne s’annule pas et le polynôme g’(x) eststrictement négatif pour tout réel x.x 0 11g’(x) –g(x)10021,97. c. g ‘(x) = – 1 – 9 × = – 1 + == .Comme ( x + 1 )² > 0 sur [ 0 ; 10 ] :g ‘(x) est du signe de – x² – 2x + 8 .On détermine les solutions del’équation – x² – 2x + 8 = 0 :on trouve 2 et – 4 .Sur [ 0 ; 10 ] la dérivée s’annule en 2 en changeant de signe(positif , puis négatif) donc la fonction g admet unmaximum en x = 2 .On peut aussi visualiser la courbe g sur [ 0 ; 10 ] à lacalculatrice en utilisant , par exemple sur Ti :ex6 On a f ’(x) = 3x² – 12x + 9 . Donc :f ’(0,5) = 3,75 et f ’(2,5) = –2 ,25 .À une ½ h, le rythme d’appel est de 37,5 appels à l’heure etau bout de 2,5 h les appels diminuent de 22,5 appels parl’heure.ex7 f(x) = x² – 4x + 5 . f ’(x) = 2x – 4 .• Soit A le point de la courbe d’abscisse 0 :f(0) = 5 et f‘(0) = – 4 .Équation de la tangente en A :y = f ’(0) ( x – 0) + f(0) y = – 4x + 5 .• Soit B le point de la courbe d’abscisse 2.f(2) = 1 et f ’(2) = 0 .Livre du professeur - Mathématiques STMG Term STMG © Hachette Livre 2013 58


Équation de la tangente en B : y = 1 .• Soit C le point de la courbe d’abscisse 3 :f(3) = 2 et f ’(3) = 2 .Équation de la tangente en C :y = f ’(3) ( x – 3) + f(3) y = 2 ( x – 3) +2 y = 2x – 4 .ex8 a) f ’(x) = 0,5 – b) f ’(x) = 2x – 3 +c) f ’(x) = –1 + .ex9 On a f (x) = , sur [ 1 ; 5 ] .f ’(x) = = = .ex10 f(x) = sur [ 1 ; 6 ] .f ’(x) = = .La fonction dérivée est positive sur [ 1 ; 6 ] et la fonctiond’offre f est donc croissante sur [1 ; 6 ] .g(x) = – 0,25x + 3 + sur [ 1 ; 6 ] . g’(x) = – 0,25 – .La fonction dérivée est négative sur [ 1 ; 6 ] et la fonctionde demande g est donc décroissante sur [1 ; 6 ] .ex11 QCM1. Réponse c. 2. Réponse b.ex12 1. Réponse c. f ’(x) = – 2x + 3 .Ne pas oublier que la dérivée de "– 5" est "0" .2. Réponse a. f ’(x) = 3 x² – x + 7 .ex13 a) f ’(x) = 2x + 4 . b) f ’(x) = – 9 x² + 0,8 .ex14 a) f ’(x) = x – 9,5 . b) f ’(x) = – 10 + 90x + 4 x 3 .ex15 a) f ’(x) = 25 x 4 + 2 b) f ’(x) =15 x 5 + 6 x .ex16 a) f ’(x) = 0,01x – 0,56 . b) f ’(x) = 7x 3 – 5x .ex17 a) f ’(x) = 3 x² + 2x + 1 . b) f ’(x) = 6 x² – 6x + 9 .ex18 a) f ’(x) = 2000 x .ex19 a) f ’(x) = – 3 x² + 15 x – 2 .b) f ’(x) = 0,15 x² – 0,4 x + 0,3 .b) f ’(x) = 16 x 3 + 15 x².ex20 a) f ’(x) = –10 x 4 .b) f ’(x) = 40 – ( 3 x² – 4x + 500 ) = – 3 x² + 4 x – 360 .ex21 a) f ’(x) = 2 x² + x . b) f ’(x) = 0,015 x² – 0,6 x + 0,25 .ex22 a) C ‘(q) = 2q + 5 . b) R ‘(q) = – 20 q + 2500 .ex23 a) B’(q) = – 2 q² + 26 q . b) f ’(t) = 3 t² – 8 t + 20 .ex24 a) B ‘(x) = – 1,6 x + 0,64 . b) C ‘(q) = 3 q² – 1,2 q + 1,5 .ex25 a) B ‘(q) = q² – 6q + 10 . b) f ’(x) = 12 x² –240 x + 900.ex26 = x 3 . D’où f ’(x) = × 3 x² +1 = x² + 1 .ex27 a) u’(x) = – x + 7 et v’(x) = 12 x² – 0,4 x + 3 .b) f ’(x) = u’(x) + v’(x) = 12 x² – 1,4 x + 10 .ex28 a) u’(x) = 0,03 x² + 3x – 0,2 et v’(x) = – x + 8b) On en déduit f ’(x) = u’(x) + v’(x) = 0,03 x² + 2x + 7,8 .ex29 a) f ’(t) = –3 t² + 48 t – 180 pour t [ 6 ; 10 ] .b) f(6) = 68 et f(10) = 100 .Le coût horaire est de 68 € pour 6 h d’utilisation de lamachine et de 100 € pour 10 h d’utilisation.ex30 a) C’(x) = 0,2 x + 0,5 .b) Tableau de valeursCoût de fabrication de 10 guirlandes : 1 260 € ;de 50 guirlandes : 1700 € .Bien faire attention aux unités : 10 guirlandes donne x = 1et C(1) = 12,6 donne 12,6 centaines d’euros.ex31 1. Tableau de variation de la fonction f :x –6 –3 2 4f ’(x) + 0 – 0 +f(–3)f(4)f(x) f(–6)f(2)2. a) Vrai : La courbe f traverse trois fois l’axe desabscisses.Les abscisses de ces trois points d’intersection sont lessolutions de l’équation f(x) = 0 .b) Vrai : La fonction est croissante sur [ – 6 ; – 3 ] , puisdécroissante sur [ –3 ; 2 ] : on en déduit que la fonction fatteint un maximum en – 3 .c) Vrai : Comme la fonction f est décroissante sur [ – 3 ; 2 ] ,alors la dérivée f ’(x) est négative sur [ –3 ; 2 ] .ex32 On a f (x) = x 3 – 3 x 2 – 9 x + 31. Réponse c. f ’(x) = 3x² – 6x – 9 .2. Réponse c. car la fonction change de variation3. Réponse c. f(1) = 1 – 3 – 9 + 3 = – 8 .ex33 QCM 1. Réponse c. 2. Réponse b.ex34 Tableau de variation completx –3 –2 0 1f ’(x) + 0 – 0 +44f(x)00Livre du professeur - Mathématiques STMG Term STMG © Hachette Livre 2013 59


ex35 Tableau de variation completx –2 0 2 4g’(x) – 0 + 0 –193g(x)–1–17ex36 a) Sur [ –2 ; 1 ] , la courbe Γ (gamma) de la dérivée f ‘est en dessous de l’axe des abscisses, on a donc f ’(x) 0,alors la fonction f est décroissante sur [ – 2 ; 1 ] .b) La dérivée f ‘ étant positive sur l’intervalle ] – ; – 2 ] , lafonction f est croissante sur ] – ; – 2 ] . signe d’une fonction p. 214ex37 • Comme la fonction f est croissante, sa dérivée estpositive. On en déduit que la courbe de la fonction dérivéede f est la courbe 2 .• La fonction g est décroissante sur ] – ; 1 ] et croissantesur [ 1 ; + [ . On en déduit que sa dérivée est négative sur] – ; 1 ] et positive sur ] 1 ; + [ .La droite 1 représente donc la fonction dérivée de g .ex38 1. La courbe de la fonction dérivée de f est 2 .La courbe de la fonction dérivée de g est 1 .2. Tableau de variation de la fonction fx 0 4f ’(x) – 0 + 0 –f(x)110Tableau de variation de la fonction gx –1 2g ‘(x) + 0 – 0 +g(x)– 4– 8Les ordonnées sont données avec la précision permise parle graphique.ex39 f(x) = 2x² – 8x + 9 sur [ 0 ; 5 ] . f ’(x) = 4x – 8 .4x – 8 = 0 x = 2 .Tableau de variationx 0 2 5f ’(x) – 0 +919f(x)1ex40 f(x) = – x² + 1,75 x + 7,5 sur [ – 3 ; 5 ] .f ’(x) = –2x + 1,75 . On résout –2x + 1,75 = 0 x = 0,875 .Tableau de variationx –3 0,875 5f ’(x) + 0 –≈8,27f(x)–6,75–8,75ex41 Soit f(x) = 0,02 x 3 + 0,4 x sur [ 0 ; 8 ] , alors :f ’(x) = 0,06 x² + 0,4 .La dérivée est toujours positive, comme somme de deuxnombres positifs. tableau de signe p.217x 0 8f ’(x) +f (x)13,440Conseil : calculer les valeurs aux bornes de l’ensemble dedéfinition dans un tableau de variation.ex42 f(x) = – 0,01 x 3 + 20 sur [ 0 ; 10 ] . f ’(x) = – 0,03 x² .Tableau de variationx 0 10f ’(x) –f(x)2010ex43 f(x) = x 3 + 5x + 4 sur . f ’(x) = 3 x² + 5 .Tableau de variationx – + f ’(x) +f(x)ex 44« Déterminer » : lecture graphique ne suffit pas p. 210Tous les points de l’axe des ordonnées ont une abscisse :x = 0 .Tous les points de l’axe des abscisses ont une ordonnée :y = 0 . p. 2141. a) f(0) = – 0² + 0 + 6 = 6Le point d’intersection de avec l’axe des ordonnées adonc pour coordonnées ( 0 ; f(0) ) = ( 0 ; 6 ) .b) On résout l’équation f(x) = 0 :– x² + x + 6 = 0 ; ∆ = 25 et = 5 .Les solutions sont 3 et – 2 . Donc les points d’intersectionde avec l’axe des abscisses sont ( 3 ; 0 ) et ( – 2 ; 0 ) .2. f ’(x) = – 2 x + 1 . On résout – 2 x+1= 0 x = 0,5x –3 0,5 5f ’(x) + 0 –f (x)6,25–6–14ex45 1. a) B(x) = – x² + 52 x – 480 sur [ 0 ; 50 ] .B(15) = 75 .b) Pour la vente de 15 bougies, le bénéfice est 750 € .2. a) On résout l’équation – x² + 52x – 480 = 0.∆ = b² – 4 a c = 784, les solutions sont 40 et 12 .D’où le signe de B(x) :x 0 12 40 50B(x) – 0 + 0 –b) Du tableau précédent, on déduit que Laetitia réalise unbénéfice si elle vend entre 12 et 40 bougies.3. a) B ‘(x) = –2x + 52 . On résout –2x + 52 = 0 x = 26 .Livre du professeur - Mathématiques STMG Term STMG © Hachette Livre 2013 60


Tableau de variationx 0 26 50B ‘(x) + 0 ––196 –B(x)4803804. Pour réaliser un bénéfice maximal, Laetitia doit vendre26 bougies et ce bénéfice maximal est alors de 1 960 €.ex46 1. a) f(x) = – x 3 + 7,5 x² –12x + 10 sur [ 0 ; 6 ] .f ’(x) = – 3 x² + 15x – 12 .b) On résout l’équation – 3 x² + 15x – 12 = 0 .∆ = b² – 4 a c = 15² – 4 ( – 3) (– 12) = 81 .∆ > 0 et = 9 . Les solutions sont :x 1 = = = 4 et x 2 = = = 1 .Tracer la parabole et placer 1 , 4 , 0 et 6 .c) D’où le tableau de signe de f ’(x) sur [ 0 ; 6 ] :x 0 1 4 6f ’(x) – 0 + 0 –2. Tableau de variation de la fonction f sur [ 0 ; 6 ] :x –2 1 4 6f ’(x) – 0 + 0 –1018f(x)4,5– 8ex47 1. On résout l’équation – 3 x² + 3 x + 6 = 0 .∆ = b² – 4 a c = 3² – 4 ( – 3) × 6 = 81 .Les solutions sont 2 et – 1 . Tracer la parabole.D’où le signe de A(x) = – 3 x² + 3x + 6 sur .x –1 2A(x) – 0 + 0 –2. f(x) = – x 3 + 1,5 x² + 6x + 5 sur [ 0 ; 5 ] .a) f ’(x) = – 3 x² + 3x + 6 .b) De la question 1. on déduit que l’équation f ’(x) = 0admet pour solution x = 2 dans l’intervalle [ 0 ; 5 ] et on endéduit le signe de f ’(x) sur cet intervalle.c) Tableau de variation de la fonction fx 0 2 5f ’(x) – 0 +f(x)155– 52,5ex48 1. a) On résout 3 x² – 12x + 40 = 0 . = b² – 4 a c = – 336 , négatif .b) L’équation n’a pas de solution et comme a = 3 > 0, onobtient l’allure de la parabole ; donc letrinôme est strictement positif pour toutréel x.2. a) f ’(x) = 3 x² – 12 x + 40 .b) D’après 1. b) 3x² – 12x + 40 > 0 sur , donc sur [ 0 ; 10 ] ,la dérivée est positive et la fonction f est croissante.x 0 10f ’(x) +f (x)9001003. a) Les coûts fixes sont de 100 euros.b) f(4) = 228 et f(10) = 900 .Le coût total de fabrication de 4 paires est 228 € et pour 10paires, il est de 900 €.c) Le coût total de fabrication est croissant.ex49 1. a) f(x) = x 3 – 12 x² + 36x sur [ 0 ; 8 ] .f ’(x) = 3 x² – 24 x + 36 .b) On résout l’équation 3 x² – 24 x + 36 = 0 .∆ = b² – 4 a c = 144 , les solutions sont 2 et 6 .c) Tracer la parabole et placer 2, 6, 0 et 8 .x 0 2 6 8f ’(x) + 0 – 0 +Sur [ 0 ; 2 ] et sur [ 6 ; 8 ] , la fonction dérivée est positive etpar conséquent la fonction f est croissante. Sur [ 2 ; 6 ] , lafonction dérivée est négative et la fonction f estdécroissante. On justifie ainsi le sens de variation observésur le graphique.2. a) Tableau de variation de la fonction f sur [ 0 ; 8 ]x 0 2 6 8f ’(x) + 0 – 0 +3232f(x)00b) Le minimum de la fonction f sur [ 0 ; 8 ] est 0 .On en déduit que pour tout réel x de [ 0 ; 8 ] , f(x) ≥ 0 .La courbe est située au dessus de l’axe des abscisses.ex50 1. a) f(x) = x 3 – 15 x² + 75x sur [ 0 ; 15 ] .f ’(x) = 3 x² – 30 x + 75 = 3 ( x² – 10 x + 25) = 3 ( x – 5 )² .On peut calculer ∆ = 0 .b) On en déduit que la fonction dérivée de la fonction f estpositive ou nulle sur [ 0 ; 15 ] . Elle est nulle pour x = 5 .2. Tableau de variationx 0 15f ’(x) +f(x)11250ex51 a) La tangente en A est horizontale, donc soncoefficient directeur est nul : f ’(2) = 0 .b) f ’(4) est le coefficient directeur de la tangente au pointC :f ’(4) = = .Et f(4) = 4 ; c’est l’ordonnée du point C .ex52 1. Sur l’intervalle ] – ; 1 ] , la fonction f estdécroissante et pour tout réel x ] – ; 1 ] , donc f ’(x) ≤ 0 .Sur l’intervalle [ 1 ; + [ , la fonction f est croissante etpour tout réel x [ 1 ; + [ , f ’(x) ≥ 0 .2. a) Au point d’abscisse 1 , la tangente à la courbe esthorizontale, son coefficient directeur est nul. On en déduitque f ’(1) = 0.b) f(3) = 0,5 et f ’(3) = 2 .c) f(– 1) = 0,5 et f ’(– 1) = – 2 .3. Équation de la tangente à en C ( 0 ; – 1 ) :y = f ’(0) ( x – 0) + f(0) y = – x – 1 .Livre du professeur - Mathématiques STMG Term STMG © Hachette Livre 2013 61


ex53 QCM1. Réponse c. 2. Réponse b. 3. Réponse c.ex54 QCM1. Réponse b. 2. Réponse a. 3. Réponse c.ex55 1. a. On lit les abscisses des points d’intersection de lacourbe avec l’axe des abscisses.2. c. L’ensemble solution est composé de deux intervalles :la courbe a deux parties au dessus de l’axe des abscisses.3. a. La courbe en dessous ou sur la droite horizontaled’équation y = 2 .ex56 1. Tableau de variation de la fonction fx – 5 0 6 8f ’(x) – 0 + 0 –3,12f(x)– 4– 42. a) S = { –2 ; 4 ; 7 } . b) S = [ –2 ; 4 ] ⋃ [ 7 ; 8 ] .3. a) f ’(2) = 1 ; c’est le coefficient directeur de la tangente àla courbe au point A d’abscisse 2 .b) Le coefficient directeur de la tangente à la courbe aupoint B d’abscisse – 4 est f ’(– 4) = .c) Équation de la tangente à en D ( 8 ; – 4 ) :y = f ’(8) × ( x – 8) + f(8) y = –7 ( x – 8 ) + ( – 4) y = – 7x + 52 .4. a) f ’(x) ≥ 0 x [ 0 ; 6 ] .Cette écriture signifie que l’on cherche toutes les valeurs dex telles que le nombre dérivé en x est supérieur ou égal à0. Donc on cherche le(s) intervalle(s) où la dérivée estpositive. L’ensemble des solutions de cette inéquation estdonc l’intervalle sur lequel la fonction f est croissante.b) La fonction f est décroissante sur chacun des intervalles[ –5 ; 0 ] et [ 6 ; 8 ] . La réunion de ces deux intervalles estl’ensemble solution de l’inéquation f ’(x) ≤ 0 .ex57 a) Tableau de variation de la fonction fx – –2 2 +f ’(x) – 0 + 0 –5f(x)0La fonction f est croissante sur l’intervalle [ – 2 ; 2 ] . Elle estdécroissante sur ] – ; – 2 ] et sur [ 2 ; + [ .b) f(2) = 5 et f ’(4) = = – 1,5 .f ’(4) est le coefficient directeur de la tangente à la .courbe au point C d’abscisse 4 .c) La droite (AE) a pour coefficient directeur , et 2 estl’ordonnée à l’origine ; d’où son équation : y = x + 2 .d) Équation de la tangente à au point E ( – 5 ; 4 ) :y = f ’(– 5) ( x – (– 5) ) + f(– 5) y = – 3 ( x + 5 ) + 4 y = – 3 x – 11 .ex58 1. a) f(x) = x 3 – 6x² + 9x – 1 sur f ’(x) = 3x² – 12x + 9 .b) f ’(1) = 0 et f ’(2) = – 3 . f ’(3) = 0 .(Attention ! les unités sont différentes sur chaque axe.)3. Équation de la tangente à f au point F ( 4 ; 3) :y = f ’(4) ( x – 3 ) + f(4) y = 9 ( x – 4 ) + 3 y = 9 x – 33 .ex59 f (x) = 0,5 x² + x – 1 sur .1. a) f ’(x) = x + 1 .b) f ’(– 4) = – 3 ; f ’(– 1) = 0 et f ’(1) = 2 .2. Tableau des variationsx –5 –1 3f ’(x) – 0 +6,56,5f (x)–1,53. a) Dans le repère, on place les points :A ( – 4 ; 3 ) ; B (– 1 ; – 1,5 ) et C ( 1 ; 0,5 ) .Les tangentes à en ces points ont pour coefficientdirecteur respectif – 3 ; 0 et 2 .b) Tangentes et parabole .Pour tracer une tangente, on place le point de tangence,puis on utilise le coefficient directeur de la tangente pourplacer un autre point par la méthode de « l’escalier ».ex60 1. a) f(x) = x 3 – x² – 2x + 4 . f ’(x) = x² – x – 2 .b) f ’(– 1) = 0 , f ’(1) = – 2 et f ’(2) = 0 .c) Les nombres dérivés sont nuls en –1 et 2 .On en déduit qu’aux points d’abscisses –1 et 2 , lestangentes à la courbe f sont horizontales.2. a) Tableau de valeursx –3 –2 –1 0 1 2 3 4f(x) –3,5 3,3 5,2 4 1,8 0,7 2,5 9,3b) c) Tracé de la courbe et des tangentes-3-2Ay87654321-1 0 1-1-2-3BCC f2 3 4 5xLivre du professeur - Mathématiques STMG Term STMG © Hachette Livre 2013 62


ex61 1. Le maximum de la fonction f sur [ –5 ; 5 ] est 4,atteint en x = – 2,5 .2. D’après le tableau de variation, l’équation f(x) = 0 a uneseule solution dans [ – 2,5 ; 5 ] . Cette solution est x = 2 .3. Comme la fonction f est décroissante sur [ – 2,5 ; 5 ] , ona f ’(x) ≤ 0 .4. Équation de la tangente à au point ( – 5 ; 1 ) :y = f ’(– 5) × ( x – (– 5) ) + f(– 5) y = 2 ( x + 5 ) + 1 y = 2x + 11 .5. Tracé d’une courbe représentative possible de f :On place les points puis les tangentes et on trace unecourbe « lissée » passant par ces points et ayant les droitesvertes pour tangentes.yex70 a) f ’(x) = .Si f = , alors f ‘ = .Il n’y a pas de x au numérateur de f(x) .b) f ’(x) = = .ex71 a) f ’(x) = . b) f ’(x) = .ex72 a) f ’(x) = . b) f ’(x) = .ex73 a) f ’(x) = . b) f ’(x) = 2 – .43ex74 a) f ’(x) =1 – . b) f ’(x) = .-6-5-4-3-221-1 0 1-12 3 4 5 6xex75 a) f ’(x) = + .b) f ’(x) = = . pour réduire au même dénominateur p. 215-2-3ex76 a) f ’(x) = . b) f ’(x) = .ex62 Réponse b. On calcule :f ’(x) = = = .ex63 QCM1. Réponse a. 2. Réponse c.ex64 a) f ’(x) = . b) f ’(x) = .ex77 a) f ’(x) = 2x – 20 – . b) f ’(x) = – 0,4 x + 50 –.ex78 a) C M ‘(q) = 1 – . b) f ’(q) = 1 – .ex79 a) = .b) f ’(q) = 2q – 6 – .ex65 a) f ’(x) = . b) f ’(x) = .ex66 a) f ’(x) = 8 x – .b) f ’(x) = – 2x + 9 – 7 × = – 2x + 9 + .ex67 f(x) = sur [ 0 ; 10 ] . On a u’(x) = 4 et v’(x) = 2 .f ’(x) = = .ex68 f(x) = sur [ 1 ; + [ . On a v’(x) = 5 .f ’(x) = = .ex80 f ’(t) = .ex81 f ’(x) = .ex82 f ’(x) = .ex83 C M (x) = , avec x >0On applique = + et on simplifie .C M (x) = – = x² – 15x + 85 + .b) C M ‘(x) = 2x – 15 – .ex69 a) f(x) = sur [ 0 ; + [ . On a v’(x) = 3 .f ’(x) = =b) f(x) = sur . On a v’(x) = 2x .f ’(x) = .ex84 On a f (x) = sur [ 0 ; 6 ] .a) f ’(x) = = = .b) f ’(x) est le quotient de – 13, négatif, par un carrétoujours strictement positif sur [ 0 ; 6 ] , donc la dérivéef ’(x) est toujours négative.c) On déduit que la fonction f est décroissante sur [ 0 ; 6 ] .Livre du professeur - Mathématiques STMG Term STMG © Hachette Livre 2013 63


d) On calcule f(0) = = 8 et f(6) = = = 2 .On peut aussi utiliser le tableau de valeurs de la fonctionobtenu sur calculatrice.e) Tableau de variationx 0 6f ’(x) –f (x)82ex85 g(x) = sur [ 1 ; 8 ] .a) g’(x) = = .b) La dérivée est toujours positive comme quotient de deuxnombres strictement positifs sur [ 1 ; 8 ] .c) On en déduit que la fonction g est croissante sur [ 1 ; 8 ] .d) g(1) = = 2,25 et g(8) = 4 .e) Tableau de variation de gx 1 8g ’(x) +g(x)42,25On en déduit que f ’(x) est du signe du numérateur – x² + 4(tracer la parabole qui coupe en –2 et 2).x –2 2– x² + 4 – 0 + 0 –c) D’où le signe de f ’(x) sur [ 0,5 ; 8 ]x 0,5 2 8f ’(x) + 0 –3. a) Tableau de variation de fx 0,5 2 8f ’(x) + 0 –f(x)0,550,5b) D’après le tableau de variation précédent, les valeurs def(x) sont dans l’intervalle [ 0,5 ; 5 ] , ainsi f(x) est strictementpositive pour tout réel x de [ 0,5 ; 8 ] .ex88 On a f (x) = sur [ 1 ; 11 ] .1. L’expression f (x) est saisie en Y3 .Attention aux parenthèses : s’écrit ( N ) / ( D ) .2. a) Tableau de valeursex86 On a f (x) = x – 4 + sur [ 1 ; 9 ] .1. a) D’après la courbe, la fonction f semble croissante surl’intervalle [ 3 ; 9 ]. « Conjecturer » p. 210b) f ’(3) est le coefficient directeur de la tangente au pointd’abscisse 3 tracée sur le graphique, d’où f ’(3) = 0 .f(3) = 2 est l’ordonnée du point.c) On trace la droite horizontale d’équation y = 3,5 quicoupe la courbe en deux points. On lit les abscisses :l’équation f (x) = 3,5 a deux solutions 1,5 et 6 . p. 2142. a) f ’(x) = = .On résout l’équation x² – 9 = 0 x = 3 ou x = – 3 .D’où le tableau de signe de x² – 9x –3 3x² – 9 + 0 – 0 +f ’(x) est le quotient de x² – 9 par x² , strictement positif sur[ 1 ; 9 ] , donc f ’(x) est du signe de x² – 9 sur [ 1 ; 9 ] .b) Tableau de variationx 1 3 9f ’(x) – 0 +f (x)662ex87 f(x) = – x + 9 – sur [ 0,5 ; 8 ] .1. D’après la courbe , il semble que la fonction f soitcroissante sur [ 0,5 ; 2 ] et décroissante sur [ 2 ; 8 ] .2. a) f ’(x) = – 1 + = .b) f’(x) est un quotient dont le dénominateur x² eststrictement positif sur [ 0,5 ; 8 ] .On lit f(1) = 8 et f(11) = 3,2 .b) Représentationc) On conjecture que la fonction f semble décroissante surl’intervalle [ 1 ; 11 ] .ex89 f(x) = 2 + sur [ 1 ; 9 ] .1. a) f ’(x) = . La dérivée est toujours négative.b) Donc la fonction f est décroissante sur [ 1 ; 9 ] .c) f(1) = 7 et f(9) = 3 .Concrètement cela signifie qu’à 7 € le kg, la demande est de1 tonne et à 3 € le kg, la demande est de 9 tonnes.2. a) f ’(4) est le coefficient directeur de la tangente à lacourbe au point d’abscisse 4 . D’où f ’(4) = – 0,4 .b) f ’(1) = = – 2,5 .3. Tracé de lacourbe représentative etdes tangentesen A( 1 ; 7 ) etB ( 4 ; 4 ) .Livre du professeur - Mathématiques STMG Term STMG © Hachette Livre 2013 64-1y76543210 1ABC f2 3 4 5 6 7 8 9Cx


ex90 f(x) = sur [ –6 ; 6 ] .1. a) Sur calculatriceb) f(– 6) = 1 et f(6) = 7D’après le tableau devaleurs, la fonction fn’est pas monotone.c) Courbe sur calculatrice2. a) On résout l’équation 4x² + 20x + 16 = 0 . = b² – 4 a c = 144 ; les solutions sont – 4 et – 1 .Les solutions de cette équation sont les abscisses des pointsd’intersection de la courbe avec l’axe des abscisses.b) f(0) = 4 . On en déduit que la courbe coupe l’axe desordonnées au point d’ordonnée 4 .3. a) On résout – 20 x² + 80 = 0 x² = 4 x = 2 ou x = – 2 .b) f ’(x) = = .c) Comme ( x² + 4 )² est strictement positif sur [ –6 ; 6 ] ,alors f ’(x) est du signe de – 20x² + 80 .d) Tableau de variation de f sur [– 6 ; 6 ] :x ––2 2 66f ’(x) – 0 + 0 –f(x)19– 174. a) f(0) = 4 . Au moment du lancement de la vente, 400appareils sont vendus.b) Le premier jour du deuxième mois, la vente journalièreest la plus importante avec 900 appareils vendus.ex91 B(x) = – 16 x² + 220 x sur [ 0 ; 18 ] .1. a) On résout l’équation x² – 32 x + 220 = 0∆ = b² – 4 a c = 144 . Les solutions sont 10 et 22 .b) B ‘(x) = x² – 32x + 220 .x 10 22x² – 32 x + 220 + 0 – 0 +c) Tableau de variationx 0 10 18B ‘(x) + 0 –B(x)≈ 93307202. a) B(3) = 525 . La vente de 3 hl de jus de fruit rapporte unbénéfice de 525 € .b) Le bénéfice maximal est de 933 € , à 1 € près , réalisé parla vente de 10 hL de jus de fruit.ex92 C(q) = 0,2 q 3 + 2q + 50 sur [ 0 ; 5 ] .1. a) C ‘(q) = 0,6 q² + 2 .C ‘(q) > 0 , comme somme de deux nombres positifs.b) Tableau de variationq 0 5C ‘(q) +85C (q)502. a) Le coût de fabrication de 100 toupies est C(1) ,exprimé en dizaines d’euros.Or C(1) = 52,2 et C(3) = 61,4 .Ainsi, pour 100 toupies, le coût de fabrication est de 522 €et pour 300 toupies, il est de 614 € .b) D’après la question 1. le coût de fabrication des toupiesest croissant sur l’intervalle [ 0 ; 5 ] .Remarque : une fonction de coût total étant la somme descoûts fixes et coûts marginaux, le coût total en fonction dela quantité est toujours croissant.3. a) La première fois où la valeur de F est calculée, en ligne5 , N = 0 + 1 = 1 .b) En ligne 5 on calcule l’image de N par la fonction f .c) Comme f(5) = 85 et la fonction f est croissante, lorsqueN = 5, alors F = 85 et la boucle "Tant que F < 85" se termine:on passe de la ligne 3 à la ligne 6 .La valeur affichée est N = 5 .ex93 f(x) = x 3 – 6x² + 50x sur [ 0 ; 10 ] .1. a) f(1) = 45 ; f(6) = 300 et f(10) = 900 .Au bout de 1 h, 6 h et 10 heures, les commentaires sontrespectivement au nombre de 45, 300 et 900 .b) f ’(x) = 3 x² – 12x + 50 . ∆ = b² – 4 a c =– 456 .Le discriminant est négatif, la dérivée ne s’annule pas et esttoujours positive. On en déduit que la fonction f estcroissante sur [ 0 ; 10 ] .2. a) f ’(6) =86 et f ’(10) = 230 . Au bout de 6 h,l’accroissement du nombre de commentaires est de 86 parheure et au bout de 10 h il est de 230 par heure.b) Équation de la tangente T :y = f ’(6) ( x – 6) + f(6) y = 86 ( x – 6 ) + 300 y = 86 x – 216 .3. On trace la tangente T en prenant deux points ( 6 ; 300 )et ( 10 ; 644 ), on place les points de la courbe en utilisant letableau de valeurs de la calculatrice, puis on trace .-1y8007006005004003002001000 1C f2 3 4 5 6 7 8 9 10xLivre du professeur - Mathématiques STMG Term STMG © Hachette Livre 2013 65


ex94 f(x) = – 0,5 x 4 + x 3 + 19 x² + 20 x sur [ 0 ; 12 ] .1. a) f ’(x) = – 2 x 3 + 16 x² + 38 x + 20 .b) L’écran de calcul formel indique la forme factorisée de ladérivée: f ’(x) = – 2 ( x – 10 ) ( x + 1 )² .2. a) Pour tout réel x , ( x + 1 )² ≥ 0 .En particulier, si x [ 0 ; 12 ] , alors ( x + 1 )² > 0 .b) Tableau de signe de la dérivéex 0 10 12–2 – –x – 10 – 0 +( x + 1 )² + +f ’(x) + 0 –c) Tableau de variation de la fonction f :x 0 10 12f ’(x) + 0 –f(x)≈2433018243. a) f(6) = 1 308 .Pour 6 000 litres d’engrais vendus en une semaine, lebénéfice réalisé par l’entreprise est 1 308 € .b) Pour 10 000 litres vendus en une semaine, le bénéficeest maximal et vaut 2 433 € à 1 € près.ex95 f(x) =1. f ’(x) = .sur [ 0 ; 9 ] : fonction d’oubli.La dérivée étant strictement négative sur [ 0 ; 9 ] , lafonction f est strictement décroissante sur [ 0 ; 9 ] .Tableau de variation de fx 0 9f ’(x) –60f(x)62. a) Par lecture graphique du coefficient directeur de latangente au point A d’abscisse 1 , on obtient :Et par calcul sur la dérivée :f ’(1) = = – 15 .f ’(1) = = = – 15 .b) Équation de la tangente au point B d’abscisse 5 :y = f ’(5) ( x – 5 ) + f(5) y = ( x – 5 ) + 10 y = x + .3. a) f(2) = 20 . Au bout de 2 mois, Arthur se souvient de 20paragraphes et il en a oublié 40 .b) f ’(2) = ≈ – 6,7 . Au bout de deux mois, Arthur oublieau rythme de 6,7 paragraphes par mois.ex96 CT(q) = 15 q 3 – 120 q² + 500 q + 750 .q est la quantité comprise entre 0 et 10 tonnes et CT(q) estle coût total de commercialisation en euros.C m (q) = CT ’ (q) , le coût marginal est défini sur [ 0 ; 10 ] .M(q) = , le coût moyen est défini sur ] 0 ; 10 ] .1. a) C m (q) = 45 q² – 240 q + 500 , pour q [ 0 ; 10 ] .b) C m ’(q) = 90 q – 240 .On résout l’équation 90q – 240 = 0 q = = ≈ 2,7 .Tableau de variation de la fonction coût marginalq 0 10C m ’(q) – 0 +5002600C m (q)180c) Sur l’intervalle [ 0 ; 10 ] , le minimum du coût marginalest égal à 180, positif. On en déduit que le coût marginalC m (q) est strictement positif sur [ 0 ; 10 ] .2. a) M(q) = = 15 q² – 120 q + 500 + .b) M ‘(q) = 30 q – 120 – = .= .La factorisation du numérateur de M’ (q) est obtenue àl’aide d’une calculatrice formelle (ici TI Nspire) .c) On considère le trinôme P(q) = q² + q + 5 .∆ = b² – 4 a c = –19 , ∆ < 0 . Le trinôme n’a pas de racine etP(q) est toujours strictement positif.d) D’après la forme factorisée de M’(q) et d’après laquestion précédente, on déduit que :M’(q) est du signe de q – 5 sur ] 0 ; 10 ] .q 0 5 10M’(q) – 0 +875M(q)4253. a) Le coût moyen est minimum pour une quantitécommercialisée q 0 = 5 tonnes .b) C m (5) = 45 × 5² – 240 × 5 + 500 = 425 .D’après l’étude des variations de la fonction de coût moyenM, on a M(5) = 425 .c) En reprenant les tableaux de variation du coût marginalet du coût moyen, on observe que le coût marginal estreprésenté par la courbe 1 et le coût moyen par la courbe 2 .4. a) En ligne 4 , on a le calcul du coût moyen pour unequantité égale à l’entier N et en ligne 5, on a le calcul ducoût marginal pour ce même entier N .b) La condition TANT QUE M > C permet de tester si pourcet entier N, le coût moyen est inférieur ou égal au coûtmarginal.c) Le résultat affiché à la sortie est N = 5 .Dans cet exercice, on a observé la propriété :le coût marginal est égal au coût moyenlorsque le coût moyen est minimal.ex97 f(x) = 10x – 20 +. x le nombre de jours depuis lamise en rayon de l’article, avec x [ 1 ; 20 ] .f(x) le nombre d’articles vendus, en dizaines, le jour x.1. a) Soit l’équation x² + 2x + 4 = 0 .∆ =b² – 4 a c = – 12 . Le discriminant est négatif doncl’équation n’a pas de solution dans .b) N(x) = 10 x 3 – 80 = 10 ( x – 2 ) ( x² + 2x + 4 ) .Le polynôme x² + 2x + 4 ne s’annule pas et il est toujoursstrictement positif. N(x) est du signe de x – 2 .Livre du professeur - Mathématiques STMG Term STMG © Hachette Livre 2013 66


D’où le signe de N(x) sur [ 1 ; 20 ]x 1 2 20N(x) – 0 +2. a) f ’(x) = 10 + 40 × = 10 – 40 × =b) f ’(x) = .c) On a x [ 1 ; 20 ] , on en déduit que x 3 est strictementpositif et f ’(x) est donc du signe de N(x) .Tableau de variation de la fonction f :x 1 2 20f ’(x) – 0 +30180,1f(x)103. a) f(10) = 80,4 . Le 10 e jour, 804 articles sont vendus.b) Le nombre d’articles vendus est minimum le deuxièmejour avec 100 articles vendus.ex98 Partie A g(x) = sur .1. a) g’(x) = = .b) Le dénominateur de g’(x) est strictement positif, g’(x) a lemême signe sur que le numérateur N(x) = 16 – x² .c) On résout l’équation 16 – x² = 0 x = 4 ou x = – 4 .Tableau de variation de la fonction gx –4 4g’(x) – 0 + 0 –0,125g(x)–0,125Partie B1. c. = A3/(A3*A3+16)2. En cellule C3 , on saisit la formule =16*B3+13. La fonction f est définie sur [ 0 ; 12 ] par :f(x) = 16 × g(x) + 1 .Par conséquent, f ’(x) = 16 × g’(x) + 0 = 16 × g’(x) .4. La fonction f est croissante sur [ 0 ; 4 ] puis décroissantesur [ 4 ; 12 ] .Jusqu’au 4 ième mois après le lancement de la campagnepublicitaire le nombre d’abonnés a augmenté pouratteindre un maximum de 300 000 abonnés au bout de 4mois. Carmen a raison : la campagne publicitaire a permisune croissance du nombre d’abonnés.ex99 On a f(x) = + + 20 sur [ 1 ; 10 ] .Partie A1. a) f ’(x) = + .b) f ’(x) = = = .2. a) Sur l’intervalle [ 1 ; 10 ] , le dénominateur de la dérivéeest strictement positif, on en déduit que f ’(x) a le mêmesigne que 121 – 22 x . On résout :121 – 22x =0 x = 5,5 .b) D’où le tableau de variation de la fonction f.x 1 5,5 10f ’(x) + 0 –≈19,64f(x)018,9Partie Ba) Le bénéfice unitaire maximum est réalisé pour la ventede 55 pièces de moteur de voiture et il vaut 19,64 euros parpièce, à 0,01 € près.b) Dans ces conditions, le bénéfice total lorsque le bénéficeunitaire est maximum s’élève à 55 × 19,64 = 1080,20 € .ex100Partie Af(x) = sur [ 3 ; 45 ] .1. f ’(x) = = .2. Sur l’intervalle [ 3 ; 45 ] , comme le dénominateur de ladérivée f ’(x) est strictement positif, alors f ’(x) est du signede 3 ( x² – 1 ) .Signe de 3 ( x² – 1 ) sur :x –1 13( x² – 1 ) + 0 – 0 +On en déduit que pour x [ 3 ; 45 ] , f ’(x) ≥ 0 .Tableau de variation de la fonction f :x 3 45f ’(x) –0,1f(x)≈ 0,933. D’après le graphique, le plus petit nombre entier a quivérifie l’inéquation f(x) > 0,6 est a = 8 .Partie B1. D’après la partie A , 60 % des billets ont été vendus aucours du 8 ième jour après le début de la mise en vente desbillets.2. a) Trente jours après le début de la mise en vente desbillets, 90 % des billets étaient déjà vendus.b) Si l’utilisateur choisit 0,6 comme valeur de P, 8 est lavaleur de X qui apparait à la sortie de l’algorithme.ex101 Texte en françaisOn considère la fonction f définie sur [ –7 ; 7 ] par :f(x) = .Sa courbe représentative est visualisée ci–dessous àl’aide d’une calculatrice (TI 82), dans la fenêtre donnée.1. a) Lire f(0,5) .b) Calculer f(0) et f(–2) .2. Calculer f ’(x), où f ‘ est la dérivée de f.3. a) Résoudre l’équation – 4 x² – 6 x + 4 = 0 .b) Montrer que f ’(x) a le même signe que le polynôme– 4 x² – 6 x + 4 .c) En déduire les variations de la fonction f sur [ – 7 ; 7 ] .ex102 QCM1. Réponse b. 2. Réponse a. 3. Réponse c.ex103 QCM1. Réponse c. 2. Réponse c. 3. Réponse a.ex104 QCM1. Réponse a. 2. Réponse c. 3. Réponse c.Livre du professeur - Mathématiques STMG Term STMG © Hachette Livre 2013 67


ex105 Partie A1. Deux solutions, abscisses des points d’ordonnée 6 .2. f ’(x) = 0 x = 2 , abscisse du point où la tangente esthorizontale.3. f ’(1) = – 3, coefficient directeur de la tangente aupoint d’abscisse 1 de .Partie BOn a f (x) = x + 1 + sur ] 0 ; + [ .1. f ’(x) = 1 – = .2. La dérivée f ’(x) est du signe de x² – 4 sur ] 0 ; + [ .On résout l’équation x² – 4 = 0 x = –2 ou x = 2 .Comme a = 1 > 0, on a l’allure de laparabole et sa position par rapport àl’axe des abscisses.D’où le signe de la dérivée f ’(x) sur ] 0 ; + [ et le tableaudes variations de la fonction f .x 0 2 +f ’(x) – 0 +f (x)5Partie C1. On a C(x) = f (x) × x sur [ 1 ; 8 ] .C(2) = f(2) × 2 = 5 × 2 = 10 .Le coût total de production d’une quantité de 2 tonness’élève à 10 milliers d’euros.2. C(x) = x ( x + 1 + ) = x² + x + 4 .3. C ‘(x) = 2 x + 1 et C ‘(x) > 0 sur [ 1 ; 8 ] .D’où le tableau des variations de la fonction C .x 1 8C ‘(x) +C(x)766ex106 Coût , en euros, d’une production mensuelle de xtonnes donné par C(x) = x 3 – 36 x² + 445 x où x [ 0 ; 24 ] .Partie A Coût moyenC M (x) = = x² – 36 x + 445 sur ] 0 ; 24 ] .1. C M ‘ (x) = 2 x – 36 .2. Variations de la fonction coût moyen sur ] 0 ; 24 ] :x 0 18 24C M ‘(x) – 0 +(445)157C M (x)121Ne pas trop insister sur le cout moyen non défini en 0 .3. Coût moyen minimum : 121 € par tonne.Partie B Bénéfice1. a) B(x) = 160x – ( x 3 – 36 x² + 445x ) = – x 3 + 36 x² – 285 x .b) B(15) = 450 € .2. a) On résout l’équation – 3 x² + 72 x – 285 = 0 . = b² – 4 a c = 1764 . Les solutions sont 19 et 5 .b) B’(x) = – 3 x² + 72 x – 285 .c) Tableau de variation de la fonction Bx 0 5 19 24B ‘(x) – 0 + 0 –0722B(x)– 650723. Pour 19 tonnes vendues, le bénéfice est maximal et cebénéfice vaut 722 € .4. Par lecture graphique, on constate que l’entreprise estdéficitaire pour les ventes inférieures à 12 tonnes.ex107Partie AOn a P(x) = 3x² – 2x – 1 .1. On résout l’équation P(x) = 0 . = b² – 4 a c = 16 . Les solutions sont et 1 .2. Allure de la parabole représentant le trinôme P :3. Signe de P(x) sur x 1P(x) + 0 – 0 +Partie BOn a f(x) = sur ] 0 ; 9 ] .1. u’(x) = 4 x – 1 et v’(x) = 2x + 1 . D’où :f ’(x) = = = .2. D’après le résultat obtenu en calcul formel (Ti Nspire), onen déduit que f ’(x) ≥ 0 pour x [ 1 ; 9 ] .Comme f ’(x) est du signe de P(x) , on peut aussi utiliser lerésultat de la question 2. de la partie A qui donne le signede P(x) sur .3. f ’(x) étant positif sur [ 1 ; 9 ] , la fonction f est donccroissante sur l’intervalle [ 1 ; 9 ] .Partie C1. f(4) = 1,45 .2. En C2 , on saisit la formule =1/(A2+1)3. En D2 , on saisit la formule =B2 – 4*C2 + 2Partie Da) Jusqu’à 3 jours de fonctionnement, le coût unitairejournalier de l’atelier est inférieur à 1 330 € .b) On observe le tableau de valeurs de la fonction f .Si on choisit 1,7 comme valeur de V , la valeur 9 apparaît àla sortie de cet algorithme.Jusqu’à 8 jours de fonctionnement, le coût unitairejournalier de l’atelier est inférieur à 1 700 € .Livre du professeur - Mathématiques STMG Term STMG © Hachette Livre 2013 68


Intentions des auteures et conseilsL’objectif de ce chapitre est l’acquisition de la loibinomiale vue en première et l’introduction de la loinormale.Dans le cadre d’un enseignement en spirale, cechapitre peut être traité en 2 parties.Cours 1 et 2 : reprenant exclusivement le programmede première, cette partie peut être vue tôt.Cependant, nous conseillons de l’étudier après lesprobabilités conditionnelles. On peut alors parler duschéma de Bernoulli comme un arbre de probabilitéparticulier. La notion d’indépendance, en dehors ducas du schéma de Bernoulli n’est pas au programme.Cours 2 et 3 : peut être traitée après le début dessuites ou des fonctions. Il sera nécessaire que lesélèves sachent bien utiliser leur calculatrice.Nous conseillons un délai d’au moins un mois entreces deux parties.• Les calculs étant totalement réalisés à l’aide desoutils calculatoires, l’enseignement doit se concentrersur les situations et l’interprétation des résultatsobtenus. Les élèves doivent comprendre les notationsdes événements { X = k } et { X ≤ k } et les probabilitésP( X = k ) et P( X ≤ k ) , que ce soit pour la loi binomiale(où k est obligatoirement un entier) et la loi normale(où k est un réel).• Pour la loi normale, l’objectif est bien différent decelui de la classe de BTS des années précédentes :– pas de loi normale centrée réduite, puisque lacalculatrice effectue directement les calculs ;– pas de loi normale approchant une loi binomiale,puisque celle-ci peut être traitée directement à lacalculatrice.• Le programme indique : « La loi normale peut êtreintroduite à partir de l’observation, à l’aide d’unlogiciel, de la loi binomiale ». Et plus loin « *…+ enapprochant la loi binomiale par la loi normale demême espérance et d’écart type p 1 − p n *…+ » .Après mûres réflexions, nous avons choisi de réservercette introduction en atelier TICE seulement. En effetnous l’avons jugée bien trop théorique pour les élèvesde STMG et comportant un risque majeur : associersystématiquement la loi normale à la loi binomiale.• Nous avons choisi de lier la loi normale à unerépartition dite «normale» d’une population, dans unsens plus concret que mathématique. Nous avonsvoulu très vite visualiser la courbe en cloche de cetterépartition et ainsi suivre le commentaire duprogramme : « Les élèves doivent connaître l’allure dela courbe de densité, ainsi que sa symétrie ». Cetteintroduction englobe bien toutes les situations liées àla loi normale, y compris si on approche la loibinomiale (sous certaines conditions) par une loinormale. Les conditions introduites en TES ou TS nesont pas à aborder en STMG puisque les élèves n’ontpas à faire cette approximation ! .• Dans les deux chapitres 5 et 7, nous avons faitattention à appeler proportion la part (connue ouinconnue) d’un caractère dans la population TOTALEet fréquence la part de ce caractère dans unéchantillon de taille connue, « choisi » au hasard danscette population.D’où le choix des couleurs, dans toute la mesure dupossible : proportion en rouge et fréquence et tailleen vert.Avant de commencerQCM1 : Réponse c) P( X = 3 ) = 0,2 3 = 0,008 .Ce QCM permet de revenir sur le schéma de Bernoulli,le principe multiplicatif sur l’arbre, ainsi que lanotation des événements { X = k } .Ne pas hésiter à demander le calcul des probabilitésde tous les événements { X = k } .La réponse a) revient sur les valeurs de k de 0 à n .Pour b) demander P( X = 2 ) = 3 × 0,2² × 0,8 = 0,096et P( X = 1 ) = 3 × 0,2 × 0,8 ² = 0,384 .QCM2 : Réponse b) de façon immédiate.Ce QCM revient sur la notation P( X ≤ k ) , sa lecture« au moins » , « au plus » …Ne pas hésiter à faire remplir le tableau :P( X = 4 ) = 0,2 et P( X ≤ 4 ) = 1 – 0,08 = 0,92 .Livre du professeur – Mathématiques Term STMG © Hachette Livre 2013 69


QCM3 : Réponse c) Ce QCM revient sur la moyenneet l’écart type. Il est nécessaire d’avoir étudié le cours1 du Ch1 et l’emploi de la calculatrice.La moyenne est x = 8,3 et l’écart type est σ ≈ 1,11 .On confirme en calculant les bornes de l’intervalle dit« standard » en anglais.QCM4 : Réponse b) La moyenne est x = 125 .La encore, la calculatrice est obligatoire.a) σ ≈ 8,25 . c) Faux , car 11+5+2 = 18 .L’histogramme, présenté comme un diagramme enbâtons aux centres de classes, donne une idée d’unerépartition «normale» pour un prix continuOn peut aussi imaginer que l’on a donné un prix de 5 €en 5 € aux acheteurs potentiels.3. Les probabilités sont très faibles. En effet, avoir plusde 60 clients intéressés est pratiquement impossible.On peut montrer les valeurs données par le logiciel :elles ne sont pas nulles !4.a) P( X = 36 ) ≈ 0,0793 dans le tableau de valeurs.P( X ≤ 36 ) ≈ 0,5449 d’après la probabilité calculée.Demander une interprétation.CoursLoi binomiale de paramètres n et pÉtude d’une situationNous avons fait le choix de reprendre entièrement laloi binomiale, en utilisant les résultats obtenus à l’aided’un logiciel (ici GeoGebra).Remarque : Depuis l’écriture de l’ouvrage, le visuel achangé sur ce logiciel. Nouveau visuel :Nous conseillons de faire vérifier les résultats de cettesituation en utilisant la calculatrice. p. 118 : mise en pratique pour P( X ≥ 36 ) et lacorrection de l’exercice 2 , donnée en p. 229.1. Lecture de l’énoncé : on répète 120 foisl’expérience consistant à interroger un client sur sonintérêt pour l’achat de ce jeu. Revenir sur ce que l’onappelle le succès et sa probabilité.2. a) P( X = 30 ) = 0,04 .b) Traduire en notation mathématique :P( X = 26 ) ≈ 0,0107 .Demander d’interpréter de même la question 1.a).b) 36 est l’espérance de la loi binomiale : µ = n × p .Elle est donnée dans la nouvelle version du logiciel (enbas du graphique). On peut faire apparaître la courbede la loi normale sur le graphique en cliquant .Objectifs des exercices : Revoir le schéma de Bernoulliet calculer des probabilités à partir d’un arbrepondéré. Dans le cadre de la loi binomiale, calculerdes probabilités «ponctuelles» P( X = k ) ou cumuléesP( X ≤ k ) , à l’aide de la calculatrice ou sur tableur, etinterpréter les résultats obtenus.Fluctuation d’une fréquenceÉtude d’une situationOn revient sur l’intervalle de fluctuation rencontré enpremière. La visualisation du diagramme présentantcet intervalle permet une approche de la courbe encloche et son interprétation. Nous pensons qu’il n’estLivre du professeur – Mathématiques Term STMG © Hachette Livre 2013 70


pas utile d’insister si les élèves ont du mal à trouvercet intervalle.Il est important que les élèves comprennent que laprobabilité de trouver la fréquence observée dans cetintervalle est supérieure à 0,95 et connaissent la prisede décision et savoir que cette fréquence dépend del’échantillon. D’où le schéma : les échantillons sonttous de même taille, représentés par un carré vert,mais suivant le tirage aléatoire dans la population, onobtient des fréquences différentes.On peut aussi reprendre le calcul proposé dans lemanuel de 1 re STMG p. 161 . Créer éventuellement unprogramme pour les élèves qui le veulent.Rien de nouveau par rapport au cours de première.1. P( X ≤ 40 ) ≈ 0,8157 . La probabilité que au plus 40clients soient intéressés parmi les 120 est de 0,8157 .2. On lit k = 36 , car P( X ≤ 36 ) ≈ 0,5449 .3. a) a = 26 , car P( X ≤ 26 ) ≈ 0,0266 .b) b = 46, car P( X ≤ 46 ) ≈ 0,9800 .Sous GeoGebra, la figure présentée dans « Ce qu’ilfaut retenir » peut se faire par « Copier versgraphique ».On obtient P( X * 26 ; 46 + ) ≈ 0,964 > 0,95 .Il faut faire comprendre aux élèves que l’on n’a pasexactement 0,95 , vu les arrondis faits, mais au moins95 % . 120 n’est pas une grande taille d’échantillon.Cette idée sera encore plus mise en valeur dans lechapitre 7 .4. L’intervalle de fluctuation est :I =a; b120 120=26; 46120 120≈ * 0,2166 ; 0,3834 + .Emma observe une fréquence de 0,18 . Or 0,18 ISi on est « certain » de la proportion de 30 %, on peutpenser que l’échantillon d’Emma n’est pasreprésentatif. Sinon, c’est la proportion de 30 % declients intéressés dans la population totale qui est àrejeter.Objectifs des exercices : toujours pratiquer lacalculatrice, et obtenir la liste des probabilitéscumulées sur calculatrice comme sur tableur.Sur Casio, l’instruction BinomialCD (C pour cumulé)n’est pas simple à trouver. Nous avons donc choisid’utiliser la touche Catalog par .De plus, dans une classe où les deux types decalculatrices existent, le professeur explique ainsi lamême démarche à tous.Sur TI, il est facile de calculer P( X [ 19 ; 28 ] ) par :binomFRèp( 30 , 0.8 , 28 ) – binomFRèp( 30 , 0.8 , 19 )Nous ne savons pas comment faire sur Casio 35+.Les exercices d’approfondissement proposés (62, 63et 64) reviennent sur la loi binomiale.Loi normale d’espérance µ et d’écart type σÉtude d’une situationNous avons choisi de présenter la loi normale, sanschercher à la lier à la loi binomiale, à partir d’unedistribution de poids donnant bien une idée de loicontinue. On revient ainsi à la notion très simple deprobabilité, en analysant la valeur d’un caractère surun individu pris au hasard dans une population, enassociant une variable aléatoire et en cherchant uneprobabilité sur cette variable aléatoire.Nous avons parlé de distribution des fréquences sansdonner de valeurs à ces fréquences (pas d’ordonnéesur le graphique) .Le graphique est un histogramme etnon un diagramme en bâtons. Les valeurs entières,tous les kg, sont des centres de classe. Cependantmieux vaut ne pas insister sur cette idée et passerrapidement à la «courbe en cloche» qui lisse cediagramme et son interprétation. L’essentielle est debien faire comprendre que la somme des fréquences,c’est-à-dire 1, est représentée par la surface sous lacourbe.Remarque : Si l’on veut introduire à l‘aide de la loibinomiale : Atelier TICE 1 et 2 p. 126 .On peut aussireprendre la loi binomiale vue p. 118 : le logicielGeoGebra permet de dessiner la courbe en cloche surla distribution de la loi binomiale.Mais il nous a semblé « risqué et réducteur » decommencer ainsi : difficile ensuite de passer à une loicontinue si la variable aléatoire prend des valeursdiscrètes. Ce n’est déjà pas facile de trouver des cas« concrets » avec une variable aléatoire qui prend desvaleurs négatives. On peut présenter la variablealéatoire Y correspondant à l’écart du poids d’un veaupar rapport à la valeur moyenne de 35 kg : on centreainsi à la moyenne. On peut alors faire vérifier que laprobabilité P( – 10 ≤ Y ≤ 10 ) ≈ 0,6827 .La courbe en cloche est simplement translatée.Livre du professeur – Mathématiques Term STMG © Hachette Livre 2013 71


On rappelle que la loi normale centrée réduite n’estpas au programme de cette classe.1. Non. C’est une question délicate, il faut que lesélèves comprennent que cet événement estimpossible, donc sa probabilité est nulle. Tout le resteva en découler ! Le poids est une variable continue.2. La courbe est symétrique par rapport à la droiteverticale d’équation x = 35 . D’où les résultats :a) P( X ≤ 35 ) = 0,5 b) P( X ≥ 35 ) = 0,5 .3. Par symétrie : P( X ≤ 30 ) = P( X ≥ 40 ) .Donc P( X ≤ 30 ) = 1 2× ( 1 – P( 30 ≤ X ≤ 40 ) )≈ 1 2× ( 1 – 0,6827 ) = 0,1586 .On peut commencer à utiliser cette symétrie pourd’autres calculs.Nous avons évité d’écrire de façon mathématique quel’aire sous la courbe est égale à 1 .Comment l’écrire proprement avec les notationsconnues des élèves de STMG ? Nous avons mêmeparlé le plus possible de « surface », plus facile àappréhender que la notion d’aire.Il nous semble essentiel que les élèves comprennentque ce modèle « normal » répond à des critères. Onpeut ainsi commencer à faire « sentir » le rôle del’écart type.Objectifs des exercices : manipuler la calculatrice leplus vite possible pour obtenir une probabilité. Savoiraussi lire sur la courbe en cloche et utiliser sasymétrie.Le rôle de l’écart type est vu dans l’atelier TICE 3 etl’exercice 26 p. 133.Intervalle de fluctuation [ µ – 2 σ ; µ + 2 σ ]Étude d’une situationOn s’intéresse à un intervalle particulier, intervalle defluctuation de la variable aléatoire : la probabilité quela taille d’un bébé quelconque soit dans cet intervalleétant au moins 0,95 . Ici la variable aléatoire est biencontinue et on reste dans le langage courant, afin debien ancrer la notion.Le diagramme de distribution des fréquences est undiagramme en bâtons, puisque les tailles sontarrondies au cm. Maiscomme pour le cours 3,inutile d’insister. L’importantest d’imaginer la courbe encloche et l’aire sous la courbeégale à 1 .1.a) X suit la loi normale centrée en µ = 50 et d’écarttype σ = 2 .b) P( 48 ≤ X ≤ 52 ) ≈ 0,6827 .c) P( 46 ≤ X ≤ 54 ) ≈ 0,9545 .La probabilité qu’un bébé aitune taille située a plus dedeux écarts types de la taillemoyenne est 0,9545 .2. Parmi tous les bébés, le pourcentage de ceux ayantune taille dite normale est d’environ 95 % .Objectifs des exercices : Interpréter l’intervalle defluctuation et retrouver les paramètres de la loinormale à partir de cet intervalle. Comprendre le rôlede l’écart type et l’interpréter sur la courbe dedensité. On retrouve la modélisation d’unedistribution par une loi normale : plus on s’écarte dela valeur moyenne, plus la probabilité est faible.L’exercice résolu p. 125 présente l’approximationd’une loi binomiale par une loi normale. Mais, a priori,on n’a pas besoin de cette approximation si ondétermine directement à la calculatrice le tableau dela loi binomiale, avec p = 40= 0,05 probabilité qu’un800article ait un défaut.On retrouve alors le même intervalle de fluctuation à95 % que pour la loi normale, soit [ 28 ; 52 ] et laprobabilité, par la loibinomiale , est :P( X * 28 ; 52 + ) ≈0,9751 – 0,0264 = 0,9487Atelier TICELave-vaisselle chez les 25–39 ansLes deux ateliers 1 et 2 peuvent se traiter en classeentière, la moitié de la classe sur calculatrice et l’autremoitié sur logiciel GeoGebra. Et on change les groupesau bout de 20 min. Si les élèves maîtrisent bien lacalculatrice, cela peut aller très vite.Le programme indique le calcul de l’écart type de laloi binomiale. Inutile d’insister sur cette formule.Depuis l’écriture de ce chapitre, le logiciel GeoGebra aintégré cette demande, et lorsque l’on représente laloi binomiale, on voit apparaître l’espérance µ etl’écart type σ sur le graphique.A. Étude de la loi binomialea) Les paramètres sont n = 100 et p = 0,5 .D’où l’espérance µ = n × p = 50 .b) Ni 1, ni 0,5 . La probabilité d’obtenir exactement 50ménages équipés est P( X = 50 ) ≈ 0,0796 .c) P( X ≤ 50 ) ≈ 0,5398 .d) P( 45 ≤ X ≤ 55 ) ≈ 0,6803 .Livre du professeur – Mathématiques Term STMG © Hachette Livre 2013 72


B. Loi normalea) P( 40 ≤ X ≤ 60 ) ≈ 0,9545B. Étude de la loi normalea) n × p × (1 – p) = 100 × 0,5 × 0,5 = 25 . D’où σ = 5 .b) Oui, propriété de la loi normale.c) P( 45 ≤ Y ≤ 55 ) ≈ 0,6827 .Représentations à l’aide de GeoGebraNous avons indiqué les deux vocabulaires et visuelsque l’on peut rencontrer suivant la version du logiciel.Peut–être que dans 2 ou 3 ans, cela changera encore…Le but est de faire le lien entre les deuxreprésentations : diagramme en bâtons de la loibinomiale, détermination des paramètres de la loinormale l’approchant, et visualisation de la courbe dedensité. Comparaison de probabilités.A. Loi binomialea) Lecture directe du visuel donné :P( 40 ≤ X ≤ 60 ) ≈ 0,9648 .On obtient un diagramme en bâtons, donnant lesprobabilités des entiers k de 0 à 100 . Le bâtoncentral, en X = 50 donne l’axe de symétrie.b) On obtient P( X ≤ 50 ) ≈ 0,5398 .c) On obtient P( 45 ≤ X ≤ 55 ) ≈ 0,7287 .La probabilité que la variable aléatoire soit compriseentre 40 et 60 est 0,9545 .Le graphique est une courbe en cloche, symétriquepar rapport à la droite verticale d’équation x = 50 .b) On obtient P( X ≤ 50 ) = 0,5 .c) P( 45≤ X ≤ 55 ) ≈ 0,6827 .C. Comparaisona) Les deux graphiques ont le même axe de symétrie.Si on choisit la même échelle sur l’axe des abscisses, lacourbe de densité englobe le diagramme en bâtons.b) On peut comparer en dressant un tableau :probabilité binomiale normaleP( X [ 45 ; 55 ] ) 0,7287 0,6827P( X [ 40 ; 60 ] ) 0,9648 0,9545P( X [ 35 ; 65 ] ) 0,9982 0,9973Les probabilités pour la loi binomiale sont plusgrandes.2. B ( 900 ; 0,5 ) et la loi normale µ = 450 et σ = 15probabilité binomiale normaleP( X [ 420 ; 480 ] ) 0,958 0,9545On peut aller plus loin et augmenter la taille n del’échantillon, en conservant p = 0,5 .Plus n est grand, plus lesprobabilités sont proches.Pour n = 10 000, on obtientpour la loi binomiale0,95449 (à faire sur lacalculatrice, car le logicielGeoGebra « plante »).Livre du professeur – Mathématiques Term STMG © Hachette Livre 2013 73


Même moyenneLe but est de comprendre le rôle de l’écart type et devisualiser son influence sur la courbe de densité. Celapeut permettre de comprendre que l’aire sous lacourbe est toujours égale à 1 .Il est nécessaire de fixer l’échelle de l’axe desordonnées comme indiqué sur le visuel. Pour cela onclique droit sur l’axe des ordonnées.1. a) et b) Sélectionner B4 et A5 et tirer vers le bas,jusqu’en ligne 24 .2. a) et b) Ci–après la courbe obtenue pour σ = 2 .La courbe en cloche monte plus haut et est plusétroite, afin que l’aire sous la courbe soit toujourségale à 1 .Demander aux élèves de construire la courbe pourd’autres valeurs de l’écart type : σ = 3 …Fixer l’échelle de 0 à 0,22 sur l’axe des y et regarderl’ordonnée à deux écarts types de part et d’autre del’espérance µ .2. b. « VRAI » dans la formule indique une probabilitécumulée, donc la réponse n’est pas a. On lit 0,0409 .3. c. On détermine les plus petits entiers a et b tels que :P( X ≤ a ) > 0,025 et P( X ≤ b ) ≥ 0,975 .•Or P( X ≤ 78 ) ≈ 0,0284 , donc a = 78et P( X ≤ 102 ) ≈ 0,9824 , donc b = 102 .D’où l’intervalle de fluctuation :[ 78; 102] = [ 0,52 ; 0,68 ] .150 150La fréquence du nombre d’élèves de BTS utilisant unordinateur est entre 52 % et 68 % , avec une probabilité de0,95 .4. a. X suit une loi normale d’espérance 135 et d’écart typeσ = 15 : P( 130 ≤ X ≤ 140 ) ≈ 0,261 .On ne peut que répondre en effectuant le calcul à lacalculatrice.5. a. Par symétrie de la courbe de densité par rapport à135.150 = 135 + 15 et 120 + 15 = 135P( X ≥ 150 ) = P (X ≤ 120 ) = 0,159 .6. b. D’après l’énoncé, la moyenne du temps defonctionnement donne l’espérance de la loi normale :µ=950 .On sait que [ 750 ; 1150 ] est l’intervalle de fluctuation de lavariable à 95 % . Or son amplitude est 4σ .D’où l’écart type σ =1150 −7504= 100 .Corrigés des exercices du chapitre 5ex1 a) P( X ≤ 180 ) ≈ 0,534 .b) P( X > 180 ) ≈ 1 – 0,534 ≈ 0,466 .c) P( X ≤ 185 ) ≈ 0,907.Associer courbe de densité et loi normaleSi l’atelier précédent a été traité, celui–ci est simple.Le but est de tester les qualités d’analyse des élèves.Faire représenter ces courbes sur tableur, ou sur lelogiciel GeoGebra.Tableau des résultatscourbe 1 2 3espérance 50 40 50écart type 5 5 10QCM page 129 pour répondre à un QCM p. 1951. a. On peut imaginer que le nombre d’élèves de BTS estassez grand pour que le choix de 150 élèves interrogéscorrespond à 150 tirages identiques. La variable aléatoiresuit la loi binomiale B ( 150 ; 0,6 ) d’espérance :µ = n × p = 150 × 0,6 = 90 .ex2 L’expérience aléatoire répétée 250 fois a deux issues.L’événement succès « le ménage de 60 ans et plus estconnecté à internet » a pour probabilité p = 0,37 .La variable aléatoire X associée au nombre de succès suit laloi binomiale B ( 250 ; 0,37 ).À la calculatrice, on obtient P( X = 100 ) ≈ 0,032 .Sur TIbinomFdp(Sur CasioMenu Statex3 a) Dans les instructions de la calculatrice, on remplace30 par 40 et 0,8 par 0,85 ..Par lecture des listes on obtient : a = 29 et b = 38 .D’où l’intervalle de fluctuation à 95 % :an ; b n=29; 3840 40= [ 0,725 ; 0,95 ] .Livre du professeur – Mathématiques Term STMG © Hachette Livre 2013 74


) Comme la fréquence observée f = 0,7 n’est pas dansl’intervalle trouvé à la question a) on rejette l’hypothèseque la proportion est p = 0,85 .c) De même si f = 0,65 , on rejette l’hypothèse p = 0,85 .ex4 a) P( 240 ≤ X ≤ 250 ) ≈ 0,477 .b) P( X ≤ 250 ) = 0,5 . Résultat immédiat puisque 250 est lavaleur de l’espérance de cette loi normale.c) P( X ≤ 260 ) ≈ 0,977 .La probabilité que le volume de produit d dans le flacon soitinférieur ou égal à 260 mL est 0,977 .d) P( X > 255 ) = 1 – P( X ≤ 255 ) ≈ 0,159 .La probabilité que le volume de produit dans le flacon soitsupérieur à 255 mL est 0,159 .e) P( 245 ≤ X ≤ 255 ) ≈ 0,683 .ex5 a) On utilise la calculatrice :P( X ≤ 6 ) ≈ 0,691 ex 30 p.133D’où la représentation sous la courbe de densité, courbe encloche centrée en µ = 5, avec peu de hauteur à 2 σ :5 – 2 × 2 = 1 et 5 + 2 × 2 = 9 .2. a) Listes pour un seul success : S–E–E ; E–S–E ; E–E–S .b) Probabilité de la liste E–S–E : 0,6 × 0,4 × 0,6 = 0,144 .c) P( X = 1 ) = 3 × 0,144 = 0,432 .ex9 QCM 1. Réponse b. 2. Réponse a.ex10 QCM 1. Réponse c. 2. Réponse b.ex11 n = 3 , p = 0,07 et 1 – p = 0,93 .1.a) 0,93 × 0,07 ² = 0,004557 .b) Sur l’arbre, on compte 3 chemins contenant deux succès.D’où P( X = 2 ) = 3 × 0,004557 = 0,013671 .2. P( X = 0 ) = 0,93 3 = 0,804357 .P( X ≥ 1 ) = 1 – P( X = 0 ) = 0,195643 .3. E(X) = n × p = 3 × 0,07 = 0,21 .Interprétation de l’espérance : si on recommence un trèsgrand nombre de fois l’expérience correspondant à ceschéma de Bernoulli, c’est-à-dire la répétition des troisexpériences à deux issues S et E, et que l’on note à chaquefois le nombre de succès S, le nombre moyen de succèsobtenu est 0,21 .ex12 1. Arbre pondéré complétéx-1 0 1 2 3 4 5 6 7 8 9 10b) P( 3 ≤ X ≤ 5,5 ) ≈ 0,44 .Conseil : pour la courbe de densité, dessiner un « chapeaude Napoléon », tracer l’axe de symétrie, et placerl’espérance, tracer les droites verticales à + ou – 2σ del’espérance , quand la courbe est basse, et graduer.ex6 a) On a I 2 = [ 2,7 ; 3,9 ] . Donc :µ =2,7+ 3,92= 3,3 et =3,9 − 2,74= 0,3 .b) P( 3,3 ≤ X ≤ 3,6 ) ≈ 0,341 . On peut l’obtenir par lacalculatrice, ou bien en utilisant la courbe en cloche :P( X [ µ ; µ + σ ] ) = 0,682= 0,341 .2ex7 a) On a I 2 = [ 152 ; 174 ] . Donc :µ = 152+174= 163 et = 174−15224b) P( X ≤ 163 ) = P( X ≤ µ ) = 0,5 .= 5,5 .ex8 1. a) La variable aléatoire X prend les valeurs del’ensemble { 0 ; 1 ; 2 ; 3 } .b) X suit une loi binomiale de paramètres n = 3 et p = 0,4 ,notée B ( 3 ; 0,4 ) .2. a) P( X = 3 ) = 0,8 3 = 0,512 .b) P( X = 2 ) = 3 × 0,8 ² × 0,2 = 0,384 . Donc la probabilitéd’obtenir exactement deux cartons verts est égale à 0,384 .3. P( X = 0 ) = 0,2 3 = 0,008 .D’où la probabilité de l’événement contraire : « obtenir aumoins un carton vert » égale à 1 – 0,008 = 0,992 .ex13 1. a) P( X = 0 ) = 0,7 4 = 0,2401 .b) Quatre chemins conduisent à l’événement { X = 1 } .Les listes correspondantes sont :S–E–E–E ; E–S–E–E ; E–E–S–E ; E–E–E–S .D’où P( X = 1 ) = 4 × 0,3 × 0,7 3 = 0,4116 .2. Loi de probabilité de la loi binomialek 0 1 2 3 4P( X = k ) 0,2401 0,4116 0,2646 0,0756 0,0081ex14 1. a) P( X = 3 ) = 0,3125 et P( X ≤ 3 ) ≈ 0,65625 .b) L’événement contraire de { X ≤ 3 } s’écrit { X > 3 } ou bien{ X ≥ 4 } , puisque k est un entier naturel.c) P( X ≤ 2 ) ≈ 0,34375 etP( X ≥ 4 ) = 1 – P( X ≤ 3 ) ≈ 1 – 0,65625 ≈ 0,34375 .Livre du professeur – Mathématiques Term STMG © Hachette Livre 2013 75


Les résultats sont identiques, il y a une symétrie du schémade Bernoulli de cette loi binomiale, car les probabilités desuccès et d’échecs sont égales.3. La probabilité P( X = k ) est la plus élevée pour k = 3.ex15 On utilise la loibinomiale B( 400 ; 0,33 ) .P( X = 130 ) ≈ 0,0416 .ex16 On utilise la loi binomiale B( 150 ; 0,8 ) .P( X = 125 ) ≈ 0,0505 .ex17 1. La variable X suit la loi binomiale B( 80 ; 0,5 ) .2. a) P( X ≤ 45 ) ≈ 0,891 .b) P( X > 45 ) = 1 – P( X ≤ 45 ) ≈ 0,109 .ex18 La variable X suit la loi binomiale B( 40 ; 0,5 ) .P( X = 21 ) ≈ 0,119 .ex19 QCM 1. Réponse b. 2. Réponse a.3. Réponse a. 4. Réponse b. 5. Réponse c.ex20 1. a. On détermine les plus petits entiers a et b telsque P( X ≤ a ) > 0,025 et P( X ≤ b ) ≥ 0,975 .On lit dans la colonne rouge de la table :a = 2 et b = 8 .2. b. D’où l’intervalle2; 810 10= [ 0,2 ; 0,8 ] .ex21 1. a) P( X ≤ 18 ) ≈ 1,5 × 10 – 5 .b) P( X ≤ 26 ) ≈ 0,35256 .2. a) « Au moins 3 élèves échouent au bac » signifie que« au plus 27 élèves réussissent au bac ».b) D’où la probabilité qu’au moins trois élèves échouent aubac : P( X ≤ 27 ) ≈ 0,58865 .3. a) a = 23 et b = 30 . b)23; 3030 30≈ * 0,766 ; 1 ] .ex22 a) f = 24= 0,8 .30b) Dans l’exercice 21 , on trouve a = 23 et b = 30 .Donc l’intervalle de fluctuation à 95 % de la fréquence est :I = [ 23; 30] .30 30La fréquence observée est dans cet intervalle : f I .Remarque : on peut aussi comparer 24 à l’intervalle[ a ; b ] = [ 23 ; 30 ] .ex23 1. = LOI.BINOMIALE (A2 ; 50 ; 0,65 ; VRAI)2. P( X ≤ 30 ) ≈ 0,2736 ; P( X ≤ 33 ) ≈ 0,6111 .P( X > 36 ) = 1 – P( X ≤ 36 ) ≈ 1 – 0,8837 ≈ 0,1163 .3. a) a = 26 et b = 39 .b)26; 3950 50= [ 0,52 ; 0,78 ] .4. La fréquence observée dans l’échantillon est :f = 3750 = 0,74 .Comme f = 0,74 appartient à l’intervalle [ 0,52 ; 0,78 ] ,intervalle de fluctuation à 95 % de la fréquence desménages parisiens qui possèdent une voiture, on ne rejettepas l’affirmation du journaliste.ex24 a) x ≈ 130,3 cm et ≈ 2 cm.b) Par son allure, la courbe 1 ne convient pas, car elle vacouper l’axe des abscisses.La courbe 3 n’a pas d’axe de symétrie, bien qu’elle soitplus proche des sommets des bâtons.Donc 2 est la courbe de densité de cette loi normale.ex25 1. x = 1200 € et ≈ 230 € .2. L’écart type est plus faible si l’on considère la répartitiondes salaires dans le groupe selon une loi normale.ex26 La courbe de densité de la loi normale d’espérance1200 et d’écart type 200 est 1 .En effet, on sait que :P( 1200 – 2 × 200 ≤ X ≤ 1200 + 2 × 200 ) ≈ 0,954 .C’est-à-dire P( 800 ≤ X ≤ 1600 ) ≈ 0,954 .Les deux courbes en bleu et en vert sont trop « hautes » auniveau de 800 et de 1 600 .En effet, à deux écarts types, la courbe de densité doit êtreproche de l’axe des abscisses et il doit rester moins de 5 %de la surface sous la courbe en cloche en dehors del’intervalle [ µ – 2σ ; µ + 2 σ + .ex27 Les réponses se lisent sur la table.1. a) P( X ≤ 250 ) = 0,5 . b) P( X ≤ 254 ) ≈ 0,8413 .c) P( X ≤ 247 ) ≈ 0,2266 .2. a) P( 247 ≤ X ≤ 250 ) = P( X ≤ 250 ) – P( X ≤ 247 )≈ 0,5 – 0,2266 ≈ 0,2734 .b) P( 248 ≤ X ≤ 254 ) = P( X ≤ 254 ) – P( X ≤ 248 )≈ 0,8413 – 0,3085 ≈ 0,5328 .ex28 La variable aléatoire X suit la loi normale d’espéranceµ = 100 et d’écart type = 9.1. a) P( X ≤ 102 ) ≈ 0,588 . b) P( 90 ≤ X ≤ 110 ) ≈ 0,733 .c) P( 98 ≤ X ≤ 105 ) ≈ 0,299 . d) P( X ≤ 95 ) ≈ 0,289 .2. a) P( X > 95 ) = 1 – P( X ≤ 95 ) ≈ 0,711 .b) P( X > 102 ) = 1 – P( X ≤ 102 ) ≈ 0,412 .ex29 Loi normale d’espérance µ = 60 et d’écart type = 4 .1. a) P( X ≤ 57 ) ≈ 0,227 . b) P( X ≤ 64 ) ≈ 0,841 .c) P( 57 ≤ X ≤ 63 ) ≈ 0,547 .2. P( X > 57 ) = 1 – P( X ≤ 57 ) ≈ 0,723P( X > 64 ) = 1 – P( X ≤ 64 ) ≈ 0,159 .ex30 Loi normale d’espérance µ = 3 et d’écart type = 1 .1. a) P( 0 ≤ X ≤ 8 ) ≈ 0,99865 . b) P( – 7 ≤X ≤ 8 ) ≈ 1 .c) P( – 100 ≤ X ≤ 8 ) ≈ 1 . d) P( – 10 9 ≤ X ≤ 8 ) ≈ 1 .Livre du professeur – Mathématiques Term STMG © Hachette Livre 2013 76


P( X ≤ 3,5 ) ≈ P( –2 ≤ X ≤ 3,5 ) ≈ 0,8413 .b) P( X ≤ 2,5 ) ≈ P( –2 ≤ X ≤ 2,5 ) ≈ 0,1587 .Coquille : lire 2 au lieu de 8 , à la calculatrice sur TI :2. On a P( X ≤ 2 ) ≈ 0,158655 et P( 0 ≤ X ≤ 2 ) ≈ 0,157305 .Si on prend a = 0 , on fait une approximation à 10 – 3 près .ex31 À la calculatrice , sur Casio :P( 50 ≤ X ≤ 70 ) ≈ 0,7887 , à 10 – 4 près.P( 52 ≤ X ≤ 68 ) ≈ 0,6827 , à 10 – 4 près.ex32 µ = 60 et = 1 .P( 59 ≤ X ≤ 61 ) ≈ 0,0995 et P( X > 47 ) ≈ 0,9479 .ex33 µ = 20 et = 1 .P( 18 ≤ X ≤ 22 ) ≈ 0,9545 et P( 20,5 ≤ X ≤ 22 ) ≈ 0,2858 .ex34 µ = 10 et = 3 .a) P( 5 ≤ X ≤ 10 ) ≈ 0,4522 et P( – 10 ≤ X ≤ 16 ) ≈ 0,9772 .b) P( – 15 ≤ X ≤ 13 ) ≈ 0,8413 et P( 0 ≤ X ≤ 13 ) ≈ 0,8409 .Vu la différence, on ne peut pas prendre 0 comme borneinférieure. P( X ≤ 13 ) ≈ 0,8413 .ex35 La loi normale d’espérance 0 et d’écart type 1 estaussi appelée la loi normale centrée réduite.À la calculatrice, à 10 – 4 près.a) P( – 2 ≤ X ≤ 2 ) ≈ 0,9545 et P( – 1,96 ≤ X ≤ 1,96 ) ≈ 0,9500.b) P( –20 ≤ X ≤ 2 ) ≈ 0,977249938P( –10 ≤ X ≤ 2 ) ≈0,977249938pas de différence visible.P( –5 ≤ X ≤ 2 ) ≈ 0,9772496509On peut donner :P( X ≤ 2 ) ≈ 0,97725 .ex37 Loi normale d’espérance µ = 11 et d’écart type = 5 .1. a) En A5 , on saisit = A4 + 1b) Formule corrigée en B4:=LOI.NORMALE ( A4 ; $A$2 ; $B$2 ; VRAI )2. a) P( X ≤ 10 ) ≈ 0,4207 .b) P( X ≤ 15 ) ≈ 0,7881 . Ce résultat est la probabilité que lacopie tirée au hasard ait une note inférieure ou égale à 15 .3. a) P( 8 ≤ X ≤ 12 ) = P( X ≤ 12 ) – P( X ≤ 8 )≈ 0,5793 – 0,2743 ≈ 0,305 .b) La probabilité que l’un des candidats ayant passé cetteépreuve ait une note comprise entre 8 et 12 est 0,305 .ex38 Loi normale d’espérance µ = 60 et d’écart type = 4 .a) Par complément de l’aire sous la courbe :P( X > 64 ) = 1 – P( X ≤ 64) ≈ 1 – 0,841 ≈ 0,159 .b) P( X ≤ 60 ) = 0,5 , car µ = 60 .c) P( 56 ≤ X ≤ 64 ) = P( X ≤ 64 ) – P( X ≤ 56 )Or par symétrie par rapport à la valeur centrale 60, on a :P( X ≤ 56 ) = P( X ≥ 64 ) et de plus P( X ≥64 ) = P( X > 64 )pour une loi normale. Donc :P( 56 ≤ X ≤ 64 ) = P( X ≤ 64 ) – P( X > 64 )≈ 0,841 – 0,159 ≈ 0,682 .d) P( 60 ≤ X ≤ 64 ) = P( X ≤ 64 ) – P( X ≤ 60 )≈ 0,841 – 0,5 ≈ 0,341 .ex39 X suit loi normale d’espérance µ = 10 et d’écart type = 1,5 . Donc P( X ≤ 10 ) = 0,5 . On a P( X ≤ 11 ) ≈ 0,748 .D’où P( 10 ≤ X ≤ 11 ) ≈ 0,748 – 0,5 ≈ 0,248 .ex40 Pour la loi normale d’espérance µ = 120 et d’écarttype = 10 , on a P( X ≤ 112) ≈ 0,212 .a) P( X > 112 ) ≈ 1 – 0,212 = 0,788 .b) P( 112 ≤ X ≤ 120 ) ≈ 0,5 – 0,212 ≈ 0,288 .c) P( 112 ≤ X ≤ 128 ) ≈ 2 × P( 112 ≤ X ≤ 120 ) ≈ 0,576 .d) Par symétrie par rapport à l’espérance µ = 120, on a :P( X > 128 ) = P( X ≤ 112 ) ≈ 0,212 .ex41 La hauteur de la boîte en carton suit la loi normaled’espérance µ = 5 cm et d’écart type = 0,2 cm .a) La probabilité P( X ≤ 5,3) est représentée en bleuex36 µ = 3 et = 0,5 .1. P( 2 ≤ X ≤ 3,5 ) ≈ 0,8186 et P( 0 ≤ X ≤ 3 ) ≈ 0,5 .2. a) Pour calculer P( X ≤ 3,5 ) , on utilise P( a ≤ X ≤ 3,5 )où a est éloigné de plus de 10 écarts types de l’espérance.On prendra a = 3 – 10 × 0,5 = – 2 .b) La probabilité P( 4,6 ≤X ≤ 5,4 ) est représentée en bleuLivre du professeur – Mathématiques Term STMG © Hachette Livre 2013 77


Cette espérance correspond à 11,3 ans.On peut espérer qu’un composant électronique de ce typea une durée de vie, en moyenne, estimée à 11,3 ans .b) P( 80 ≤ X ≤ 110 ) ≈ 0,517 .c) P( X ≤ 70 ) ≈ 0,084 (prendre a = – 10 9 , voir ex 30) .c) La probabilité P( X > 5,4 ) est représentée en bleuex46 Le poids de confiture par pot suit la loi normaled’espérance µ = 250 g et d’écart type = 10 g .a) P( X ≤ 260) ≈ 0,841 . La probabilité qu’un pot de confiturecontienne au plus 260 g de confiture est 0,841 .b) P( X > 275 ) ≈ 1 – P( X ≤ 275)≈ 1 – 0,9938 ≈ 0,0062 .La probabilité qu’un potdéborde et que la chaînes’arrête est 0,0062 .ex42 Loi normale d’espérance µ = 5 et d’écart type = 0,2.ex47 a) Intervalle [ µ – 2 ; µ + 2 ]a) P( X ≤ 5,3 ) ≈ 0,9332 . b) P( 4,6 ≤ X ≤ 5,4 ) ≈ 0,9545 .= [ 60 – 2 × 5 ; 60 + 2 × 5 ] = [ 50 ; 70 ] .c) P( X ≥ 5,4 ) = 1– P( X ≤ 5,4 ) ≈ 1 – 0,9772 ≈ 0,0228 .b) Courbe de densité et représentation de la probabilité :ex43 1. Comme M = 15 et S = 2,1 , la loi normale a pourparamètres l’espérance µ = 15 et l’écart type = 2,1 .On lit P( –3 ≤ X ≤16 ) ≈ 0,683 .2. a) P( 0 ≤ X ≤ 3,9) ≈ 0,7625 .ex48 La variable X suit la normale d’espérance µ = 10 etd’écart type = 1,8 .a) I 2 = [ 10 – 2 × 1,8 ; 10 + 2 × 1,8 ] = [ 6,4 ; 13,6 ] .b) P( 120 ≤ X ≤ 130) ≈ 0,1306 .b) Courbe de densité et représentation de la probabilité del’événement { X I 2 } .D’après le cours, P( X [ 6,4 ; 13,6 + ) ≈ 0,954 .c) P( – 3 ≤ X ≤3 ) ≈ 0,9973 .ex49 Plus l’écart type est grand, plus la courbe de densitéest « applatie » sur l’axe des abscisses.La courbe 1 est la courbe de densité de la loi normaled’écart type = 5. 2 est la courbe de densité de la loinormale d’écart type 15. 3 est la courbe de densité de laloi normale d’écart type 20 .Intervalle de fluctuation au seuil de 95 % pour chaque loi :ex44 Le temps mis par le joueur suit la loi normalea) I 2 = [ 60 ; 140 ] . b) I 2 = [ 70 ; 130 ] .d’espérance µ = 35 min et d’écart type = 6 min.c) I 2 = [ 90 ; 110 ] .a) P( 30 ≤ X ≤ 35 ) ≈ 0,2976 .La probabilité que le joueurex50 On a [ µ – 2 ; µ + 2 ] = [ 94 ; 166 ] donc :mette entre 30 et 35 minpour terrasser le dragon estµ = 94+166= 130 et = 166−94= 18 .240,298 ( à 0,001 près).b) P( X ≤ 25 ) ≈ 0,0479 .ex51 µ = 10h = 600 min et = 20 min .La probabilité que le joueura) I 2 = [ 560 ; 640 ] = [ 9 h 20 min ; 10 h 40 min ]mette moins de 25 min est 0,048 ( à 0,001 près).b) Environ 95 % des batteries de tablette tactile ont uneautonomie comprise entre 9 h 20 min et 10 h 40 min .ex45 Loi normale d’espérance µ = 99 milliers d’heures etd’écart type = 21 milliers d’heures.ex52 On s’intéresse à la loi normale d’espérance µ = 180 et99a) 1 année = 365×24 = 8,76 milliers d’heures et ≈ 11,3 . d’écart type = 23 .8,76Livre du professeur – Mathématiques Term STMG © Hachette Livre 2013 78


a) D’où l’intervalle I 2 = [ 134 ; 226 ] .b) Pour une semaine choisie au hasard pendant une année,95 % des demandes pour cette nourriture sont comprisesentre 134 kg et 226 kg.c) P( 160 ≤ X ≤ 200 ) ≈ 0,615 .Pour une semaine choisie au hasard , la probabilité que lademande soit comprise entre 160kg et 200 kg est 0,615 .ex53 X suit la loi normale d’espérance μ = 120 et d’écarttype σ = 15 et Y suit la loi normale d’espérance μ = 160 etd’écart type σ = 15 .a) P( 90 ≤ X ≤ 150 ) ≈ 0,954 .b) P( 130 ≤ y ≤ 190 ) ≈ 0,954 .c) On pouvait prévoir d’obtenir les mêmes résultats, cardans chaque cas il est demandé de calculer la probabilitéque la variable aléatoire appartienne à l’intervalle defluctuation [ µ – 2 ; µ + 2 ] .ex54 On a µ = 100 et = 5 .a) [ µ – 1,96 ; µ +1,96 ] = [ 90,2 ; 109,8 ] .b) P( X [ 90,2 ; 109,8 + ) ≈ 0,95 .ex55 On a µ = 1700 et = 100 .a) [ µ – 1,96 ; µ +1,96 ] = [ 1504 ; 1896 ] .b) P( X [ 1504 ; 1896 + ) ≈ 0,95 .ex56 On a I 2 = [ 680 ; 820 ] . On obtient :µ = 680+820= 750 et = 820−68024= 35 .ex57 Loi normale d’espérance 10 et d’écart type 2 .a) On obtient I 2 = [ 6 ; 14 ] . La probabilité que la variable Xappartienne à I 2 est l’aire de la surface bleue ci–dessous :b) Ci–dessous la représentation de la probabilité del’événement { X [ 10 ; 14] } :b) D’après le cours P( X [ 2,1 ; 4,9 ] ) ≈ 0,954 > 0,95 .D’après la symétrie de la courbe de densité, on en déduit :P( X > 4,9 ) < 1 − 0,95 = 0,025 .2ex60 Loi normale d’espérance 40 et d’écart type 2.1. L’abscisse du point C est 40 , celle de A est 40 – 4 = 36 etcelle de E est 40 + 4 = 44 . Or 36 = µ – 2 et 44 = µ + 2 .Cela montre que le segment [ A ; E ] représente l’intervalle[ µ – 2 ; µ + 2 ] , c’est-à-dire l’intervalle de fluctuation àenviron 95 % de la variable X .2. En utilisant les propriétés de symétrie de la courbe dedensité, on obtient :a) P( 36 ≤ X ≤ 40 ) ≈ 0,477 .b) P( X > 44 ) ≈ P( X > 40) – 0,477 = 0,5 – 0,477 ≈ 0,023.c) P( X ≤ 36 ) = P( X > 44 ) ≈ 0,023 .d) P( X [ 40 ; 44 + ) ≈ 0,477 .ex61 a) I 2 = [ 480 ; 520 ] . D’où = 520−480= 10 g .b) P( X ≤ 490 ) ≈ 0,159 .La probabilité qu’un ballotin de chocolat pris au hasardpèse moins de 490 g et soit rejeté est égale à 0,159 .ex62 1. a) La variable aléatoire X prend les valeurs entièresde 0 à 250 .2. a) E(X) =n × p = 250 × 0,02 = 5 .b) Sur un très grand nombre de jours, le nombre moyen depannes sera de 5 pannes par semaine. ex 11 p. 1303. a) P( X = 0 ) ≈ 0,0064 .b) P( X ≥ 1 ) = 1 – P( X = 0 ) ≈ 0,9936 .4. a) P( X = 4 ) ≈ 0,1765 .La probabilité, qu’au cours d’une semaine, 4 ascenseurstombent en panne est 0,1765 . exo 2. p. 119b) P( X < 4 ) = P( X ≤ 3 ) ≈ 0,2622 .4D’après le cours P( X [ 6 ; 14 + ) ≈ 0,954 .On en déduit que P( X [ 10 ; 14 + ) ≈ 0,9542≈ 0,477 .ex58 a) D’après le cours, P( 172 ≤ X ≤ 228 ) ≈ 0,954 .Dans 95 % des échantillons de 5000 animaux, on compteentre 172 et 228 animaux ayant contracté la maladietestée.b) De cet intervalle, on déduit :µ = 172+2282= 200 et = 228−1724= 14 .ex59 Le nombre de pommes de terre nécessaire pour 1 kgde chips Zweifel suit la loi normale d’espérance 3,5 kg etd’écart type 0,7 kg .a) L’intervalle de fluctuation est I 2 = [ 2,1 ; 4,9 ] .Conseil : Ne pas confondre les calculs sur la loi normale etloi binomiale : bien maîtriser les fonctionnalités de sacalculatrice.ex63 1. Loi binomiale B( 250 ; 0,02).1. n = 250 et p = 0,02 . On lit a = 1 et b = 10 . D’où :a; b = [ 0,004 ; 0,04 ] .n n2. Fréquence d’ascenseurs en panne observée par Yann-Loïc une semaine de décembre : f = 12= 0,048 .250Comme 0,048 [ 0,004 ; 0,04 ] , cette semaine n’est pas unéchantillon représentatif.ex64 1. a) La variable aléatoire X suit la loi binomiale :B( 100 ; 0,6 ) .Livre du professeur – Mathématiques Term STMG © Hachette Livre 2013 79


) E(X) = 100 × 0,6 = 60 . En considérant un très grandnombre d’échantillons de 100 personnes de cette ville, lenombre moyen d’adultes par échantillon sera égal à 60 .c) P( X = 60 ) ≈ 0,081 .2. a) En B2 on saisit =LOI.BINOMIALE(A2 ; 100 ;0,6 ;VRAI) .b) a = 50 et b = 69 . On obtient I = [ 0,5 : 0,69 ] .ex65 Les probabilités sont lues sur la table donnée. On peutcependant vérifier à la calculatrice.1. a) P( X ≤ 100 ) ≈ 0,43 .b) P(X < 96) = P( X ≤ 96 ) ≈ 0,0880 .2. a) P( 98 ≤ X ≤ 102 ) = P( X ≤ 102 ) – P( X ≤ 98 )≈ 0,6597 – 0,2222 ≈ 0,4375 .b) La probabilité qu’un pot de glace contienne entre 98g et102g de glace est 0,4375 . Sur Sinequanon :ex66 Partie A1. a) En D3 on saisit = D2 + C3 .b) En E2 , on saisit = D2/180 .2. a) 82,78 % des clients ont acheté des jouets pour unmontant inférieur à 240 € .b) 0,8278 – 0,2556 = 0,572257,22 % des clients ont dépensé entre 120 € et 240 € .3. Le montant moyen des achats est x = 170 € avec un écarttype ≈ 77,7 € .Partie B Loi normale d’espérance 170 et d’écart type 77,7 .1. P( X 240 ) ≈ 0,8162En décembre, la probabilité qu’un client pris au hasarddépense moins de 240 € est 0,8162 . Si le tirage au hasardd’un client dans l’ensemble des clients est équiprobable,alors on peut en conclure que 81,62 % des clients ontdépensé moins de 240 € . Le résultat obtenu dans la partieA 2. a) est proche de celui-ci.2. P( 120 X 240 ) ≈ 0,5562 .Selon ce modèle, la probabilité qu’un client pris au hasarddépense entre 120 € et 240 € est 0,5562. Autrement dit55,62 % des clients dépensent entre 120 € et 240 €. Lerésultat obtenu dans la partie A 2. b) est proche de celui-ci.ex67 Partie A1. En B4 : =LOI.NORMALE(15,5;17;1,5;VRAI)On a le même calcul si on saisit en B4 la formule :=LOI.NORMALE(A4;$A$2;$B$2;VRAI)2. On lit P( X 15,5 ) ≈ 0,1587. On en déduit :a) P( X 15,5 ) ≈ 1 – 0,1587 ≈ 0,8413.b) P( X [ 15,5 ; 17 ] ) = P( X 17 ) – P( X 15,5 )≈ 0,5 – 0,1587 ≈ 0,3413 .c) P( X [ 15,5 ; 18,5 ] ) = 2 × P( X [ 15,5 ; 17 ] )≈ 2 × 0,3413 ≈ 0,6826 .Partie B1. a) Age médian : Me = 17 ans.Intervalle interquartile : [ Q1 ; Q3 ] = [ 16 ; 18 ] .Écart interquartile : IQ = Q3 – Q1 = 2 ans.b) Au moins 50 % des spectateurs ont entre 16 et 18 ans.2. a) On obtient µ = Me = 17 ans et = 1,5 × IQ 2 = 1,5 .b) On utilise les résultats de la partie A.La probabilité qu’un spectateur pris au hasard ait moins de15,5 ans est 0,1587 .c) P( 16 X 18 ) ≈ 0,495 .Avec le modèle de la loi normale, 49,5 % des spectateursont entre 16 et 18 ans.Ce résultat est très proche de celui donné par la descriptionà l’aide du diagramme en boîte où l’on observe qu’environ50 % des spectateurs sont âgés de 16 à 18 ans.d) I 2 = [ µ – 2 ; µ + 2 ] = [ 14 ; 20 ] .On choisit au hasard un spectateur de ce concert. Laprobabilité qu’il ait entre 14 et 20 ans est environ 0,954 .ex68 Texte en Français1. Soit une variable aléatoire X qui suit une loi normaled’espérance µ et d’écart type σ . L’intervalle de fluctuationde cette loi est [ 8,5 ; 16,5 ] .a) En déduire les valeurs de µ et de σ .b) Dessiner la courbe de densité et représenter laprobabilité :P( X [ µ – σ ; µ + σ + ) .En donner une valeur arrondie à 10 –3 près.2. Les notes à un concours se répartissent suivant la loinormale définie en 1.a) Quelle est la moyenne des notes à ce concours ?b) Quelle est la probabilité qu’un candidat, pris au hasard,ait une note supérieure à 16,5 ?ex69 QCMPartie A1. Réponse c. 2. Réponse b. 3. Réponse a.Partie B1. Réponse a. 2. Réponse a.3. Réponse b. 4. Réponse a.Livre du professeur – Mathématiques Term STMG © Hachette Livre 2013 80


ex70 Partie A1.a) Valeurs de la série ( x i ; n i ) .centre de classe x i nombre de conserves n i237,5 32242,5 66247,5 217252,5 133257,5 52b) À la calculatrice x ≈ 248,6 et s ≈ 5,1 .Ainsi, une conserve a en moyenne une masse de 248,6 g ,+ ou – 5,1 g . ch1 ex. 11 p. 22c) 66 + 217 +133 = 416 et 416= 0,832 .50083,2 % de ces 500 conserves ont une masse comprise entre240 et 255 g.2. P(A) = 32+66+217500P(B) = 500−32500= 315= 0,630 .500= 0,936 .Partie B1. a) P(A) = 1 – P(A) = 1 – 0,63 = 0,37 .b) Arbre pondéré : schéma de Bernoulli à 3 niveaux, lesuccès étant l’événement A.Ainsi 25 % des personnes de la population étudiée ont unQI inférieur à 90 et 75 % des personnes ont un QI inférieurà 110.D’où l’intervalle interquartile : [ 90 ; 110 ] .Écart interquartile : IQ = 110 – 90 = 20 .2. Selon cette loi normale :a) P( X 100 ) ≈ 0,5 .b) P( 90 X 110 ) ≈ 0,495 .c) La probabilité pour qu’une personne ait un QI supérieurà celui de Madonna est P( X > 140 ) ≈ 0,0038 .3. 70 = µ – 2 et 130 = µ + 2 et d’après le cours :P( X [ 70 ; 130 + ) ≈ 0,954 .Par conséquent le psychologue a raison puisque selon cemodèle environ 95,4 % des personnes ont un QI comprisentre 70 et 130ex72 Partie A1. P( F1 ) = 0,75 et P F2 ( D ) = 0,02 .2. a) Arbre pondéré complet.b) P( F1 D ) = 0,75 × 0,01 = 0,0075et P( F2 D ) = 0,25 × 0,02 =0,005 .c) P(D) = P( F1 D ) + P( F2 D ) = 0,0125 .2. P( X = 3 ) = 0,63 3 ≈ 0,250 .La probabilité que les trois conserves, choisies au hasarddans l’échantillon de 500, pèsent moins de 250 g est 0,25 .b) L’arbre contient 3 chemins contenant une seule foisl’événement A.P( X = 1 ) = 3 × 0,63 × 0,37 ² ≈ 0,259 .Partie CLa variable Y suit la loi normale d’espérance µ = 249 etd’écart type σ = 5 .P( 240 ≤ Y ≤255 ) ≈ 0,849 .Ainsi la probabilité que lamasse d’une conserve soit comprise entre 240 g et 255 gest 0,849 .ex71 Partie A1. Réponse a. 2. Réponse b.Partie BLa variable X qui suit la loi normale d’espérance μ = 100 etd’écart type σ = 15 .1. On observe l’histogramme : 2,2 + 6,7 + 16,1 = 25 .Partie BLoi normale d’espérance µ = 22 h et d’écart type = 3 h .1. P( X 20 ) ≈ 0,252 .2. a) La probabilité que le temps d’installation d’un systèmesoit comprise entre 20 h et 22 h est environ 0,248.D’après la symétrie de la courbe de densité, on en déduit :P( 20 X 24 ) ≈ 2 × 0,248 ≈ 0,496et P( X 24 ) = P( X 20 ) ≈ 0,5 – 0,248 ≈ 0,252 .3. P( X [ µ – 2 ; µ + 2 ] ) = P( X [ 16 ; 28 + ≈ 0,954 .4. La probabilité que l’installation d’un système soitrentable est donnée par P( X 28 ) ≈ 0,977 .ex73 1.La variable aléatoire X suit une loi normaled’espérance 600 s et d’écart type 45 s .La probabilité qu’un exposé choisi au hasard soit jugé dedurée convenable est donnée par :P( 510 X 690 ) ≈ 0,954 ,car on reconnaît ici l’intervalle :[ µ – 2 ; µ + 2 ] = [ 510 ; 690 ] .2. La variable aléatoire X suit une loi normale d’espérance µet d’écart type .Livre du professeur – Mathématiques Term STMG © Hachette Livre 2013 81


On suppose que l’intervalle de fluctuation au seuil de 95 %des âges est l’intervalle [ 16 ; 52 ] = [ µ – 2 ; µ + 2 ] .On en déduit µ = 16+522= 34 et = 52−164= 9.On détermine la probabilité que l’âge d’un abonné, pris auhasard, s’écarte de l’espérance d’âge des 2/3 de l’écarttype : 2 3 = 6 ; 34 – 6 = 28 et 34 + 6 = 40 .Donc P( 28 X 40) ≈ 0,495 .Livre du professeur – Mathématiques Term STMG © Hachette Livre 2013 82


Intentions des auteures et conseilsBien que les suites soient en première partie duprogramme de maths, avant les fonctions, nous neconseillons pas de commencer l’année par cechapitre. Beaucoup d’élèves ont de mal sur cesnotions et cela risque de les rebuter. Ne pas hésiter àfaire un sondage en début d’année pour connaître lesparties qui les ont gênés en première.En première, les élèves ont vu les modes degénérations d’une suite et n’ont abordé que laformule de récurrence pour les suites arithmétiquesou géométriques. La nouveauté du programme determinale est l’expression du terme général pour lessuites arithmétiques et géométriques et lacomparaison de suites.Dans le cadre d’un enseignement en spirale, cechapitre peut-être traité en 3 ou 4 , voire 5 , parties :Cours 1 et 2 : après les premières parties deschapitres 1 , 2 et 3. Il est bon d’avoir revu les fonctionsaffines du chapitre 1 cours 2.Cours 3 : après avoir revu les pourcentagesd’évolution et le taux d’évolution moyen du chapitre2.Cours 4 : après avoir pratiqué un peu d’algorithme«simple» comme les calculs d’images.Cours 5 : en fin d’année, comme synthèse de cechapitre et révisions pour le Bac.• Un premier objectif est de consolider les notationssur les suites, ne pas confondre un terme et sonindice. Nous avons repris les modes de générationvues en première pour revoir la «traduction» d’unprocédé en formule de récurrence et calcul à l’aide dutableur, par recopie. On peut ainsi aborder des calculsfinanciers compliqués comme le tableaud’amortissement d’un prêt !• La mise en place des formules donnant le termegénéral d’une suite arithmétique ou géométrique enfonction de n est l’objectif principal de ce chapitre.Nous conseillons de laisser un temps entre les deuxtypes de suite pour ne pas ajouter de la confusionentre les deux formules.• Les formules donnant la somme de n termesconsécutifs d’une suite arithmétique ou géométriquen’est plus au programme de Term STMG. En revanche,l’unique indication ◊ de l’emploi de l’algorithmiquedu programme concerne le calcul de cette somme quidoit se faire à l’aide d’un tableur ou de la calculatrice.• Ce chapitre se prête bien aux calculs sur calculatriceet tableur, en particulier pour la comparaison desuites (salaires, placements, évolutions, modèles).Il ne suffit pas de savoir calculer les termes, mais avoirl’esprit critique devant les résultats obtenus.Pour les situations du cours, nous avons choisi dessuites croissantes, il est alors plus facile de mettre enplace les formules. Les élèves verront les suitesdécroissantes en exercices, quand les notions serontdéjà en place.Conformément aux commentaires du programme,nous avons insisté sur les calculs financiers,totalement adaptés à la série STMG : placementsdivers, à intérêts simples ou composés,remboursement d’emprunts…Nous conseillons les activités sur le site«lafinancepour tous.com» comme :http://www.lafinancepourtous.com/IMG/pdf/modulesmaths/1ere/Math_1ere_et_Tale_suites_geometriques.pdfhttp://www.lafinancepourtous.com/Espace–Enseignants/Mathematiques/2nde/Programme–de–PFEG–et–applications–mathematiques–possiblesAvant de commencerLes trois premiers QCM permettent une révision desconnaissances sur les suites concernant les notationset le mode de génération.Le QCM 4 peut introduire les suites géométriques, àpartir du coefficient multiplicateur global.Le QCM 5 introduit la modélisation.QCM 1 : Réponse b) en G2 on aura = 50 – 0.2 * F1donc 50 – 0.2 × 5 = 49 .a) Faux : Le 8 e terme est u 7 = 48,6 .c) Faux. Rappeler la proportionnalité : u n = k × n .Livre du professeur - Mathématiques Term STMG © Hachette Livre 2013 83


QCM 2 : Réponse c) Faire le calcul de la somme ouéliminer les autres réponses.Le mieux est de faire cet exercice en ouvrant unefeuille de calcul de tableur, ou demander aux élèvesd’entrer les valeurs dans une liste 2 de calculatrice etde demander la somme de la liste 2 : rabats de couverturea) Faux : c’est = SOMME(B2:J2)b) Faux : la suite commence en n = 1 .QCM 3 : Réponse c) Il est nécessaire d’écrire leprocédé : u n+1 = u n × 3 – 5 et u 0 = 10 .On peut vérifier : 5470 ×3 – 5 = 16405 . Mais il fauttrouver le terme 5470 dans la suite !Corriger les élèves qui traduisent avec des «égalités»incorrectes. En profiter pour revoir la signification dusymbole = . Insister sur : si A = B , alors B = A p. 214On peut prouver que les deux autres réponses sontfausses.a) Faux : vérifier à la calculatrice.On peut utiliser la touche ANS ou Rép .b) Faux : insister sur l’écriture en «indice» n + 1 .QCM 4 : Réponse b) Sur un an, CM = 1 – 0,02 = 0,98 .L’évolution se fait «pendant 10 ans», donc :CM global = CM 10 .a) Faux : la variation absolue de la population n’estpas constante.Revenir sur la variation absolue , dans la même unitéque la valeur et qui conduit à une suite arithmétiquelorsqu’elle est constante, et la variation relative quin’a pas d’unité, s’exprime souvent en pourcentage etconduit à une suite géométrique. p. 212c) Faux : serait vrai si la population perd 13 000habitants par an.CoursModes de génération d’une suiteÉtude d’une situationSeule situation non contextualisée, le but étant dereprendre le calcul sur tableur vu en première.Pour l’entier naturel n, nous conseillons de parler de« rang » et non d’indice.Nous n’avons pas insisté sur le lien entre suite etfonction de variable entière. Ce n’est pas nécessairepour le reste du chapitre. Quelques exercices font lelien en demandant des calculs de termes : voir lesexercices 13 , 20 , 21 et 22.Les suites démarrent au rang n = 0 ou au rang n = 1 .En première, dans l’introduction des suites, nousavons parlé de terme initial plutôt que de 1 er terme,peu naturel lorsque le rang initial est n = 0 . Mais lessujets de Bac parlent le plus souvent de 1 er terme.De même qu’une droite passant par A et B se noteavec des parenthèses (AB) , on note une suite ( u n )ayant pour terme général u n .Nous avons évité les notations plus lourdes ( u n ) n ≥ 0rencontrées il y a 10 ans et plus…1.a) 1 er terme de rang n = 0 et u 0 = 2 .b) D’après la formule en C2 =2 × B2 – 1 , on obtient :u 1 = 2 × u 0 – 1 .c) u 5 = 2 × u 4 – 1= 2 × 17 – 1 = 33 .Terminer en donnant u n+1 en fonction de u n :u n+1 = 2 × u n + 1formule de récurrence.2.a) En C5 , on a la formule = C4 * (C4 + 3) /2 ; d’oùv 1 = 1 × ( 1 + 3 ) / 2 = 2 .b) En F5, la formule devient = F4 * ( F4 + 3 ) /2 .c) v 5 = 5 × ( 5 + 3 ) / 2 = 20 .Terminer en donnant v n en fonction de n :v n = n × ( n + 3) / 2 =formule explicite.n ( n + 3 )2QCM 5 : Réponse c) On effectue les calculs :2408000 = 0,03 .a) Faux : 2012 est de rang 7 .b) Faux : on calcule l’accroissement d’un terme àl’autre : 240 € puis 247 € ; donc l’augmentation eneuros n’est pas constante.On peut vérifier que la valeur suit une suitegéométrique de raison 1,03 arrondi à l’unité.On peut utiliser ce QCM après les suitesarithmétiques, juste avant les suites géométriques.3.a) Le rang du 1 er terme est n = 0 .b) En F8 on obtient la formule = E8 + F7 .c) z 6 = z 5 + 6 = 16 + 6 = 22 .Terminer par l’expression de z n : z n = z n – 1 + n , autretype de formule.Objectif des exercices : modéliser un procédé decalcul itératif décrit par une phrase ou sous formed’algorithme en l’effectuant sur tableur, oucalculatrice. Traduire un procédé en une formule derécurrence. Introduire les notations et le vocabulaireemployés pour les suites.Livre du professeur - Mathématiques Term STMG © Hachette Livre 2013 84


Suite arithmétiqueÉtude d’une situationOn rappelle qu’une TPE est une très petiteentreprise :- le nombre de salariés doit être inférieur à 10 ;- le chiffre d’affaires doit être inférieur à deux millionsd’euros ;- le total bilan doit être inférieur à deux millionsd’euros.Elles représentent 94 % des entreprises en France en2011, dont 58 % n’ont aucun salarié. On parle aussi demicro-entreprise depuis 2008.La mise en place de la formule explicite d’une suitearithmétique est très facile. L’axe gradué est une aideimportante, en particulier pour comprendre le cas oùla suite commence par u 1 .On peut faire un axe allant vers la gauche pour le casoù on retranche toujours lemême nombre. Nous ne leconseillons pas.La notation r pour la raisonétait employée dans lesouvrages d’avant 1990. Leprogramme de maths deTerm ES dans les années 90 a introduit la lettre a pourfaire le lien avec la fonction affine. De plus la lettre rest très mal écrite par les élèves et se confond avec nou 2 pour certains d’entre eux.1. En B3, on saisit la formule = B2 + 1500 .2. a) 3 × 1500 = 4500 sont ajoutés au total.D’où le montant réservé : 10 000 + 4500 = 14 500 .b) 12 ajouts font au total 12 × 1500 = 18 000 .Soit un montant réservé de 10 000 + 18 000 = 28 000 .3. a) On effectue n ajouts, pour un total de n × 1500 .b) D’où le montant réservé de 10 000 + n × 1500 .4. On recherche n tel que :10 000 + 1500 n ≥ 31 000 .On peut le faire par tâtonnement, puis vérifier enrésolvant l’inéquation.La représentation des suites est reprise lors du cours 5sur la comparaison de suites.Objectifs des exercices : reconnaître une situation quise modélise par une suite arithmétique. Comprendrele placement à intérêts simples, indiqué dans leprogramme de 1 re . Rechercher un seuil par utilisationde la calculatrice ou la résolution d’une inéquation.Suite géométriqueÉtude d’une situationNous n’avons pas voulu multiplier les exemplesfinanciers. L’essentiel est de revenir sur le coefficientmultiplicateur vu dans le chapitre 2. La formule duterme général en fonction de n s’établit sansdifficulté.1. Réponses a) = B3+B3+B$1 ou d) = B3 * ( 1 + B$1 )2.a) CM = 1,1 . b) CM global = 1,1 3 = 1,331 .c) En 2013, n = 3 et la production est :500 × 1,1 3 = 665,5 ; donc de 665 objets.À placer sur un axe pour les élèves.3.a) CM global après n augmentations : 1,1 nb) Production en 2010 + n après n augmentations :500 × 1,1 n .Dessiner l’axe du cours, en notant les valeurs à laplace des lettres u 0 u 1 et q . Ne garder que nObjectifs des exercices : comprendre et assimilerqu’une évolution constante en pourcentage (variationrelative constante de t %) se modélise par une suitegéométrique de raison q = 1 + t/100 . Savoir calculer leterme d’une suite géométrique, connaître leplacement à intérêts composés.Rechercher un seuil à l’aide d’un algorithme ou de lacalculatrice.Somme de termes consécutifs d’une suiteÉtude d’une situationLes formules ne sont plus au programme de STMG.L’essentiel est de comprendre la nature de la suite etnombre de termes de la somme demandée.Cette partie permet de revenir sur les suites quellesqu’elles soient.1. La suite (u n ) est arithmétique de raison a = 10 et lasuite (v n ) est géométrique.Augmenter de 13%, c’est multiplier par 1,13 .2. Réponse a) = B5 + B$2 ou c) = B$5 + (A6–1)*B$23. En C6 on peut saisir = C5*C$2 ou =C$5*C$2^(A6–1)4. En B17 on peut saisir = somme(B5 : B16)5. a) En D5 on saisit = somme(B$5 : B5)On peut faire remarquer la formule = D5 + B6 quel’on peut utiliser en D6 , mais pas valable en D5.b) Comme D16 vaut 1100, alors D17 vaut :1100 + 160 = 1260 .Objectifs des exercices : travailler sur tableur etcalculatrice, et lire un algorithme.Nous nous sommes inspirés des sujets donnés au Bacen Term L depuis quelques années pour créer desexercices susceptibles d’être posés au Bac. Les tousderniers sujets de ES et S (novembre 2012) nous ontLivre du professeur - Mathématiques Term STMG © Hachette Livre 2013 85


eaucoup aidé pour la forme de l’algorithme et desquestionnements possibles.Il nous a semblé intéressant de donner le calcul de lasomme de n termes utilisant la somme cumulée,surtout si on veut faire des comparaisons de suites.On peut aussi entrer la suite en liste 1 , puis utiliserSomme(L1) ou Sum(List 1)Pour la production de Toupies sur les 6 mois :Sur Ti : suite( K , 100*1.05^K , 0 , 5 )Sur Casio : Seq( K , 100*1.05^K , 0 , 5 , 1 ) List 1Comparaison de suitesÉtude d’une situationCette partie de cours permet de revenir en find’année sur les suites et d’aborder les sujets de Bacclassiques. De plus, cela permet de visualiser lestermes d’une suite et de reprendre les lecturesgraphiques.1. Réponse a) = B5 + B$5 * B$2ou d) = B$5 * B$2 * A6 + B$5En profiter pour écrire le terme général de la suitedonnant le placement de Madeleine.2. En C6 pour Élise = C5 * (1 + C$2)3. Voir copie d’écran ci-dessous.4. Sur les années allant de 0 à 14 , le placement deMadeleine est le plus intéressant :voir le calcul de l’écart en colonne D.L1Atelier TICEStockage d’une productionBut : Créer un algorithme de calcul de la somme de ntermes consécutifs d’une suite. Le programme indiqueque c’est le seul algorithme que les élèves de STMGdoivent connaître. Nous avons présenté l’algorithmesur une suite arithmétique afin qu’il puisse être réaliséen début d’année si on le souhaite.Dans un premier temps, on vérifie « à la main » unprogramme en prenant une petite valeur de N. Ce quel’on peut demander au Bac.Puis on utilise ce programme pour un N plus grand.On trouvera en pages 236 à 239 la façon de le saisirsur AlgoBox ou calculatrice.Pour transférer un programmePour les élèves ayant des difficultés à saisir unprogramme, on peut le transférer d’une calculatrice àune autre de même type.Sur Ti, d’un 82.fr à une 82 ou d’une 83 84 à une 82 :touches• sur la calculatrice qui reçoit , choisir RECEPTIONet attendre…• sur la calculatrice où se trouve le programme,choisir ENVOI 3:Prgm…sélectionner le programme parpuis ENVOI 1:TransmissionSur Casio:• sur la calculatrice qui reçoit, choisir RECUet attendre• sur la calcultrice où se trouve le programme,choisir TRAN (F1) MAIN (F1) SEL (F1) puis PROGRAM,sélectionner le programme à transmettre,puis TRAN F6 OUI F1Objectifs des exercices : lectures de tableaux devaleurs sur calculatrice ou tableur pour comparer;utilisation d’algorithme pour déterminer un seuil.Pour changer une instruction dans un programmeIl faut faire attention, car on ne peut pas revenir enarrière et il n’y a pas de « corbeille ». Souvent lesprogrammes des élèves ne tournent pas parce qu’ilsont effacé une instruction.Livre du professeur - Mathématiques Term STMG © Hachette Livre 2013 86


Sur TI : entrer dans le programmeEDITSe placer sur l’instruction à changer :● ce que l’on tape remplace ce qui est écrit là où sesite le curseur ;● supprime le caractère (ou l’instruction) où sesitue le curseur ;● supprime la ligne complète ;● permet d’insérer ce que l’on tape auclavier (le curseur n’est plus qu’un trait horizontal enbas).Si on se trompe d’instruction, mieux vaut valider dansun premier temps l’instruction fausse, puis lasupprimer ensuite.Puis quitter le programme, et ainsi l’enregistrer :.A Suite arithmétique1. a) S vaut 0 , c’est l’initialisation de la somme.U vaut la valeur saisie au début: U = 50 .b) Au Bac, on demande de faire tourner e programmeen remplissant un tableau.Pour K = 2 , S vaut 110 et U vaut 70.c) La boucle POUR tourne 6 fois.d) Tableau complet des valeursK S : 0 U : 501 0+50 = 60 50+10 = 602 50+60 = 110 60+10 = 703 110+70=180 70+10=804 180+80=260 80+10=905 260+90=350 90+10=1006 350+100=450 100+10=110À la sortie, S vaut 450.2. On recommence avec U = 50 , A = 10 et N = 12 .On obtient S = 1 260 . Au total, Armel produit 1 260ceintures sur l’année de janvier à décembre.TICasioSur Casio :Se placer sur le nom duprogramme à changer et EDIT par F2 .Se placer sur l’instruction à changer :• ce que l’on tape s’insère à la position du curseur,• supprime le caractère AVANT le curseur ;• permet d’insérer les caractères que l’ontape au niveau du curseur.B. Suite géométrique1. u n = u 0 × 1,1 n = 3000 × 1,1 n .Suite géométrique de raison q = 1 + 10/100 = 1,1 .2.a) On change la ligne U prend la valeur U+A et onremplace par U prend la valeur U *A où A est la raisonde la suite géométrique.TICasioPuis quitter le programme et ainsi l’enregistrerPour exécuter un programmeSur TI :Sélectionner et lancer le programme en tapant le n°du programme (ici ) .Sur Casio :Sélectionner le programmeb) U = 3000 , A = 1,1 et N = 7 , car il y a 7 termes àcalculer pour les années de 2010 à 2016 comprises.3. En 2016, u 6 = 3000 × 1,1 6 ≈ 5314,68 .S = u 0 + u 1 + … + u 6 ≈ 28461,51TICasiopuis EXE par F1Prêt : étudeBut : réaliser un tableau d’amortissement et encomprendre le principe.Lorsque l’on connaît le montant de l’emprunt, le tauxet le nombre d’annuités, on peut ainsi obtenir partâtonnement une valeur approchée de l’annuité,Livre du professeur - Mathématiques Term STMG © Hachette Livre 2013 87


correspondant au montant du remboursement. Lesdifférents questionnements devraient permettre àl’élève de comprendre le principe de ce tableau.La formule de l’annuité est donnée. Nous l’avonstrouvé donnée ainsi dans un ouvrage de Gestion de1 ère STMG. Nous avons donné sa « traduction » en uneformule sur tableur : c’est un bon exemple d’emploides parenthèses pour effectuer un calcul.Nous avons choisi un taux important pour que lavaleur de l’annuité soit très éloignée de la valeur del’emprunt divisée par le nombre d’années.De plus, les élèves peuvent aller plus loin lorsqu’ilsauront vu le taux proportionnel (exercice 77).On peut alors faire un tableau avec un taux équivalentmensuel ou un taux proportionnel mensuel, et unremboursement mensuel (mensualité constante) soit72 mensualités. La formule reste la même.1. a) En B5, on saisit = B2 .b) En C5 on calcule 10000 × 12/100 = 1200donc on saisit = B5 * $C$2 .c) En D5 on obtient : amortissement de l’année= annuité constante – intérêt de l’annéed) En F5, on saisit = B5 – D5 et en B6 = F5 .2.a) L’annuité doit être supérieure, car il reste 18,32 €à payer.b) En E2 on essaie 2432,26 , il y a un résidu de – 0,02ce qui signifie que la dernière annuité sera 2432,24 € .3. On entre la formule et on trouve 2432,257184 .Comparaison de deux prêtsBut : utiliser le tableau d’amortissement réalisé àl’atelier 2 pour la comparaison de deux emprunts.Cela permet de voir le coût total pour deux façons derembourser plus ou moins long. En général, les prêts àcourt terme ont un taux plus faibles, mais ilsconduisent à des annuités plus grandes.1. Tableau pour le prêt de Zyhan2. Tableau pour le prêt de Kezyah:3. Le coût total est de 6×1020 = 6120 pour Zyhan et 8×850 = 6800 pour Kezyah, plus cher, mais moindre par mois.On peut aller plus loin en incluant les frais. Faire étudier par les élèves une offre de prêt chez Cofinoga ou Cételem.QCM page 159 pour répondre à un QCM p. 1951. c. D’une année à l’autre, le nombre de salariés diminuede 5 % , donc est multiplié par 0,95 ; puis augmente de 20:donc u n+1 = 0,95 u n + 20 .2. b. On ne doit pas «bloquer» le nom de la colonne B .3. a. La suite ( u n ) est arithmétique, de premier termeu 0 = 90 de raison a = – 7,5 . D’où u n = u 0 + n × a = 90 – 7,5n .4. b. u 12 = 90 – 7,5 × 12 = 0 .Livre du professeur - Mathématiques Term STMG © Hachette Livre 2013 88


On peut regarder le tableau de valeurs de f(x) = 90 – 7,5 x ,pour tous les entiers x à partir de x = 0 .5. c. Chaque année, à partir de 2010, le revenu est multipliépar 1,03 . La suite ( v n ) est géométrique, de premier termev 0 = 38 400 et de raison q = 1,03 .D’où v n = v 0 × q n = 38400 × 1,03 n .6. a. S = v 0 + … + v 4 ≈ 203871 > 2000007. c. On a u 7 = 360 et v 7 ≈ 353 : u 7 > v 7 .et u 8 = 390 et v 8 ≈ 399 : u 8 < v 8 .Il est nécessaire de visualiser lesvaleurs de ces deux suites surcalculatrice.Ces deux suites sont croissantes.Corrigés des exercices du chapitre 6ex1 v 6 = 80 × 6 + 400 = 880et u 6 = ( 710 – 195 ) × 2 = 1 030 .Au bout de 6 mois, Elora possède 880 € et Matéo 1 030 € .ex2 a) On a u 0 = 400et u n + 1 = ( u n – 205 ) × 2 .b) u 5 = 90 .À la fin du 5 e mois, il reste 90 € .ex3 a) On a u 1 = 3 × u 0 – 2 = 10 etu 2 = 3 × u 1 – 2 = 28 .b) POUR K allant de 1 à Ncalculer 3 × U – 2et stocker dans UFin de la boucle POURc) Tableau . Sur AlgoBox , on peut suivre pas à pas.K 1 2 3 4U avant calcul 4 10 28 82U après calcul 10 28 82 244ex4 a) 8 000 × 0,07 = 560 .Chaque année le capital augmente de 560 € .b) On note C 0 = 8 000 et C n le capital acquis au bout de nannées. La suite (C n ) est une suite arithmétique, de raisona = 560 et de terme initial C 0 = 8 000 .D’où : C n = C 0 + n × a = 8 000 + 560 n .c) C 10 = 13 600 € .ex7 1. a) Les intérêts acquis les 6 premiers mois sont :10 000 × 1,5/100 = 150 € . D’où C 1 = 10 150 € .b) C n + 1 = C n + 0,015 × C n = 1, 015 C n .c) En cellule B4 , on écrit la formule = B3 × 1,015 .2. a) D’après la formule de récurrence, la suite ( C n ) estgéométrique, de raison q = 1,015 avec C 0 =10 000 .D’où l’expression du capital acquis C n en fonction de n :C n = C 0 × q n = 10 000 × 1,015 n .b) En cellule B4 , on écrit la formule = B$3 * 1,015 ^ A43. La durée de 8 ans correspond à 16 semestres.C 16 = 10 000 × 1,015 16 ≈ 12 690 € .ex8 a) b) Voir résultats à la calculatrice :Le 6 e mois, la production est de 146 toupies et laproduction totale du 1 er au 6 e mois est de 796 toupies.ex9 p. 153 On note u n la production le n e mois. Ainsi la suite( u n ) est arithmétique, de premier terme u 1 = 100 et deraison a = 20 .a) u 6 = u 1 + 5 × a = 100 + 5 × 20 = 200 .b) S = u 1 + u 2 +…+ u 6=100 + 120 + 140 + 160 + 180 + 200 = 900 .En 6 mois, 900 toupies sont produites.ex10 a n = a 0 × q n = 20 000 × 1,05 n .b n = b 0 × q n = 19 000 × 1,06 n .On obtient les termes des deux suites à la calculatrice :a 5 ≈ 25 526 et b 5 ≈ 25 426 ; a 6 ≈ 28 142 et b 6 ≈ 26 952 .Donc jusqu’en n = 5 le contrat A est plus intéressant et àpartir de n = 6, le contrat B est le plus intéressant.Johann a intérêt à prendre le contrat A s’il reste jusqu’à 9ans dans l’établissement et a intérêt à prendre le contrat Bs’il reste 10 ans dans cette société.ex5 a) Chaque mois Rachida retire la même somme 300 € .La suite ( u n ) est arithmétique , de premier terme u 0 = 5000et de raison a = – 300 . D’où u n = u 0 + n a = 5000 – 300 n .b) u 7 = 5000 – 300 × 7 = 2900 € .ex6 On note u 1 = 13 et u n le nombre de tableaux vendus len-ième mois de l’année.La suite ( u n ) est une suitearithmétique de raison a = 5et de terme initial u 1 = 13 .D’où u n = u 1 + ( n – 1 ) aet u 5 = 13 + 4 × 5 = 33 .En mai, Eric a vendu 33tableaux.ex11 • La première production est modélisée par la suitegéométrique ( u n ) de premier terme u 0 = 100 et de raisonq = 1,06 , où u n désigne la production le n-ième mois suivantle mois de départ.Livre du professeur - Mathématiques Term STMG © Hachette Livre 2013 89


D’où u n = 100 × 1,06 n .• La deuxième production est modélisée par la suitegéométrique ( v n ) de premier terme v 0 = 200 et de raisonq = 0,98 , où v n désigne la production le n-ième moissuivant le mois de départ.D’où v n = 200 × 0,98 n .• À la calculatrice : f(X) =100 × 1,06^X – 200 × 0,98^X .On lit f(8) négatif , donc u 8 < v 8et f(9) positif , donc u 9 > v 9 .Donc à partir du 9 e mois, lapremière production dépasse ladeuxième.ex12 a) u 7 = 3 ; u 10 = 1 et u 22 = 9 .b) Le premier chiffre 3 est de rang 7 et le premier chiffre 8est de rang 20 .c) Pour n = 46 , on a u 46 = 8 et u 47 = 4 .ex 13 a) u 3 = 12 et u 5 = 28 .b) u 10 = 10 ² + 3 = 103 . u 20 = 20 ² + 3 = 403 .ex14 a) u 2 = 37 .b) 325 est le terme de rang 4.c) u 6 = 2 917 .ex15 1. Réponse b. Pour n = 0 , (0×5+10)/2 = 5 .Pour n = 1 , (1×5+10)/2 = 7,5 .Pour n = 2 , (2×5+10)/2 = 10 .2. Réponse c.n × 5+102.ex16 1. u 1 = 3,8 ; u 2 = 4,6 et u 3 = 5,4 .2. Formule à placer en B2 : = ( A2 + 10 ) * 0,8 – 5 .3. La différence entre deux termes consécutifs est 0,8 .ex17 On part de 9 : (9 + 3)/2 = 6 ;puis (6 + 3)/2 =4,5 ; puis (4,5 + 3)/2 = 3,75 .La formule de récurrence de cette suite est :u n +1 = u n + 3, avec u20 = 9 .ex18 100 ; 110 ; 121 et 133,1 .ex19 a) Tableau des valeurs pas à pas :K 1 2 3 4 5U avant calcul 10 30 70 150 310U après 30 70 150 310 630Pour N = 3 , l’algorithme renvoie la valeur U = 150et pour N = 5, U = 630 .b) Pour N = 6 on obtient U =1270 .Pour N = 7 , U = 2550 , supérieur à 2500. Algorithme p. 236 à 239ex20 u n = 2 n ² + 3n – 10 . D’où :u 1 = – 5 ; u 5 =55 et u 20 = 850 .ex21 u n = 240 – 8n . D’où :u 2 = 224 ; u 10 =160 et u 30 = 0 .ex22 u n = 500 × 1,2 n . D’où :u 1 = 600 ; u 2 =720 et u 4 = 1036,8 .ex23 On a u 0 = 20 et u n + 1 = 5 u n – 2 .n 1 2 3 4u n 98 488 2438 12 188ex24 v 1 = 240 ; v 2 = 144 ; v 3 = 86,4 et v 4 = 51,84 .ex25 w 1 = 115 ; w 2 = 128,5 ; w 3 = 140,65 et w 4 = 151,585 .ex26 a) u 0 = 1200 . b) u n + 1 = u n + 70 .c) u 1 = 1270 ; u 2 =1340 et u 3 = 1410 .ex27 a) u 0 = 25 000 . b) u n + 1 = 0,92 u n .c) u 1 = 23 000 ; u 2 =21 160 et u 3 = 19 467,2 .ex28 a) u 0 = 100 000 . b) u n +1 = 1,05 u n – 3 000 .c) u 1 = 102 000 ; u 2 = 104 100 et u 3 = 106 305 .ex29 a) u 1 = 689 ; u 2 = 679 et u 3 = 670 .b) Algorithme obtenu à l’aide d’AlgoBox :c) Réponse de l’algorithmeLivre du professeur - Mathématiques Term STMG © Hachette Livre 2013 90


Au bout de 10 ans, l’effectif de l’entreprise sera d’environ622 personnes.ex30 1. a) Faux : le 3 e terme est u 2 . b) Vrai .c) Faux : u n + 1 = u n + 18 . Attention à l’écriture des indices.2. a) Vrai . b) Vrai . c) Faux : u 9 = 322 .ex31 1. Réponse c. On a la forme u n +1 = u n – 35 , donc lasuite est arithmétique de raison a = – 35 .2. Réponse b. La formule explicite est u n = u 0 + n a ; donc :u n = 400 + n × ( – 35 ) .3. Réponse b. On calcule les termes de la suite :u 11 = 15 positif et u 12 = – 20 négatifet comme a = – 35, la suite arithmétique est décroissante.ex32 a) u n +1 = u n + 8 et u 0 = 12 . b) u n = u 0 + n a = 12 + 8 n .c) u 9 = 84 et u 95 = 12 + 8 × 95 = 772 .ex33 a) u n + 1 = u n – 7 . b) u n = u 0 + n a = 140 – 7 n .c) u 9 = 77 et u 95 = – 525 .ex34 a) u n + 1 = u n + 1,5 . b) u n = u 0 + n a = 3,8 + 1,5 n .c) u 9 = 17,3 et u 95 = 146,3 .ex35 ( v n ) suite arithmétique de raison a = – 50 et determe initial v 1 = 1 200 .1. a) v 2 = 1 150 ; v 3 =1 100 et v 4 = 1 050 .b) v n + 1 = v n – 50 .c) v n = v 1 + ( n – 1 ) a = 1200 – 50 ( n – 1 ) .2. a) Cette suite arithmétique est décroissante.b) À la calculatrice, on obtient v 25 = 0 .c) On résout l’équation :1200 – 50 ( n – 1 ) = 0 50 ( n – 1 ) = 1200 n – 1 = 1200 n – 1 = 24 n = 25 .50ex36 On a u 8 = 392 et u 14 = 536 .a) La différence des rangs est 14 – 8 = 6 .b) La différence des deux termes est 536 – 392 = 144 .c) Pour passer du terme u 8 au terme u 14 , on ajoute 6 fois laraison a. Donc a = 1446 = 24 .ex37 a) La raison de la suite ( u n ) est a = 2 .b) u n = u 1 + ( n – 1 ) a = 3 + ( n – 1 ) × 2 = 2 n + 1 .Les termes de la suite (u n ) sont les ordonnées des points dela droite d’équation y = 2x + 1 , ayant des abscissesentières de 1 à n .c) ( 15 ; 31 ) . Ce point est sur la droite .ex38 a) Pour n , u n = 170 – 14,5 n .On reconnaît la forme u n = b + n a , donc la suite est la suitearithmétique, de premier terme u 0 = 170 – 14,5 × 0 = 170 etde raison a = – 14,5 .b) À l’aide de la calculatrice :On résout l’inéquation 170 – 14,5 n < 0 où n est un entier :– 14,5 n < – 170 14,5 n > 170 n > 170Or 17014,5≈ 11, 7 , donc n ≥ 12 .14,5 .ex39 a) Comme la ville augmente tous les ans de 580habitants, la suite ( p n ) est arithmétique de premier termep 0 = 16 000 et de raison a = 580 .D’où p n = p 0 + n×a = 16 000 + 580 × n .b) 2013 = 2000 + 13 , donc n = 13. On calcule p 13 = 23 540 .Selon ce modèle, on prévoit 23 540 habitants en 2013 danscette ville côtière.c) p 6 = 19 480 < 20 000 et p 7 = 20 060 > 20 000 .Donc la population devient supérieure à 20 000 habitantsen 2007.ex40 a) Intérêts perçu la première année :5 000 × 0,025 = 125 € .b) Soit C 0 le capital initial : C 0 = 5 000.On note C n le capital acquis en 2000 + n .Comme le placement est à intérêts simples, les intérêtsannuels supplémentaires se montent à 125 € .La suite ( C n ) est arithmétique de raison a = 125 .Ainsi C n = C 0 + n a = 5 000 + n × 125 .En 2013, n = 13 et C 13 = 6 625 € . Mathilda disposera de6 625 € sur son compte le 2 janvier 2013 .ex41 a) Soit t le taux inconnu, exprimé en pourcentage.Intérêts annuels : Int = t× 6400 = 64 t .100b) Chaque année le montant des intérêts obtenus estidentique, d’où :4 × 64 t = 960 t = 960= 3,75 .256Le capital de 6 400 € est placé à intérêts simples au taux de3,75 % .ex42 Le taux t de ce placement, exprimé en pourcentagevérifie l’équation :5 × 4 300 × t= 5 590 t = 26 .100Soit un taux de 26 % .Si 5 590 est le capital acquis, alors le taux est de 6 % .ex43 1. a) 8 000 × 0,06 = 480 .Chaque mois Selma retire 480 € de son capital.b) u 1 = 7 520 € et u 2 = 7 040 € .c) Comme Selma retire chaque mois le même montant de480 € , la suite ( u n ) est arithmétique de raison a = – 480 etde terme initial u 0 = 8 000 .D’où u n = u 0 + n × a = 8 000 – n × 480 .Livre du professeur - Mathématiques Term STMG © Hachette Livre 2013 91


2. Tableau des termes de la suiteSelma a effectué 16 retraits lorsque le montant du compteest 320 € .ex44 1. a) Julien ajoute chaque mois 55 € : E n + 1 = E n + 55 .b) La suite ( E n ) est arithmétique, de premier terme E 1 = 930et de raison a = 55 . On en déduit :E n = E 1 + ( n – 1 ) × a = 930 + 55 ( n – 1 ) = 55 n + 875 .2. En juillet, n = 7 . E 7 = 930 + 55 × 6 = 1 260 .Quand il aura été reçu au Bac, Julien aura 1 260 €d’économies.ex45 1. a) Faux : q = 4 . b) Vrai. c) Vrai.2. a) Vrai. b) Faux : u n = u 0 × q n = 12 × 4 n . c) Vrai.ex46 1. Réponse c. On a u n +1 = 0,8 u n ; donc on reconnaît laformule de récurrence u n +1 = q × u n , avec q = 0,8 .2. Réponse c. Comme la suite est géométrique et u 0 = 1000,alors u n = u 0 × q n = 1000×0,8 n .3. Réponse b. u 10 = 1000 × 0,8 10 ≈ 107 et u 11 ≈ 86 .ex47 a) u 0 = 10 et u n +1 = 1,15 u n .b) u n = u 0 × q n = 10 × 1,15 n .c) u 3 = 10 × 1,15 3 ≈ 15,21 et u 6 = 10 × 1,15 6 ≈ 23,13 .ex48 a) u n + 1 = 0,2 u n . b) u n = u 0 × q n = 500 × 0,2 n .c) u 3 = 4 et u 6 = 0,032 .ex49 a) u n + 1 = 2 u n . b) u n = u 0 × q n = 0,25 × 2 n .c) u 3 = 2 et u 6 = 16 .ex50 a) u n + 1 = 1,02 u n . b) u n = u 1 × q n – 1 = 100 × 1,02 n – 1 .c) u 3 = 104,04 et u 6 ≈ 110,4 .ex54 1. a) u 0 = 100 ; u 1 = 95 et u 2 = 90,25 .b) u n + 1 = 0,95 u n .c) La suite ( u n ) est géométrique de raison q = 0,95 et determe initial u 0 = 100 .On en déduit : u n = u 0 × q n = 100 × 0,95 n .2. Trois formules conviennent : b) c) et d) .3. a) u 5 ≈ 77,4 .b) 77,4 – 100 = –22,6 .Cet indice a baissé de 22,6 % en 5 mois.4. u 13 ≈ 51,3 > 50 et u 14 ≈ 48,8 < 50 .Le 14 ième mois, l’indice devient inférieur à 50 .ex55 1. a) Diminuer du quart donne un coefficientmultiplicateur : CM = 1 – 1 4 = 3 4 = 0,75 .Donc u n +1 = 0,75 u n et u 0 = 15 000 .b) La suite ( u n ) est géométrique, de raison q = 0,75 et depremier terme donné u 0 = 15 000 .c) u n = u 0 × q n = 15 000 × 0,75 n .2. a) u 4 = 15 000 × 0,75 4 ≈ 4746soit une valeur de 4 746 € , à 1 € près.L’appareil a encore de la valeur.b) La suite est décroissante, car q = 0,75 ] 0 ; 1 [ .On a u 5 ≈ 3560 € et u 6 ≈ 2670 € .À la fin de la 6 e année, la valeur de l’appareil passe sous leseuil de 3 000 € . On peut remarquer que, même si la valeurdiminue, celle-ci reste cependant toujours positive.ex56 1. a) La suite ( u n ) est géométrique de raison q = 1,05et de terme initial u 0 .b) En fonction de n, u n = u 0 × q n = u 0 × 1,05 n .2. De 2008 à 2014, il y a 6 évolutions de 5 % .2014 = 2008 + 6 , donc n = 6 .Et comme on connaît u 6 = 134, on en déduit :134 = u 0 × 1,05 6 u 0 = 1341,05 6 ≈ 100 .En 2008, l’indice avait pour valeur 100, à 1 unité près.ex57 1. Réponse c. 2. Réponse b. 3. Réponse b.4. Réponse c. 5. Réponse a.ex51 a) u n + 1 = 0,92 u n .b) u n = u 1 × q n – 1 = 2000 × 0,92 n – 1 .c) u 3 = 1692,8 et u 6 ≈ 1318,2 .ex52 a) u n + 1 = u n + 12100 u n = u n ( 1 + 0,12 ) = 1,12 u n .On reconnaît la suite géométrique de raison q = 1,12et u 1 = 200 donné.b) u n = u 1 × q n – 1 = 200 × 1,12 n – 1 . Attention à l’exposantu 4 = 200 × 1,12 3 .ex53 1. q = 1,1 . Donc u n = u 0 × q n = 200 × 1,1 net u 10 ≈ 518 objets en 2020.2. q = 1,04 . Donc u n = u 0 × q n = 100 × 1,04 net u 10 ≈ 148 indice en 2020.3. q = 0,965 . Donc u n = 12 500 × 0,965 n et u 10 ≈ 8754 € .4. q = 0,85 . Donc u n = 540 000 × 0,85 net u 10 ≈ 106 312 visiteurs en 2020.5. q = 0,995 . Donc u n = 100 × 0,995 net u 10 ≈ 95 Indice en 2020.ex58 1. a) Comme le cabinet prévoit une augmentation de2 % par an de la clientèle, la suite ( u n ) est géométrique deraison q = 1,02 et de terme initial u 0 = 1 700 .b) u n = u 0 × q n = 1 700 × 1,02 n .c) 2018 = 2010 + 8 , donc n = 8 .Et u 8 ≈ 1 992 , arrondi à l’unité .Le cabinet d’avocats peut prévoir 1 992 clients en 2018.2. Les formules qui conviennent sont c) , d) et f) .Livre du professeur - Mathématiques Term STMG © Hachette Livre 2013 92


3. D’après le tableur (ou la calculatrice) à partir de 2019, lenombre de clients sera supérieur à 2 000 .ex59 1. On a, sur 2011 : 36−4545= – 0,2et sur 2012 : 28,8−3636= – 0,2 .L’investissement a diminué de 20 % en 2011 et en 2012.2. Soit u n le montant de l’investissement en 2010 + n, on a :u n +1 = u n – 20100 u n = 0,8 u n et u 0 = 45 .Donc la suite est géométrique de raison q = 0,8 et depremier terme u 0 = 45 donné.D’où l’expression du terme général :u n = u 0 × q n = 45 × 0,8 n .Ainsi u 4 = 45 × 0,8 4 = 18,432 .3. a) La suite est décroissante, car la raison q est compriseentre 0 et 1 .On a u 6 = 45 × 0,8 6 ≈ 11,796 > 10et u 7 = 45 × 0,8 7 ≈ 9,437 < 10 .Donc l’année de rang 7, soit en 2017, l’investissement passesous la barre des 10 000 €.b) 2020 est l’année de rang n = 10 . u 10 = 45 × 0,8 10 ≈ 4,832 .Donc en 2020, il est prévu 4 832 € d’investissement.ex60 Partie A 1. u 1 = 11 040 € et u n + 1 = 0,92 u n .2. La suite ( u n ) est géométrique de raison q = 1 – 8100 = 0,92et de terme initial u 0 = 12 000 .D’où u n = u 0 × q n = 12 000 × 0,92 n . Et u 6 ≈ 7 276 € .Partie B 1. Réponse b)À la place du cache, on écrit : U * 0,92 .2. a) Comme u 6 ≈ 7 276 , inférieur à 8 000, et que la suiteest décroissante, on peut affirmer que l’entier N obtenu ensortie de cet algorithme est inférieur à 6 .b) On peut écrire ce programme à la calculatrice pourobtenir N ou bien tabuler la fonction définie par :Y = 12 000 × 0,92 X , par pas de 1, en commençant à 0 ,Ainsi, après 5 années de fonctionnement, les dépenses duservice seront inférieures à 8 000 €.ex61 1. a) Pour tout entier naturel n , on a :u n + 1 = 2 + ( n + 1 ) = ( 2 + n ) + 1 = u n + 1 .Donc la suite ( u n ) est arithmétique de raison a = 1 .b) u 0 = 2 ; u 1 = 3 ; u 2 = 4 et u 3 = 5 .La suite ( u n ) est la suite des entiers naturels à partir de 2 .2. S = u 0 + u 1 + … + u 5 = 2 + 3 + 4 + 5 + 6 + 7 = 27 .Cette somme comporte 6 termes.ex62 a) u 5 = 100×1,1 5 ≈ 161,05 .b) S = u 1 + u 2 +…+ u 4 = 510,51 .c) S’ =u 0 + u 1 +…+ u 5 = 771,56 .ex63 1. a) Raison q = 1,3 .b) v 2 = 33,8 .2. a) En B3 on saisit : = B2 * 1,3 .b) En cellule B9 on saisit := SOMME( B2 : B8 ) .ex64 1. a) u 2 = 370 et u 3 = 340 .b) La suite ( u n ) est arithmétiquede raison a = – 30et de terme initial u 1 = 400 .c) D’où u n = u 1 + ( n – 1 ) a= 400 – 30 ( n – 1 ) .d) Les nombres d’appareils fabriquésen juin puis en décembre sont donnéspar les termes u 6 et u 12 .u 6 = 250 et u 12 = 70 .2. La totalité des appareils fabriquésdans l’année est donnée par la sommeS’ = u 1 + u 2 + … + u 12 .3. a) En B3 on saisit : = B3 – 30 .b) En cellule B15 on saisit := SOMME( B3 : B14 ) .Le fabricant de matériel électronique afabriqué 2 820 appareils dans l’année.ex65 Sur l’année, 2820 appareils sont fabriqués.ex66 1. u n = u 1 × q n – 1 = 100 × 0,8 n – 1 .u 5 = 100 × 0,8 4 = 40,96 .2. a) S 4 = u 1 + u 2 + u 3 + u 4 = 295,2 .b) S 7 = 395,14 < 400 et S 8 = S 7 + u 8 .Or u 8 ≈ 100 × 0,8 7 ≈ 20,97 .D’où S 8 ≈ 395,14 + 20,97 ≈ 416,11 .Ainsi la somme S n = u 1 + u 2 + … + u n devient supérieure à400 pour n = 8 .On peut proposer aux élèves de chercher n pour que lasomme devienne supérieure à 450, puis à 490, puis à 500.ex67 1. a) La suite ( u n ) est géométrique de raison q = 1,2 etde terme initial u 0 = 4 .b) u n = u 0 × q n = 4 × 1,2 n .c) u 6 ≈ 11,94 . Selon ce modèle, en 2016, on estime que lataille du dossier contenant les messages électroniquesprofessionnels sera de 11,94 Go.2. En C3, on saisit la formule : = C2 * 1,2 .3. En D3, on peut saisir = D2 + C3 ou = SOMME($C$2 : C3) .4. a) Au 31 décembre 2016, la taille de l’ensemble desdossiers est 51,66 Go.b) La capacité de stockage est dépassée.ex68 1. a) La suite ( u n ) est géométrique de raison q = 1,09et de terme initial u 1 = 1 000 .b) u n = u 1 × q n – 1 = 1 000 × 1,09 n – 1 .Livre du professeur - Mathématiques Term STMG © Hachette Livre 2013 93


2. En C3 , on saisit la formule : = C2 * 1,09 et en D3 , onsaisit la formule : = D2 + C3 .La commande de 9 000 paires de lunettes pourra être livréeen juillet, car on dispose alors d’un stock de 9 200 paires.ex69 1. a) S 4 = 1 + 2 + 3 + 4 = 10 .b) 2 × S 4 représente le nombre total de carreaux roses etverts représentés sur la figure de l’énoncé.La figure comporte 4 lignes formées de 5 carreaux , d’où :2 × S 4 = 4 × 5 = 4 × ( 4 + 1 ) .2. En imaginant une figure analogue avec 10 lignes, chaqueligne aura en tout 11 carreaux. Donc on obtient :10 × 112 × S 10 = 10 × ( 10 + 1 ) S 10 = = 55qui représente la somme des 10 premiers entiers de 1 à 10 .3. On peut alors conjecturer que :S n = 1 + 2 + 3 + … + n =n n + 1en imaginant n lignes de n + 1 carreaux composées de 2empilages l’un rose, l’autre vert chacun de 1 + 2 + 3 + … + ncarreaux.ex70 1. a) La suite ( u n ) est arithmétique avec a = – 15 000et la suite ( v n ) est géométrique de raison q = 0,8 .Pour les deux suites, le terme initial est 140 000 .b) Pour tout entier n :u n = u 0 + n × a = 140 000 – 15 000 net v n = v 0 × q n = 140 000 × 0,8 n .2. a) En diminuant la production de 20 % par mois, au boutde 5 mois, la diminution de production est plus importantequ’avec une baisse de 15 000 stylos chaque mois.On justifie ce résultat en observant sur le graphique que :v 5 < u 5 .b) Le terme u n ne devient inférieur au terme v n qu’à partirdu 8 e mois.Les termes de la suite (v n ) diminuent de moins en moinsvite et tous les termes de cette suite restent positifs. Parcontre les termes de la suite ( u n ) diminuent régulièrementde 15 000 unités. Ce modèle ne convient plus à partir durang n = 10 , où les termes de la suite deviennent négatifs.Erreur de texte dans l’édition 01 : dans l’énoncé, lire« inférieur ».ex71 Le terme général de chacune des suites géométriquesest donné, pour tout n , par :u n = 1 200 × 1,025 n et v n = 1 000 × 1,04 n .Sur l’écran donné, on observe en colonne Y1 les termessuccessifs de la suite ( u n ) à partir de u 9 et dans la colonneY2, les termes successifs de la suite ( v n ) à partir de v 9 .On lit : v 12 < u 12 et v 13 < u 13 .22.Ainsi, le plus petit entier n tel que v n > u n est n = 13 .ex72 1. a) B 1 = 1 900 + 400 = 2 300et C 1 = 1 900 × 1,15 = 2 185 .b) La suite ( B n ) est arithmétique de raison a = 400 .Pour n ventes réalisées :B n = B 0 + na = 1 900 + 400 n .c) C n + 1 = 1,15 C n .La suite (C n ) est géométrique de raison q = 1,15 .On en déduit l’expression de C n , en fonction de n , pour nventes réalisées : C n = C 0 × q n = 1900 × 1,15 n .2. a) En cellule B4 , on saisit = B3 + 400 .b) En cellule C4 , on saisit = C3 * 1,15 .3. Pour 6 ventes réalisées dans le mois, le salaire selon lecontrat C est de 4 395 € et pour 7 ventes réalisées dans lemois, le salaire selon le contrat B est de 4 700 €.4. Avec au moins 6 ventes par mois, le contrat C est plusintéressant.Pour moins de 6 ventes dans le mois, c’est le contrat B leplus intéressant.ex73 1. Chaque année, le placement U augmente de12 × 30 = 360 € et le placement V est multiplié par 1,06 .1 an 2 ansvaleur U en € 4360 4720valeur V en € 4240 4494,402 a) La ligne incomplète devient :affecter U + 12 × 30 à Ub) Tableau pas à pas :étape N de U de V U – V ≥ 0 ?0 0 4000 4000 oui1 1 4360 4240 oui2 2 4720 4494 oui3. Cet algorithme, réalisé à l’aide d’AlgoBox (p. 236–237),montre que le placement U devient moins intéressant quele placement V au bout de 14 ans.On résume les calculs dans un tableau.n u n v n test13 8680 8531,71 u 13 > v 1314 9040 9043,62 u 14 < v 14Programme sur TIsur Casioex74 1. La suite ( u n ) est arithmétique de raison a = – 35 etde terme initial u 1 = 500 . u 12 = u 1 – 11 × 35 = 115 .Selon la proposition 1, la 12 e mensualité est 115 € .2. La suite ( v n ) est géométrique de raison q = 0,9 et determe initial v 1 = 500 . v 12 = v 1 × 0,9 11 ≈ 156,91 .Selon la proposition 2, la 12 e mensualité est 156,91 €.3. Pour déterminer la proposition la plus avantageuse pourVincent il faut comparer les deux sommes :S = u 1 + u 2 + … + u 12 et S’ = v 1 + v 2 + … + v 12 .Livre du professeur - Mathématiques Term STMG © Hachette Livre 2013 94


Sur TI : On réalise une table de valeurs par pas de 1, encommençant à X = 1 pour lesdeux fonctions données cicontre,à l’aide des listes.En liste 1 les entiers de 1 à 12en liste 2 : Y1(L1) L2en liste 3 : Y2(L1) L3Pour Y1: voir rabats decouverture.b) Le capital acquis au bout de 4 années de placement estdonc 8 396,88 €. Tableau des valeursY–VARS 1:Fonction… 1:Y1Sur Casio: on réalise les listes suivantes :List 1 : entiers de 1 à 12List 2 : 500 – (List 1 – 1 ) *35List 3 : 500*0.9^(List 1 – 1 )Pour chacune des fonctions,on additionne les 12 valeursobtenues à l’aide de lacalculatrice par :somme(L2) et somme(L3) .Pour la proposition 1, Vincent rembourse 3 690 € autotal et pour la proposition 2, il rembourse 3 587,85 € autotal. La proposition 2 est la plus intéressante.ex76 1. Tableau des résultatsannée n population urbaine population rurale0 5 400 10 000u11 = 5 400 + 500 = v 1 = 10 000 – 300 =5 9009 700u22 = 5 900 + 500 = v 2 = 9 700 – 300 =6 4009 4002. a) Pour chaque suite, on passe d’un terme au suivant enajoutant toujours un même nombre. Ici négatif pour la suitede la population rurale (v n ) .On reconnaît donc la définition d’une suite arithmétique.La suite ( u n ) est arithmétique de raison a = 500 et la suite(v n ) est arithmétique de raison – 300 .b) u n = u 0 + n a = 5 400 + 500 n et v n = 10 000 – 300 n .3. a) En B3 : = B2 + 500 et en C3 : = C2 – 300 .b) Résultats obtenus sur tableurÀ l’aide d’un tableur :En B3 , on saisit la formule : = B2 – 35 .En C3 , on saisit la formule : = C2 * 0,9 .Au bout de 6 ans, la population de la ville dépassera lapopulation de la zone rurale.ex75 1. a) C 1 = 3 000 × 1,03 + 1 200 = 4 290 .C 2 = 4 290 × 1,03 + 1 200 = 5 618,7 .b) Soit C n le capital , en euros, après n années deplacement. L’année suivante, ce capital augmente de 3 %, ilest donc multiplié par 1,03 puis on lui ajoute la somme fixede 1 200 € . D’où C n + 1 = 1,03 C n + 1 200 .2. a) Formule à entrer en B5 qui, par recopie vers le bas,permet de compléter la plage de cellules B5:B8 := $B$2 + B4 * ( 1 + $B$1/100 ) .ex77 1. Taux annuelMontant des intérêts du prêt : 1 000 × 0,18 = 180 €.Victoire doit rembourser 1 180 € à la fin de l’année.2. Taux équivalenta) Le taux annuel de 18 % est associé à un coefficientmultiplicateur global CM global = 1,18 .Le coefficient multiplicateur mensuel correspondant à cetteévolution est la solution de l’équation CM 12 = 1,18d’où CM = 1,18 112 ≈ 1,013888 et (CM – 1) × 100 ≈ 1,3888 .Donc le taux mensuel moyen équivalent au taux annuel de18 % est 1,3888 % .b) Pour 1 000 € empruntés, le montant des intérêts au boutd’un mois est 1000 × 1,3888≈ 13,89 € .100c) Chaque mois le capital restant dû augmente de 1,3888 %.1000 × 1,013888 12 = 1000 × 1,18 12 = 1 180 € .Livre du professeur - Mathématiques Term STMG © Hachette Livre 2013 95


Au bout de 12 mois, on retrouve la même somme àrembourser en appliquant le taux mensuel équivalent ou enappliquant le taux annuel.3. Taux proportionnela) Avec un taux proportionnel T p = 18= 1,5 12soit 1,5 % par mois. Les intérêts au bout d’un mois, sur uncapital de 1 000 € s’élèvent à 1 000 × 0,015 = 15 € .Pour 1 000 € empruntés, il faut donc rembourser 1 015 € aubout d’un mois. Ce capital est ensuite soumis à un tauxmensuel de 1,5 % .b) Chaque mois, le capital restant dû augmente de 1,5 % .Donc au bout de 12 mois, après 12 augmentations :12≈ 1 195,62 .1000 × 1 + 1,5100En appliquant le taux proportionnel chaque mois au capital,Victoire doit rembourser 1 195,62 € au bout d’un an.Cette somme à rembourser est supérieure à celle obtenueen 1. et 2.c) Le taux réel annuel de ce crédit, ou taux actuariel, est de19,562 % . Le taux de 18 % est le taux nominal.Si on intègre les frais de dossier et l’assurance, on obtient letaux effectif global TEG.ex78 1. a) La suite ( u n ) a pour raison a = 6 et pour premierterme u 0 = 50 .b) S 11 = u 0 + u 1 + … + u 10 = 880 .2. a) b) c) u n = u 0 + n a = 50 + 6 n .À la calculatrice, on complète les listes L1, L2 et L3 de lafaçon suivante.On retrouve bien S 11 = u 0 + u 1 + … + u 10 = 880 .3. Par lecture de la liste3 :S 5 = u 0 + u 1 + … + u 4 = 310 et S 7 = u 0 + u 1 + … + u 6 = 476Erreur dans l’énoncé : lire S 7 = u 0 + u 1 + … + u 6 .4. S = 11 × u 0+ u 102= 11 ×50 + 1102= 880 .ex79 1. • Le premier versement est placé pendant 6 ans etau bout des 6 ans, le capital acquis correspondant est :u 1 = 1 000 × 1,05 6 ≈ 1 340,10 € .• Le deuxième versement est placé pendant 5 ans.A la fin de la période le capital provenant de ce deuxièmeplacement est u 2 = 1 000 × 1,05 5 ≈ 1 276,28 € .2. a) u n = 100 × 1,05 7 – n .b) Les valeurs des 6 versements aubout des 6 ans sont données dans lafeuille de calcul.En B2 , on a écrit la formule := 1000 * 1,05 ^ ( 7 – A2 )3. Le capital acquis au bout des 6 ans par les 6 versementsde 1000 € est :S 6 = 7 142 € , à 1 € prèsvaleur obtenue sur tableur, sans arrondir chaque valeurannuelle.4. a) S = 1000 × 1,05 × 1−1,056≈ 7 142,01 .1−1,05b) Au centime près, S = S 6 .Cette formule peut être donnée dans un sujet de Bac pourla somme de n termes d’une suite géométrique, de premierterme connu U 1 et de raison q connue:S = U 1 + U 2 + … + U n = U 1 ×1 − qn1 − q .ex 80 1. Comme le capital est placé à intérêts composé autaux annuel de 4 % , ce capital augmente chaque année de4 % , ainsi chaque année il est multiplié par 1,04 .Pour obtenir 100 000 € au bout de 8 ans, il faut placer uncapital C tel que :C × 1,04 8 100 000= 100 000 C =1,04 8≈ 73 069,02 € .2. a) Rente annuelle d’Audrey : 12 × 500 = 6 000 € .b) Intérêts annuels dus par Michel :100 000 × 0,06 = 6 000 € .Les intérêts dus par Michel correspondent à la renteannuelle d’Audrey.3. Soit t le taux mensuel proportionnel du prêt de Michel :t = 6= 0,5 .12a) 100 000 1 + t 120,512= 100 000 1 + = 106 167,78 .100100b) Le résultat de la question 3. a) représente la valeur ducapital du par Michel au bout d’une année d’emprunt.Ainsi il doit 6 167,78 € d’intérêts.c) En assurant la rente de 500 € par mois à Audrey,l’organisme financier a gagné 167,78 € la première année.Cette différence est liée au calcul fait au taux mensuelproportionnel (taux réel appliqué) et non au taux mensueléquivalent (voir ex 77).ex81 Texte en françaisSoit (u n ) la suite de terme général u n = 100 × 0,95 n , pourtout entier n ≥ 1 .1. a) Calculer le premier terme u 1 .b) Quelle est la nature de cette suite ( u n ) ?2. Sur tableur, quelle formule saisir en cellule B2 afin decalculer les termes de la suite, par recopie vers le bas ?3. Calculer les termes manquants en cellule B11 et B15 .4. Quelle formule écrire en cellule C2 pour calculer lasomme S n des n premiers termes ?ex82 QCMPartie A 1. Réponse a. 2. Réponse b.3. Réponse b. 4. Réponse b.Partie B1. Réponse c. 2. Réponse b. 3. Réponse a.ex83Partie A1.• Marque Citroën :CM = 1 265≈ 0,8809 et ( 0,8809 – 1 ) × 100 = – 11,91 .1 436Livre du professeur - Mathématiques Term STMG © Hachette Livre 2013 96


Entre les années 2011 et 2012, les ventes de voitures demarque Citroën ont baissé de 11,91 % (à 0,01 % près).• Marque Peugeot :CM = 1 700≈ 0,8042 et ( 0,8042 – 1 ) × 100 = –19,58 .2 114Entre les années 2011 et 2012, les ventes de voitures demarque Peugeot ont baissé de 19,58 % (à 0,01 % près).2. Pour le groupe PSA , le nombre de voitures venduesentre 2011 et 2012 est passé de 3 550 milliers à 2 965milliers.CM = 2 965≈ 0,8352 et ( 0,8352 – 1 ) × 100 = – 16,48 .3 550Ainsi, pour le groupe PSA, de 2011 à 2012, les ventes devoitures neuves ont diminué de 16,5 % (à 0,1 % près).Partie BLa suite ( u n ) est géométrique de raison q = 0,881 et determe initial u 0 = 1 265 .1. a) u 1 ≈ 1114 et u 2 ≈ 982 .b) u 3 ≈ 865 et u 4 ≈ 762 . L’année de rang 4 est l’année 2016.Ainsi en 2016, le nombre des ventes sera pour la premièrefois inférieur à 800 000 voitures.2. a) u n = u 0 × q n = 1265 × 0,881 n .b) 2018 est l’année de rang 6 :1 265 × 0,881 6 ≈ 591 .Selon ce modèle, la marque Citroën peut prévoir de vendre591 milliers de voitures dans le monde en 2018.3. D’après la question 1. on peut modéliser le nombre deventes chez Peugeot, à partir de 2012, à l’aide d’une suitegéométrique ( v n ) de raison q = 0,804 .On réalise la feuille de calcul ci–dessous avec en C4, laformule = C3 * 0,881 et en D4, la formule = D3 * 0,804 .On en conclut qu’en 2016, selon ce modèle, les ventesmondiales de Citroën vont dépasser celles de Peugeot.ex84 Partie A1. Nuage de points : bien respecter les graduations.y1420140013801360134013201300128012601240-1 0 1Gx2 3 4 5 6 7 8 9 10 112. a) Point moyen G ( 4 ; 1 329 ) .b) Équation de la droite de régression :y = 22,46 x + 1 239,29.c) La droite est tracée sur le graphique en la faisant passerpar le point G et le point de coordonnées ( 0 ; 1 239 ) .3. a) 2016 est l’année de rang x = 11 .22,46 × 11 + 1 239,29 = 1 486,35 .Selon ce modèle d’ajustement affine, le SMIC mensuel en2016 est estimé à environ 1 486 € .b) 22,46 × 12 + 1 239,29 = 1 508,81 .Selon ce modèle, le SMIC mensuel dépassera 1 500 € en2017.4. De 2009 à 2012, le SMIC mensuel est passé de 1 338 € à1 398 € : CM global = 1 3981 338 ≈ 1,0448 .De 2009 à 2012, il y a trois évolutions successives.D’où le coefficient multiplicateur du taux annuel moyend’évolution :CM ≈ 1,0448 1 3 ≈ 1,0147 et ( 1,0147 – 1 ) × 100 = 1,47 ≈ 1,5 .On justifie ainsi que le taux moyen d’évolution du SMICmensuel entre 2009 et 2012 est d’environ 1,5 % .Partie B1. La suite ( u n ) est géométrique de raison q = 1,015 et determe initial u 0 = 1 398 .2. Réponse a.3. Estimation du SMIC en 2015 : 1 462 € .Estimation du SMIC en 2017 : 1 506 € .4. Les deux modèles d’estimation du SMIC donnent desrésultats proches pour l’année 2016.En effet, par l’ajustement affine, on obtient 1 486 € et àl’aide de la suite géométrique, on obtient 1 484 €.ex85 1. a) C n + 1 = C n ( 1 + 5) = C n × 1,05 .100b) La suite ( C n ) est géométrique de raison q = 1,05 et determe initial C 0 = 125 000 . D’où :C n = C 0 × q n = 125 000 × 1,05 n .2. De la même façon, B n = 30 000 × 1,071 n .3. a) P 0 = B 030 000=C 0125 000 = 0,24 . P 1 = B 130 000 × 1,071=C 1125 000 × 1,05 = 0,2448 .b) P n + 1 = B n +1C n +1= 1,071 × B n1,05 × C n= 1,0711,05 × B nC n= 1,02 P n .c) Donc la part que représente le bénéfice par rapport auchiffre d’affaires évolue selon une suite géométrique deraison q = 1,02 et de premier terme P 0 = 0,24 .On remarque que la part augmente donc de 2 % par an.P n = 0,24 × 1,02 n .d) Comme P 10 = 0,24 × 1,02 10 ≈ 0,293 , au bout de 10 ans, lebénéfice représente 29 % du chiffre d’affaires.ex86 Partie 1Malthus fait l’hypothèse : u 0 = 20 000et u 1 = 20 000 + 1100× 20 000 = 20 200 .2. La population augmente chaque année de 1 %, donc lasuite ( u n ) est géométrique, de raison q = 1,01 et depremier terme u 0 = 20 000. D’où :u n = u 0 × q n = 20 000 × 1,01 n .Selon ce modèle, l’année 1900 est l’année de rang 100 .u 100 = 20 000 × 1,01 100 ≈ 54 096 .La population en 1900 est donc estimée à 54 millions.Livre du professeur - Mathématiques Term STMG © Hachette Livre 2013 97


Partie 21.Malthus fait l’hypothèse v 0 = 25 000 et une augmentationdes ressources pour 10 milliers de personnes :v 1 = 25 000 + 10 = 25 010 .2. La suite ( v n ) est arithmétique, de raison a = 10et v 0 = 25 000 . D’où v n = v 0 + na = 25 000 + 10 × n .3. v 100 = 25 000 + 10 × 100 = 26 000 .Selon ce modèle en 1900, l’agriculture peut nourrir 26millions de personnes. On remarque que les ressourcesprévues dans ce pays sont très insuffisantes pour nourrir lapopulation attendue de 54 millions en 1 900 .Partie 31. En C3 = C2 × 1,01 2. En D3 = D2 + 103. On obtient les valeurs :Amortissement :64 752,29 – 18 734,05 = 46 018,24 € .Capital restant dû :374 680,91 – 46 018,24 = 328 662,67 € .2. a) En C3, on saisit = E2 * 0,05 .En C4, cette formule devient : = E3 * 0,05 .b) En E3, on saisit = E2 – D3 .Partie B1. Tableau d’amortissement4. Le mot « risque » va apparaître la première fois pour leplus petit entier n tel que v n < u n , c’est-à-dire lorsque lenombre d’habitants que le pays peut nourrir devientinférieur à la population de ce pays.Le mot « risque » apparaît à partir de la cellule E26, pourl’année 1824.b) Malthus parle du risque de famine puisque la productionagricole sera insuffisante pour nourrir la population.ex87Partie A1. a) En C3, on calcule les intérêts payés en 2006 :500 000 × 5= 25 000 € .100En B3, on calcule le capital remboursé en 2006 :64 752,29 – 25 000 = 39 752,29 € .b) On complète la ligne 6, concernant l’année 2009.Intérêts versés :374 680,91 × 0,05 ≈ 18 734,05 € .2. La seule suite géométrique est la suite ( a n ) . A 10 – 5 :41739 .939752 ,29 ≈ 1,049999887 ≈ 1,05 etDonc a n + 1 = 1,05 a n .Alors que pour les intérêtsi 2 23012 ,39≈i 1 25000≈ 0,9204 etet pour le capital restant dû :c 2≈ 0,9093 etc 143826 .941739 .9i 3 20925 ,39≈i 2≈ 1,05000012 ≈ 1,05 .23012 ,39 ≈ 0,9093 .c 3c 2≈ 0,8953 .3. Comme le prêt est remboursé en 10 ans, la somme desamortissements a 1 + a 2 + a 3 +… a 10 est le capital remboursésur ces 10 ans vaut 500 000 € .4. Montant total des intérêts payés par l’entrepreneur :10 × 64 752,29 – 500 000 = 147 522,9 € .Livre du professeur - Mathématiques Term STMG © Hachette Livre 2013 98


Intentions des auteures et conseilsCe chapitre clos les connaissances à acquérir enTerminale sur les probabilités.Bien que nécessitant de connaître la loi normale pourcomprendre l’intervalle de fluctuation de la fréquenceà 95 %, on peut traiter ce chapitre sans insister sur laloi normale.Dans le cadre d’un enseignement en spirale, cechapitre peut-être traité en 2 parties :Cours 1 : l’introduction de la loi normale et l’intervallede fluctuation d’une variable aléatoire suivant une loinormale doit être traitée avant, mais il est souhaitabled’étudier d’autres parties du programme et de laisserdu temps entre la fin du ch5 et le début du ch7.Cours 2 : après être revenu sur l’étude de fonctions,les suites … pour laisser du temps, après le cours 1,avant de parler de l’intervalle de confiance. En effet,cela donne quatre intervalles différents (avec celui dela loi binomiale) et les élèves risquent de tout« mélanger » !• Le premier objectif est de savoir trouver unintervalle de fluctuation d’une fréquence d’uncaractère d’un échantillon de taille n , lorsque l’onconnaît la proportion p du caractère dans lapopulation.Pour répondre aux commentaires du programme,nous avons introduit la loi normale suivie par lafréquence aléatoire F, pour permettre de comprendrepourquoi on parle de 95 % , plus exactement de laprobabilité que la fréquence soit dans cet intervalle defluctuation p – 1 ; p + 1 .n nNous rappelons ces commentaires :« On peut faire observer qu’en approchant la loibinomiale par la loi normale de même espérance n×pet d’écart type p 1 − p × n , on est conduit àl’intervalle p − 1,96p 1−pn; 1 + 1,96qui est inclus dans l’intervalle p – 1 n ; p + 1 np 1−pn. »Ceci nous a semblé bien trop compliqué pour desélèves de STMG dans le cadre d’un enseignement de2 h par semaine.Comme cette démonstration n’est pas du programmede Term STMG, on trouvera cependant une approcheet un lien avec la loi normale et la loi binomiale enexercice 35 p. 189.Si le programme indique que l’intervalle de fluctuationest « à au moins 95 % », cela n’est pas toujours vraipour une probabilité inférieure à 0,2 ou supérieure à0,8 . L’intervalle de fluctuation de la fréquence n’estpas adapté pour ces valeurs de la proportion.De plus, ce type de recherche n’est vrai que souscertaines conditions, au programme en ES et S maisnon demandées en STMG :n ≥ 30 , n × p ≥ 5 et n × ( 1 – p ) ≥ 5 .L’exercice 36 montre que si ces conditions ne sont pasremplies, l’intervalle n’a pas de signification.C’est pourquoi nous n’avons pas gardé le vocabulairelourd « à au moins » du programme. La probabilitéque la fréquence du caractère soit dans cet intervalleest proche de 0,95 , mais pas forcément supérieure.Voir ressources du site Eduscol : Proba Bin page 33.Pour une démonstration : voir activités du chapitre 8du Déclic Term ES pages 233 à 235.L’intervalle proposé dans le cours, beaucoup plusfacile à déterminer, va permettre une prise dedécision semblable à celle vue en première pour la loibinomiale (revue au chapitre 5 p. 120 et 121).Cependant, la loi binomiale étant le « vrai » modèle,on perd en précision. Cette prise de décision reposesur le raisonnement suivant.Si la proportion vaut p , on a, en gros, au moins 95 %de chances que le prélèvement d’un échantillon detaille n conduise à une fréquence f du caractère danscet échantillon située dans l’intervalle [ a/n ; b/n ].On sait bien que dans ce cas, compte tenu du hasard,la fréquence réellement observée f n’est pasnécessairement égale à p , mais qu’elle fluctue dansun voisinage de p , appelé justement intervalle defluctuation. Un intervalle de fluctuation est donc unintervalle où l’on « s’attend » à trouver la fréquenceobservée f , si l’hypothèse que la proportion soit p estbonne.Livre du professeur - Mathématiques Term STMG © Hachette Livre 2013 99


En conséquence, si la proportion vaut p , il y a très peude chances (environ au plus 5 % des échantillons) quecette fréquence observée f soit hors de l’intervalle defluctuation. Donc si elle est à l’extérieur de l’intervalle,il est cohérent de penser que ce n’est plus le seul faitdu hasard cette fois-ci, mais que c’est bien plutôt lesigne que l’hypothèse que la proportion soit p n’estpas la bonne.• Le second objectif est l’estimation de la proportionconnaissant une fréquence observée dans unéchantillon de taille donnée.Cette estimation est d’autant meilleure que la taille del’échantillon est grand, puisque l’amplitude del’intervalle est 2 × 1 n .Attention, le niveau de confiance à 95 % n’est pas uneprobabilité. Il est faux de dire que « la proportion pest dans cet intervalle de confiance avec uneprobabilité de 0,95 » ! Le commentaire du programmeprésente une ambiguïté :« Cet intervalle contient la proportion dans au moins95 % des cas pour n grand, ce qui peut être illustrépar simulation.»L’entier n dont on parle n’est pas la taille del’échantillon prélevé, mais le nombre de prélèvementsde même taille effectués dans la population. Ce sontces prélèvements que l’on simule en grand nombre. Etdans au moins 95 % des prélèvements, la proportioncherchée est dans l’intervalle obtenu à l’aide de lafréquence observée dans ce prélèvement. Cettesimulation est proposée dans l’Atelier 2.Avant de commencerCes QCM, assez divers, apportent des regards sur lesacquis nécessaires pour entrer dans ce chapitre.Le QCM 2 sur l’inverse de la fonction racine carréepermet de visualiser les valeurs de 1 très proche de 0xpour de grandes valeurs de x .Les QCM 4 et 5 abordent la fluctuation de lafréquence.QCM 1 : Réponse b) 81= 0,225 .360Bien lire l’énoncé et les données.On parle de proportion puisque l’étude porte sur lapopulation totale « 360 ménages au total ».QCM3 : Réponse a) On retrouve l’intervalle defluctuation de la loi normale vue au chapitre 5.b) Faux, P(X ≤ 0,4 ) = 0,5 , car µ = 0,4 .c) Faux, il faut s’écarter de deux écarts types pourobtenir 0,95 .QCM 4 : Réponse b) C’est la notion de fluctuation dela fréquence: la probabilité d’obtenir une des faces est0,25 , donc il est normal d’obtenir une fluctuation dela fréquence autour de cette valeur.Si on augmente le nombre de lancers, fairecomprendre que les fréquences vont se rapprocher de0,25 , mais la somme reste égale à 1 . La face 4 n’estpas plus fréquente si le dé est équilibré.Les résultats ont été obtenus par simulation surtableur de 10 000 fois =Alea.entre.bornes(1;4) dansles cellules de A1 à T 500 , puis :QCM 5 : Réponse b) entre 40 et 90 sur 100, on a 0,5 .a) Faux, car rien n’est certain avec un échantillon.c) Faux, on ne peut connaître le nombre moyen sansconnaître le nombre de cafés total.CoursÉchantillonnage et prise de décisionÉtude d’une situationAprès bien des réflexions, nous avons choisi de fairetrès simple, pour ne pas effrayer les élèves. L’écarttype d’une loi binomiale n’a pas à être connu. Aussiavons-nous choisi d’admettre la valeur de l’écart typede la loi normale qui approche la loi binomiale.On fait apparaître ces valeurs sur le logiciel GeoGebra.QCM 2 : Réponse a) On reviendra sur ce graphiquepour parler de l’amplitude des intervalles vus dans cechapitre.f(100) = 0,1 . On peut faire chercher les valeursentières qui donnent des valeurs f(n) « rondes » àl’aide de la calculatrice :25 , 64 , 100 , 256, 400, 625 , 1024 , 2500 …Livre du professeur - Mathématiques Term STMG © Hachette Livre 2013 100


Rappel : la loi binomiale associée au nombre declients satisfaits a pour espérance µ’ = n × p = 48et pour écart type σ’ = n p q = 12 ≈ 3,4641 .La variable aléatoire correspondant à la fréquencepour un échantillon de taille n = 64 a pour espéranceµ = 48σ ′= 0,75 et écart type σ = = n p q64 n n=p qn .En étudiant la fonction f : x ⟼ x ( 1 – x ) , on peutmontrer que son maximum est 1/4 sur [ 0 ; 1 ] .Doncp [ 0 ; 1 ] , p ( 1 – p ) ≤ 1 etp q≤ 1 × 1 .4 n 2 nOn obtient ainsi l’écart type σ de la loi normaleapprochant la loi de la fréquence aléatoire F .1.Tableau des fréquences observéesMathilda Selma Duncantrès satisfait 46 55 57fréquence f 0,71875 0,8594 0,89062.a) σ = 1= 0,0625 et l’intervalle I = [ 0,625 ; 0,875 ] .16b) On calcule à la calculatrice ou à l’aide du logiciel :P ( X I ) = 0,9545 ≈ 0,95 .c) Les fréquences ne sont pas toutes dans l’intervalle.On peut remarquer que, pour n = 64, le nombre declients satisfaits est dans l’intervalle [ 40 ; 56 ] avecune probabilité de 0,95 .d) Duncan n’est pas dans la «normal» , mais cela restepossible.Objectif des exercices : La prise de décision est lafinalité de cette recherche d’intervalle. Cetteapplication est très intéressante et fait l’objet de trèsnombreux exercices. Les élèves peuvent ainsi en saisirtout l’intérêt dans le cadre des prises de décisions.On rappelle que :• soit on rejette l’hypothèse faite sur la proportionavec un risque d’erreur de 5 % ;• soit on ne rejette pas l’hypothèse, mais cela ne veutpas dire que l’on admet la proportion avec certitude.On peut indiquer que faire l’hypothèse d’uneproportion p = 0,41 au lieu de 0,4 ne va pas beaucoupchanger le problème, surtout sur un échantillon detaille 100 !Une difficulté est de comprendre comment donner unencadrement de cet intervalle : c’est l’un des conseilsde l’exercice résolu et le thème de l’exercice 7Intervalle de confiance d’une proportionÉtude d’une situationL’estimation d’une proportion est une finalitéessentielle de l’enseignement de probabilité deTerminale.Cependant, attention, nous rappelons que le niveaude confiance (à 95 %) n’est pas lié à une probabilité !La probabilité que la proportion p soit dans l’intervallede confiance J est 1 ou 0 ! La proportion p dans lapopulation totale n’est pas une variable aléatoire,c’est une valeur, certaine, mais que l’on ne connaîtpas.Nous espérons que les 20 prélèvements effectuésdonneront une idée suffisante aux élèves du niveaude confiance à 95 % . L’atelier 2 peut permettre dereprendre cette notion.1. a) Comme 1 = 0,1 ; on obtient J = [ 0,27 ; 0,47 ] .nb) Voir le fichier en téléchargement pour faired’autres simulations.Tous les intervalles ont pour amplitude 0,2 = 2 n .2. 19 intervalles contiennent p = 0,4 , ce quicorrespond à 95 % des intervalles obtenus aprèssimulation.On peut terminer la situation par l’estimation de laproportion à l’aide d’une fourchette : « entre 27 % et47 % , avec une amplitude de 20 points depourcentage » ; ou bien « 37 % à + ou – 10 points depourcentage ».De plus si Mme Zoé a un fichier de 2 600 clients (parexemple) elle peut estimer avoir entre 702 et 1 222clients intéressés.Il existe un intervalle de confiance non simplifié :f − 1,96f 1−fn; 1 + 1,96f 1−fNous n’avons pas voulu présenter la démonstration :p – 1 n ≤ f ≤ p + 1 n f – 1 n ≤ p ≤ f + 1 nElle n’a que peu d’utilité pour les élèves de STMG etrisque d’apporter la confusion entre les deuxintervalles.Objectifs des exercices : Déterminer une estimationd’une proportion inconnue, connaissant unefréquence observée sur un échantillon de tailledonnée.n.Livre du professeur - Mathématiques Term STMG © Hachette Livre 2013 101


Bien comprendre que plus la taille des échantillons estgrande, plus l’estimation est précise. Mais rien n’estcertain.Le cas des élections où la proportion attendue sur latotalité des votants est de 50 % est un exercice desplus utiles.Atelier TICETaux de satisfactionBut : Créer un algorithme pour déterminer unintervalle de fluctuation et appliquer une prise dedécision.On entre la proportion p prise en hypothèse (ouconnue), la taille de l’échantillon n et la fréquenceobservée f sur l’échantillon.Un tel atelier permet aux élèves de comprendre lesattendus du chapitre à ce sujet. Ne pas hésiter à faireentrer le programme sur la calculatrice.Rappel : pour p et f, on peut rentrer la division del’effectif par l’effectif total si besoin.a) b) Tableau des résultats obtenus sur tableurRésultats pour Axel et Dylan sur CasioRésultat pour Bryan et Elodie sur TITest Logique ou s’obtient parOn peut utiliser If… Then… Else… End pour laconclusion sur calculatrice :Sur CasioSur TiRépondre par oui ou non au hasardBut : réaliser une simulation d’un grand nombre deprélèvements de même taille sur une population où laproportion d’un caractère est 0,5 (répondre par oui ounon au hasard). On obtient ainsi un grand nombred’intervalles de confiance à partir des fréquencesobservées sur les prélèvements et on peut ainsivérifier que 95 % des échantillons contiennent 0,5 .Le plus sera de faire réaliser l’atelier par les élèves, etrécolter leurs résultats (nombre de prélèvements necontenant pas 0,5). On peut alors obtenir trèsrapidement 20 × 25 × 20 = 10 000 échantillons …Comme la proportion attendue est en A1, on peutsimuler une autre proportion.1. a) La 7 e simulation donne [ 0,36 ; 0,56 ] et la 9 edonne [ 0,28 ; 0,48 ] .b) La 9 e simulation donne un intervalle de confiancequi ne contient pas la proportion p = 0,5 . Donc pour95 % des simulations, la proportion 0,5 appartient àl’intervalle de confiance.2. Récolte des résultats dans la classe.Ce qui permet de faire comprendre l’idée d’un niveaude confiance à 95 % .Convocation par la directionBut : Obtenir un intervalle de fluctuation et écrire untest sur un tableur. Lire «Manon» au lieu de Marion.1. En B5 on saisit la formule = B4 / B3 .En B6 on saisit = $B$1 – 1 / RACINE( B$3 )et en B7 on saisit = $B$1 + 1 / RACINE( B$3 ) .2. Bien vérifier la place de $ .3. =SI ( OU ( ( B5B7 ) ) ; «rejet» ; «ok» )L’instruction OU ( ( B5B7 ) )Livre du professeur - Mathématiques Term STMG © Hachette Livre 2013 102


correspond au test «la fréquence observée dansl’échantillon, calculée en B5 , n’appartient pas àl’intervalle de fluctuation défini par [ B6 ; B7 ] .En F8, cela devient :=SI ( OU ( ( F5F7 ) ) ; «rejet» ; «ok» )Pour aller plus loin : demander la formule à écrirepour écrire « OK » si la réponse est vraie.Puis , pour l’atelier 2 , le test placé en cellule AA2pour savoir si p [ a ; b ] .=SI ( ET ( ( $A$1>=Y2 ) ; ( $A$1


2. Résultat sur Casio :Donc I = [ 0,637 ; 0,703 ] .ex8 n = 800 ; f = 0,40 . I = [ 0,36 ; 0,44 ] .ex9 L’intervalle de fluctuation à 95 % de la fréquence estcentré en la proportion connue. On a :I = [ 0,32 ; 0,42 ] et I ‘ = [ 0,56 ; 0,66 ] .D’où p = 0,32+0,422= 0,37 et p’ = 0,56+0,662= 0,61 .ex10 a) p = 0,478+0,546= 0,512 .2b) 2 = 0,562 – 0,462 2 = 0,1 n = 20 n = 400 .n nex11 a) Plus la taille n de l’échantillon est grande, plusl’amplitude de l’intervalle de fluctuation, à environ 95 % ,de la fréquence est petite. D’où le tableau suivant :intervalle I1 I2 I3 I4amplitude 0,052 0,072 0,096 0,064taille del’échantillon1500 800 450 1000ex12 La proportion de français qui connaissent le label est67 % . Attention l’information 84 % n’a pas de rôle à jouerdans l’exercice.a) p = 0,67 , n = 400 et 400 = 20 .D’où l’intervalle de fluctuation de la fréquence, à 95 % :[ 0,67 - 1; 0,67 - 1] = [ 0,62 ; 0,72 ] .20 20b) La fréquence des réponses « oui » dans un échantillon de400 personnes est dans l’intervalle de fluctuation de lafréquence [ 0,62 ; 0,72 ] avec une probabilité d’au moins0,95 . On a : 0,62 × 400 = 248 et 0,72 × 400 = 288 .Cela indique que pour un échantillon de 400 clients, legérant peut s’attendre à un nombre de « oui » comprisentre 248 et 288 avec une probabilité d’au moins 0,95.Fourchette : de 248 à 288 clients.ex13 a) p = 0,3 et n = 250 . I = [ 0,236 ; 0,364 ] .b) 0,236 × 250 = 59 et 0,364 × 250 = 91 .On estime que pour 250 dossiers concernant un accidentmortel, le nombre d’accidents où l’alcool est un facteurprésent est compris entre 59 et 91 .ex14 On suppose que l’on a la proportion p = 0,48 . Dans unéchantillon de taille n = 1 000, l’intervalle de fluctuation dela fréquence des personnes pratiquant un sport est :[ p – 1 ; p + 1 ] = [ 0,48 - 1; 0,48 + 1]n n 1000 1000= [ 0,448 ; 0,512 ] .La fréquence observée dans un échantillon de taille 1 000est f = 0,509 .On constate que 0,509 est dans l’intervalle. Doncl’hypothèse sur la proportion p = 0,48 n’est pas rejetée.ex15 p = 0,43 et n = 400 . I = [ 0,38 ; 0,48 ] .Fréquence observée : f = 165400 = 0,4125 .Comme la fréquence observée dans l’échantillon appartientà l’intervalle I , alors l’hypothèse sur la proportion p = 0,43n’est pas rejetée.ex16 p = 0,77 .1. Arrondissement A : n = 400 et I A = [ 0,72 ; 0,82 ] .Arrondissement B : n = 500 et I B = [ 0,725 ; 0,815 ] .Arrondissement V : n = 1000 et I C = [ 0,738 ; 0,802 ] .2. Tableau des résultatsarrondissement A B C295fréquence=356=795=400 500 1000observée 0,7375 0,712 0,795L’échantillon de l’arrondissement B n’est pas représentatif,en effet la fréquence observée 0,712 I B .ex17 1. a) En D3 on saisit = C3 – 1/racine( B3 ) .b) En E3 on saisit = C3 + 1/racine( B3 ) .2. a) Résultats sous Excel : voir 3)b) En G3 on saisit = F3 / B3 .3. Résultats sous Excelex18 On a p = 0,13 et n = 100 .On détermine l’intervalle I , intervalle de fluctuation à 95 %de la fréquence des enfants de 11 à 14 ans ayant déjà euune crise d’asthme dans un échantillon de taille 100 .I = p − 1 ; p + 1 = [ 0,03 ; 0,23 ] .n nLa fréquence observée par le médecin est f = 0,19 etcomme 0,19 I , il est inutile de mettre en place unenouvelle investigation.ex19 On a p = 0,18 , proportion d’enfants entre 3 et 10 ansdans la totalité de la population de la commune.1. a) n = 860 dans le 1 er quartier, alors I ≈ * 0,14 ; 0,22 ] .b) On obtient 0,14 × 860 ≈ 120 et 0,22 × 860 ≈ 190 .Si l’on veut faire un cadeau à chaque enfant sans risqued’en manquer ou d’en prévoir un trop grand nombre, avecLivre du professeur - Mathématiques Term STMG © Hachette Livre 2013 104


une probabilité d’environ 0,95 , il faut prévoir entre 120 et190 cadeaux.2. On a n = 600 dans le second quartier , alors :I ≈ * 0,13 ; 0,23] .0,13 × 600 = 78 et 0,23 × 600 = 138Il faut prévoir entre 78 et 138 cadeaux pour pouvoir endonner un à chaque enfant dans ce second quartier avecune probabilité de 0,95 . Par conséquent avec 80 cadeauxprévus, l’organisateur aura assez de cadeaux.3. On a n = 240 , donc I ≈ * 0,11 ; 0,25 + .0,11 × 240 ≈ 26 et 0,25 × 240 = 60 .L’organisateur aura assez de cadeaux pour ce 3 e quartier,avec une probabilité de 0,95 .Remarque : Pour ce 3 e quartier, si l’on modifie l’arrondi desbornes de l’intervalle de fluctuation de la fréquence, laconclusion change. En effet si I ≈ * 0,115 ; 0,245 ] , lafourchette du nombre de cadeaux à prévoir devient de 27 à59 compris . Dans ce cas, l’organisateur n’aura pas assez decadeaux, car 26 [ 27 ; 59 ] .ex20 1. On considère la population des jeunes de 15-24 ansau troisième trimestre 2012.Par lecture du tableau, la proportion p des jeunes auchômage dans ce groupe est connue et p = 0,242.On considère un échantillon de taille n = 400 .On détermine l’intervalle de fluctuation, à environ 95 % , dela fréquence des jeunes de 15-24 ans au chômage pour unéchantillon de taille 400 . I = [ 0,192 ; 0,292 ] .La fréquence observée dans l’échantillon interrogé enSeptembre 2012 est f = 93= 0,2325 . Or 0,2325 I .400On en déduit que cet échantillon est représentatif desrésultats nationaux.2. On considère la population des femmes actives de 25 à49 ans au troisième trimestre 2012. La proportion desfemmes au chômage dans ce groupe est p = 0,093 .On détermine l’intervalle de fluctuation, à environ 95 % , dela fréquence des femmes actives de 25 à 49 ans auchômage pour un échantillon de 1000 femmes.I = [ 0,061 ; 0,125 ] . Or f observée = 0,121 I .L’échantillon observé est représentatif des résultatsnationaux.3. On considère la population des hommes de 15 à 24 ansau deuxième trimestre 2012. Parmi eux la proportion dechômeurs est p = 0,234 .On détermine l’intervalle de fluctuation, à environ 95 % , dela fréquence des hommes de 15-24 ans au chômage dansun échantillon de taille 400.I = [ 0,184 ; 0,284 ] . Or f observée = 208= 0,52 I .400L’échantillon observé n’est pas représentatif des résultatsnationaux.Pour aller plus loin : On peut proposer aux élèves de créereux-mêmes des questions à partir du tableau donné à poseraux autres élèves de la classe pour s’exercer.ex21 On note [ a ; b ] l’intervalle de fluctuation à 95 % desfréquences. Tableau des résultats sur feuille de calcul.ex22 Réponse b. La fréquence observée est f = 0,75 dansun échantillon de taille n = 100 . n = 10 et1n = 1= 0,1 .10Donc une estimation de la proportion p est l’intervalle deconfiance, à 95 % :J = [ f – 1 ; f + 1 ] = [ 0,65 ; 0,85 ] .n nex23 QCM Réponse b.ex24 Réponse c.On connaît la fréquence f = 0,52 des intentions de votepour le candidat dans un échantillon de taille n = 900 .On estime que la proportion p de personnes qui vont voterpour ce candidat est dans l’intervalle de confiance à 95 % :J = [ f - 1 ; f + 1 ] = [ 0,52 - 1; 0,52 + 1]n n 900 900= [ 0,486 ; 0,554 ] .ex25 a) On a f = 0,47 et n = 400 . Donc :f − 1 ; f + 1 = [ 0,42 ; 0,52 ] .n nb) D’après ce sondage, on estime que dans cette commune,entre 42 % et 52 % des habitants sont mécontents del’organisation des transports.ex26 On note [ a ; b ] l’intervalle de confiance dans chacundes cas suivants. Tableau des résultatsEn cellule C2, on a saisi la formule = A2 – 1 / RACINE( B2 ) .En cellule D2, on a saisi = A2 + 1 / RACINE( B2 ) .Puis on a recopié vers le bas jusqu’en ligne 6 pour obtenirles bornes de l’intervalle de confiance dans chaque cas.ex27 1. a) On a f = 340= 0,85 et n = 400 . Donc :400f − 1 n ; f + 1 n= [ 0,8 ; 0,9 ] .Livre du professeur - Mathématiques Term STMG © Hachette Livre 2013 105


) Dans l’ensemble des véhicules assurés depuis au moinsun an dans ce groupe d’assurances, entre 80 % et 90 %n’ont pas eu de sinistre.2. Le résultat précédent signifie qu’au maximum 20 % devéhicules auront un sinistre, et au minimum 10 % .Ainsi, on peut prévoir que, dans l’année à venir, entre 10 %et 20 % des véhicules auront un sinistre, au niveau deconfiance à 95 % .0,255 × 32 400 = 8 262 ≈ 8 260 .0,345 × 32 400 = 11 178 ≈ 11 180 .On estime alors qu’entre 8 260 et 11 180 ménages de cetteville possèdent deux voitures.ex31 a) Le sondage sur un échantillon de 150 lycéens donne35 % des intentions de vote à Émilie.D’où f = 0,35 et n = 150 . Ainsi, pour Emilie :ex28 1. L’intervalle de confiance de la proportion p depersonnes ayant un Smartphone S, au niveau de confianceà 95 % , est centré en la fréquence observée et les bornes1sont à + ou – du centre.Commenf = 110= 0,44 . La taille est n = 250 .2501n = 1250≈ 0,0632 ≈ 0,07 , on prend la valeurapprochée par excès pour donner la fourchette :44 % à + ou – 7 points de pourcentage.2. 312 personnes sur 650 possède un Smartphone A :f = 312= 0,48 . n = 650 et 1= 1≈ 0,039 ≈ 0,04 .650 n 650Donc la proportion de personnes ayant un Smartphone Aest estimée à 48 % à + ou – 4 points de pourcentage.3 Comme chacun des intervalles de confiance contient laproportion p = 0,5, alors les deux marques peuventannoncer que 50 % des personnes ont un Smartphone deleur marque, avec un risque d’erreur de 5 % .ex29 1. On a f = 180= 0,2 et n = 900 . Donc :900f − 1 ; f + 1 ≈ * 0,166 ; 0,234 ] .n n2. Selon ce sondage, on estime que dans l’entreprise, lenombre de salariés intéressés par ce bilan de compétenceest compris entre environ 16 % et 24 % , au niveau deconfiance à 95 % . Par conséquent le souhait que ce bilande compétences concerne 30 % des salariés a très peu dechance de se réaliser.Attention, on ne peut pas chiffrer la probabilité que cela seréalise !ex30 On utilise les données de l’échantillon de 500ménages parmi lesquels 30 % possèdent deux voitures.Ainsi f = 0,3 et n = 500 . D’où l’intervalle de confiance, auniveau de confiance à 95 % :f − 1 ; f + 1 ≈ * 0,255 ; 0,345 ] .n nD’après l’intervalle de confiance précédent, on estime quedans cette ville, la fourchette des ménages qui possèdentdeux voitures est de 25,5 % à 34,5 % .I E = f − 1 ; f + 1 ≈ * 0,268 ; 0,432 ] .n nb) À partir du même échantillon, Julien obtient 23 % desintentions de vote. D’où I J ≈ * 0,148 ; 0,312 ] .c) Émilie peut donc espérer obtenir entre 26,8 % et 43,2 %des voix et Julien peut espérer en obtenir entre 14,8 % et31,2 %.Les deux fourchettes de résultats ne sont pas disjointes. Onne peut pas affirmer qu’Émilie aura plus de voix que Julien.ex32 1. a) La population étudiée est constituée de tous lesbénévoles d’une association d’une grande ville.b) La proportion est connue : p = 0,75 . C’est celle desbénévoles prêts à poursuivre leur bénévolat.c) La taille de l’échantillon considéré est n = 60 .d) Intervalle de fluctuation à 95 % de la fréquence f desbénévoles prêts à poursuivre leur action :I = p − 1 ; p + 1 = 0,75 − 1 ; 0,75 + 1 n n60 60≈ * 0,62 ; 0,88 ] .Sur un très grand nombre d’échantillons de 60 personnes,entre 62 % et 88 % des bénévoles sont prêts à continuerleur action.2. a) On considère un échantillon de taille n = 100 et lafréquence des bénévoles qui acceptent de participer ledimanche est f = 0,45 . D’où f − 1 ; f + 1 n n= 0,45 − 1; 0,45 + 1= [ 0,35 ; 0,55 ] .10 10À partir de cet intervalle de confiance, on estime qu’entre35 % et 55 % des bénévoles de cette association vontaccepter de participer le dimanche.b) À partir d’un échantillon de taille n , l’amplitude del’intervalle de confiance est 2 . On cherche n tel que :nLivre du professeur - Mathématiques Term STMG © Hachette Livre 2013 1062n= 0,1 n = 20 n = 400 .Il faut interroger 400 bénévoles pour que l’amplitude del’intervalle de confiance, à 95 %, de la proportion, soit égaleà 0,1 .ex33 1. a) 1 = 1≈ 0,05976 ≈ 0,060 .n 280b) Le premier mois, la proportion de clients satisfaits estestimée à 72,1 % à + ou – 6 points de pourcentage.2. Tableau des résultats sous Excel :en C3 =B3/A3 ; en F3 = C3 au format % ;en D3 = C3 – 1 / RACINE( A3 )et en E3 = C3 + 1 / RACINE( A3 )


Jacques Chirac et Jean-Marie Le Pen et ce sondage nepermettait pas de départager les trois candidats.3. Pauline peut donner l’argument suivant : en observantchaque intervalle de confiance, quel qu’il soit, il a toujoursune partie commune avec les autres intervalles deconfiance. En particulier la valeur 78 % est commune à tousles intervalles.Pour être sûr que, pour 95 % des échantillons, le services’améliore, il aurait fallu trouver des intervalles deconfiance disjoints avec des bornes de plus en plus grandes.ex34 1. a) Intervalles de confiance à partir du sondageréalisé par l’institut IPSOS :Jean Marie Le Pen :0,135 − 1; 0,135 + 11000Jacques Chirac :10000,195 − 1; 0,195 + 11000Lionel Jospin :1000≈ * 0,103 ; 0,167 ] .≈ * 0,163 ; 0,227 ] .0,175 − 1; 0,175 + 1≈ * 0,143 ; 0,207 ] .1000 1000b) D’après la question précédente, l’institut IPSOS a donnéles intentions de vote suivantes, à 1 % près :• entre 10 % et 17 % pour Jean-Marie Le Pen ;• entre 16 % et 23 % pour Jacques Chirac ;• entre 14 % et 21 % pour Lionel Jospin.2. D’après la question 1. b. les résultats du premier toursont alors compatibles avec les intervalles de confiancepour les trois candidats puisque les résultats des troiscandidats appartiennent respectivement aux intervalles deconfiance donnés en question 1. b.3. Le CSA donne f = 0,14 comme fréquence de votants pourJean Marie Le Pen.a) A partir de cette fréquence on détermine l’intervalle deconfiance, au niveau de confiance à 95 %, de la proportionde votants pour Jean Marie Le Pen.J = 0,14 − 1; 0,14 + 1≈ * 0,108 ; 0,172 ] .1000 1000D’après cet intervalle, Jean Marie Le Pen peut obtenir entre11 % et 18 % des voix. Les pourcentages obtenus sontlégèrement supérieurs à ceux obtenus à la question 1.4. Le résultat du premier tour de l’élection présidentielle du21 Avril 2002 est prévisible à partir du sondage de l’institutIPSOS : les deux candidats ayant obtenu le plus de voix sontex35 1. I n = 0,5 − 1 ; 0,5 + 1 n nOn retrouve l’intervalle de fluctuation à 95 % de lafréquence des internautes qui utilisent internet pourrechercher de l’information sur l’émission qu’ils regardentdans un échantillon de taille n.1= 1= 12 n 2 4002. Pour n = 400 , on a == 0,0252×20et I 400 = [ 0,45 ; 0,55 ] .On en déduit :[ A ; B ] = [ 400 × 0,45 ; 400 × 0,55 ] = [ 180 ; 220 ] .Le nombre de personnes qui recherchent de l’informationsur internet à propos de l’émission qu’elles regardent à latélévision va fluctuer entre 180 et 220 , sur 400 internautes.3. En utilisant la loi normale d’espérance µ = 0,5 et d’écarttype = 0,025, à la calculatrice, on obtient :P( f [ 0,45 ; 0,55 + )≈ 0,9545 .4. La variable aléatoire X suit la loi binomiale B( n ; 0,5 ) .Son écart type est donné par ‘ = n p (1 − p) .a) Comme p = 0,5, alors ‘= n × 0,5 ² =0,5 n .D’autre part n × = n × = 1 × n = 0,5 n .2 n 2 nOn a bien vérifié que ‘= n × .b) Pour n = 400, on calcule :1P( 180 X 220 ) = P( X 220 ) – P( X 179 ) ≈ 0,96 .Voir commentaires généraux en début de chapitre.ex36 1. On a n = 38 et p = 0,9 .a) On a bien n 30 , n × p = 34,2 5mais n × ( 1 – p ) = 38 × 0,1 = 3,8 < 5 .La troisième condition n’est pas vérifiée.On va voir que les calculs conduisent à des incohérencesb) p − 1 ; p + 1 = 0,9 − 1 ; 0,9 + 1n n38 38≈ * 0,737 ; 1,063 ] .Ce premier résultat n’est pas cohérent avec le fait qu’unefréquence ne dépasse pas la valeur 1.En multipliant les bornes par la taille de l’échantillon n = 38,on obtient l’intervalle [ 28 ; 41 ] .De même on ne peut pas trouver que, dans un grandnombre d’échantillons de taille 38, il y ait un effectifsupérieur à 38 individus vérifiant un caractère donné.Les informations données par l’intervalle de fluctuation à95 % ne sont pas exploitables si les trois conditions initialesne sont pas vérifiées.Ces conditions ne sont pas dans le programme officiel deSTMG. Les applications faites dans cette classe, où les.Livre du professeur - Mathématiques Term STMG © Hachette Livre 2013 107


échantillons sont de grandes tailles et les proportions entre0,2 et 0,8 restent bien dans les conditions.ex37 Texte en françaisUne société de hotline réalise un sondage de satisfaction :sur les 300 personnes de l’échantillon, 250 personnes sontsatisfaites.On note p la proportion de satisfaits dans l’ensemble desclients.Déterminer une estimation de p par un intervalle deconfiance à 95 % .ex38 QCM1. Réponse a. 2. Réponse c.3. Réponse b. Dans ce cas l’intervalle de fluctuation de lafréquence est I ≈ * 0,309 ; 0,393 ] .Fréquence observée : f = 245≈ 0,43 et f I .570ex39 QCM1. Réponse c. I = [ 0,56 - 1 ; 0,56 + 1 ] = [ 0,45 ; 0,67 ] .90 902. Réponse a. Fréquence observée f = 230= 0,575 .400Taille de l’échantillon n = 400 1 = 1= 1= 0,05 . Donc :n 400 20J = [ 0,575 -1; 0,575 + 1400 400] = [ 0,525 ; 0,625 ] .3. Réponse c. On multiplie les bornes de l’intervalle defluctuation de la fréquence par la taille de l’échantillon.0,42 × 400 = 168 et 0,52 × 400 = 208 .Ainsi le nombre de personnes ayant environ 15 € dans leurpoche, dans un échantillon de taille 400, est compris entre168 et 208, avec une probabilité d’au moins 0,95 .ex40 1. a) Résultatslycée n°1 n°2 n°3 n°4fréquence0,833 0,81 0,867 0,78des admisb) Dans l’ensemble des quatre lycées du département, on a757 admis pour 925 candidats, soit une fréquence de757≈ 0,818 .925Ou 81,8 % d’admis dans l’ensemble des quatre lycées.2. a) Avec n = 360 et f = 0,833 , Intervalle de confiance de laproportion d’admis d’après le lycée n° 1 :I 1 = [ 0,880 ; 0,886 ] .b) Avec n = 195 et f = 0,81 on obtient I 2 = [ 0,738 ; 0,882 ] .c) Tableau résumé des résultatslycéefréquence des intervalle deadmisconfiancen°1 0,833 [ 0,880 ; 0,886 ]n°2 0,81 [ 0,738 ; 0,882 ]n°3 0,867 [ 0,775 ; 0,958 ]n°4 0,78 [ 0,716 ; 0,844 ]3. Le taux de 84,5 % d’admis au BAC 2012 a été prévu dansles trois premiers intervalles de confiance précédents maispas dans le 4 e .ex41 1. a) La population étudiée est celle des bénévoles del’association France Bénévolat âgés de 25 à 39 ans.b) La taille de l’échantillon considéré est n = 150 .c) Fréquence observée dans cet échantillon f = 110150 ≈ 0,733 .2. a) 0,8 − 1 ; 0,8 + 1 = 0,8 − 1 ; 0,8 + 1 n n150 150≈ [ 0,71 ; 0,89 ] .b) La fréquence des bénévoles, qui jugent que la mentionde leur engagement bénévole sera un atout dans leur CV,dans un échantillon de 150 bénévoles âgés de 25 à 39 ans,est comprise entre 0,71 et 0,89 avec une probabilité de0,95 .Comme la fréquence observée dans l’échantillon est 0,833et que 0,833 0,8 − 1 ; 0,8 + 1 , on ne rejette150 150pas la proportion de 80 % présentée dans le tableau.3. a) Fréquence de bénévoles hommes ayant indiqué lebénévolat sur leur CV dans cet échantillon de 400 hommes :f = 330= 0,825 .400b) Intervalle de confiance à 95 % de la proportion debénévoles hommes l’ayant mentionné sur leur CV, à partirde cet échantillon de taille 400 :J = 0,825 − 1 ; 0,8 + 1 = [ 0,775 ; 0,875 ] .400 400c) La proportion de 0,77 n’est pas dans cet intervalle.Le taux de 77 % d’hommes qui mentionnent leur bénévolatsur leur CV n’a pas été prévu par l’intervalle de confianceétabli à partir de cet échantillon de taille 400 .ex42Partie ANombre total de voitures : n = 800 .Nombre de personnes venant en voiture :1 × 235 + 2 × 270 + 3 × 155 + 4 × 140 = 1800 .2. x = 2,25 et ≈ 1,061 .Partie B1. p = 18003750 = 0,48 .2. Réponse b. On calcule P( X ≥ 1 )= 1 – P( X = 0 ) = 1 – 0,52 4 ≈ 0,927 .Ou bien à la calculatrice :Partie C1. La fréquence observée des personnes ayant l’intentionde venir en voiture est f = 188= 0,47 .400On détermine J , l’intervalle de confiance à 95 % de laproportion p de personnes venant en voiture dansl’ensemble du public de ce concert.J = [ 0,47 -1; 0,47 + 1] = [ 0,42 ; 0,52 ] .400 4002. D’après la question 1. les organisateurs estiment que aumaximum 52 % des 3750 spectateurs viennent en voiture,soit 0,52 × 3750 = 1950 spectateurs.Livre du professeur - Mathématiques Term STMG © Hachette Livre 2013 108


Et 19503= 650 , c’est-à-dire que 650 places de parkingsuffiront, avec une confiance de 95% , si les spectateursviennent à 3 par voiture.ex43Partie A1. n = 400 et p = 0,1 . Donc :1I = 0,1 − ; 0,1 + 1= [ 0,05 ; 0,15 ] .400 4002. Le service ne prend pas de risque car le pourcentage depièces défectueuses est compris entre 5 % et 15 %, avecune probabilité de 0,95.La probabilité qu’un lot contiennent plus de 15 % de piècesdéfectueuses est d’environ 0,025 .Partie B1. On considère un échantillon de 200 pièces dans lequel 18pièces ont un défaut. On note f = 18= 0,09 .200On estime à partir de cet échantillon un intervalle deconfiance, à 95 % ,du pourcentage p de pièces ayant undéfaut dans la production :1J = 0,09 − ; 0,09 + 1≈ * 0,019 ; 0,161 ] .200 200Selon cet échantillon, on estime que la productioncomportera entre 1,9 % et 16,1 % de pièces défectueuses.2. a) On considère un échantillon de 950 pièces dont 103sont défectueuses. Soit f = 103950 ≈ 0,108 .On détermine un intervalle de confiance, à 95 %, dupourcentage p de pièces ayant un défaut dans laproduction :J = 0,108 − 1 ; 0,108 + 1 ≈ * 0,075 ; 0,141 ] .950 950b) Pour tenir compte de cette estimation, le service devraitannoncer un pourcentage de pièces défectueuses comprisentre 7,5 % et 14,1 % .Partie C1. P(A) = 0,5 et P A (D) = 0,1 . P( A D ) = 0,5 × 0,1 = 0,05 .2. Arbre complété par les probabilités3. P( B ⋂ D ) = 0,3 × 0,08 = 0,024 .P(D) = P( A ⋂ D ) + P( B ⋂ D ) + P( C ⋂ D ) =0,05 + 0,024 + 0,05 = 0,125 .P(B ∩ D)4. P D (B) = = 0,024≈ P(D) 0,124 0,194 .Livre du professeur - Mathématiques Term STMG © Hachette Livre 2013 109

Hooray! Your file is uploaded and ready to be published.

Saved successfully!

Ooh no, something went wrong!